Business > Final Exam Review > Business 100 | Over 340 Correct Q&A /(GRADED A+) (All)

Business 100 | Over 340 Correct Q&A /(GRADED A+)

Document Content and Description Below

Chapter 1 The U.S. Business Environment 25) Which environment is NOT an important dimension of a business organization's external environment? A) the political-legal environment B) the socioc... ultural environment C) the technological environment D) the corporate cultural environment E) the global business environment Answer: D Explanation: D) The external environment consists of everything outside an organization's boundaries that might affect it. The corporate cultural environment is internal to the organization. Page Ref: 6-7 Difficulty: Easy Objective: 1.2 Learning Outcome: Compare and contrast different economic systems Skill: Concept 26) What term denotes a nation's basis for allocating its resources among its citizens? A) capital structure B) economic system C) ownership processes D) distribution network E) national regulations Answer: B Explanation: B) An economic system is a nation's system for allocating its resources among its citizens, both individuals and organizations. Page Ref: 7 Difficulty: Easy Objective: 1.3 Learning Outcome: Compare and contrast different economic systems Skill: Concept 27) What is the process for converting government enterprises into individually owned firms known as? A) production allocation B) privatization C) entrepreneurship D) demand assessment E) profit maximization Answer: B Explanation: B) Privatization is the process of converting government enterprises into privately owned companies, which will then have the right to run such a business for profit. Page Ref: 10 Difficulty: Easy Objective: 1.3 Learning Outcome: Compare and contrast different economic systems Skill: Concept 28) What is the point at which the supply curve and the demand curve intersect on a graph? A) equilibrium price B) decision point C) surplus price D) perfect price E) parity point Answer: A Explanation: A) The equilibrium price is determined by the shape of the supply curve and the demand curve when plotted by amount available and price. The equilibrium price is set at the point at which the supply curve and the demand curve intersect. Page Ref: 12 Difficulty: Easy Objective: 1.4 Learning Outcome: Discuss strategies for setting and adjusting prices Skill: Concept 29) What is the price at which the quantity of goods demanded and the quantity of goods supplied are equal? A) the going rate B) the margin rate C) the market price D) the optimum price E) the cost price Answer: C Explanation: C) The market price is set by the demand and supply for a given good. It is defined as the price at which the quantity of goods demanded and the quantity of goods supplied are equal. Page Ref: 12 Difficulty: Easy Objective: 1.4 Learning Outcome: Discuss strategies for setting and adjusting prices Skill: Concept 30) What occurs when the quantity demanded exceeds the quantity supplied? A) demand deficit B) surplus C) equilibrium point D) shortage E) supply schedule Answer: D Explanation: D) When demand exceeds supply, there will not be enough goods available to meet the demand for them. This is known as a shortage. Page Ref: 12 Difficulty: Easy Objective: 1.4 Learning Outcome: Compare and contrast different economic systems Skill: Concept 31) Which of the following does NOT represent a degree of competition in a private enterprise system? A) oligopoly B) socialism C) monopoly D) perfect competition E) monopolistic competition Answer: B Explanation: B) Competition is the economic condition in which there are certain demands that can be met by more than one business, so businesses in the same industry compete for the same resources or customers. Page Ref: 14 Difficulty: Easy Objective: 1.4 Learning Outcome: Compare and contrast different economic systems Skill: Concept 32) Which of the following exists when an industry or market has only one producer? A) competition B) oligopoly C) monopoly D) communism E) socialism Answer: C Explanation: C) A monopoly is a market or industry in which there is only one producer that supplies a certain product. Monopolies in the United States economy are either discouraged or regulated so that prices are not too high. Page Ref: 16 Difficulty: Easy Objective: 1.4 Learning Outcome: Compare and contrast different economic systems Skill: Concept 33) Which term refers to the pattern of short-term ups and downs in an economy? A) aggregate output B) business cycle C) standard of living D) balance of payments E) demand and supply schedule Answer: B Explanation: B) The business cycle is the short-term pattern of economic expansions and contractions. Page Ref: 17 Difficulty: Easy Objective: 1.5 Learning Outcome: Compare and contrast different economic systems Skill: Concept 34) Which term refers to the total quantity and quality of goods and services that people living in an economic system can purchase? A) business cycle B) demand and supply schedule C) standard of living D) aggregate output E) consumer price index Answer: C Explanation: C) The standard of living is the total quantity and quality of goods and services that people can purchase with the currency used in their economic system. Standard of living is determined by economic conditions such as inflation and purchasing power. Page Ref: 17 Difficulty: Easy Objective: 1.5 Learning Outcome: Explain how economic performance is monitored Skill: Concept 35) Which term refers to the total value of all goods and services produced within a given period by a national economy? A) standard of living B) aggregate output C) gross domestic product D) gross national produce E) purchasing power parity Answer: C Explanation: C) Gross domestic product is the total value of all goods and services produced within a given period by a national economy through domestic factors of production. Page Ref: 17 Difficulty: Easy Objective: 1.5 Learning Outcome: Explain how economic performance is monitored Skill: Concept 36) Which of the following measures indicates the prices of typical products purchased by American consumers living in urban areas? A) GDP per capita B) standard of living C) consumer price index D) purchasing power parity E) Big Mac index Answer: C Explanation: C) The consumer price index is a measure of the prices of typical products purchased by consumers living in urban areas. It is used as a comparative standard, to tell whether inflation or deflation is occurring, for example. Page Ref: 22 Difficulty: Easy Objective: 1.5 Learning Outcome: Explain how economic performance is monitored Skill: Concept 37) What does productivity measure? A) the total value of all goods and services produced by a national economy B) how much a system produces with the resources needed to produce it C) how much output is necessary to produce a certain level of demand D) the standard of living relative to purchasing power parity E) how much gross national product results from inputs of labor Answer: B Explanation: B) Productivity is a measure of economic growth that compares how much a system produces with the resources needed to produce it. Page Ref: 19 Difficulty: Easy Objective: 1.5 Learning Outcome: Explain how economic performance is monitored Skill: Concept 38) What is the economic condition characterized by widespread increased prices without increased purchasing power? A) unemployment B) inflation C) expansion D) deflation E) recession Answer: B Explanation: B) Inflation is the condition that occurs when widespread price increases are present throughout an economic system. This happens when the amount of money in the economy grows faster than the amount of products available to buy. Page Ref: 21-22 Difficulty: Easy Objective: 1.5 Learning Outcome: Explain how economic performance is monitored Skill: Concept 39) What is the condition in an economic system in which the amount of money available and the number of goods and services produced are growing at about the same rate? A) unemployment B) stability C) deflation D) inflation E) oversupply Answer: B Explanation: B) Economic stability happens when the amount of money available in an economic system and the quantity of goods and services produced in it are growing at about the same rate. When this occurs, there is neither inflation nor deflation. Page Ref: 23 Difficulty: Easy Objective: 1.5 Learning Outcome: Explain how economic performance is monitored Skill: Concept 40) What are profits? A) total money taken in by a corporation B) increases in income from year to year C) increases in a corporation's stock price D) revenue generated by goods and services E) difference between revenues and expenses Answer: E Explanation: E) Profits represent the difference between an organization's revenues and its expenses; profits are the goal of most businesses and allow people to open and expand businesses. Page Ref: 4 Difficulty: Moderate Objective: 1.1 Learning Outcome: Explain how economic performance is monitored Skill: Concept 41) A new U.S. company plans to introduce an inexpensive, high-quality line of shoes to the American market. It has found a manufacturer in another country that can produce the shoes at a low enough cost that they will still be cheaper than other brands of the same quality. Which of the following represents a potential ethical implication that the company should consider before beginning production? A) the country's existing labor laws and the factory working conditions B) the average exchange rate of the country's currency over a ten-year period C) the challenges of doing business in a country with a nonconvertible currency D) the energy demands of the manufacturer's facility E) the basic international business strategy it will use Answer: A Explanation: A) When choosing a country to outsource to, a company should not only consider the financial implications of offshoring, but also the ethical implications of conducting business in a country that may have very different laws and legal standards. Page Ref: 6 Difficulty: Moderate AACSB: Reflective thinking skills Objective: 1.1 Learning Outcome: Compare and contrast different economic systems Skill: Application 42) What marks the key difference between economic systems? A) the way they manage the factors of production B) the way they transport goods within their borders C) the way rules and regulations are legislated D) the way workers are relocated to different regions E) the way basic necessities are determined Answer: A Explanation: A) Economic systems manage the factors of production differently; management of these factors often revolves around the degree of government control of them. Page Ref: 7 Difficulty: Moderate Objective: 1.3 Learning Outcome: Compare and contrast different economic systems Skill: Concept 43) What does a planned economy rely on a centralized government to do? A) support free enterprise in every way possible B) allocate all or most factors of production C) encourage citizens to buy shares of stock in small companies D) keep its control activities to a minimum E) direct workers to start their own small businesses Answer: B Explanation: B) A centralized government controls all or most of the factors of production. Page Ref: 10 Difficulty: Moderate AACSB: Dynamics of the global economy Objective: 1.3 Learning Outcome: Compare and contrast different economic systems Skill: Concept 44) Which economic system emphasizes the private ownership of most factors of production? A) socialism B) communism C) capitalism D) Marxism E) federalism Answer: C Explanation: C) Capitalism makes use of the profit incentive for private ownership of the factors of production and encourages entrepreneurship by offering profits as an incentive. Page Ref: 10 Difficulty: Easy AACSB: Dynamics of the global economy Objective: 1.3 Learning Outcome: Compare and contrast different economic systems Skill: Concept 45) A city of 150,000 people has a large commercial sector bringing in businesses from the surrounding region. In 2005, there was 2.75 million square feet of commercial space available. Five years later, after a building boom, there was 5.5 million square feet of commercial space available. Which effect will most likely occur? A) The equilibrium price of commercial real estate will not be effected. B) The increase in the supply of commercial real estate will lead to a corresponding increase in its price. C) The price of commercial real estate will decrease, absent a dramatic increase in demand. D) The increase in the supply of commercial real estate will lead to an increase in resource prices. E) The increase in supply will cause a corresponding increase in demand. Answer: C Explanation: C) Since the supply of commercial real estate has dramatically increased, unless there was a dramatic increase in demand we would expect the price of commercial real estate to decrease. Page Ref: 12 Difficulty: Moderate AACSB: Reflective thinking skills Objective: 1.4 Learning Outcome: Discuss strategies for setting and adjusting prices Skill: Application 46) A new watch manufacturer would like to determine the market price of a new line of watches. How could the manufacturer most effectively determine the market price? A) Place several of the new watches on sale at a watch store and see which sells most quickly. B) Arrange to sell the watches in a number of diverse auctions. C) Set the price of one watch higher than another and see which sells the most quickly. D) Survey potential customers and retailers on what they think the price should be. E) Canvass executives from competing companies about what they think the price should be. Answer: B Explanation: B) Auctions are an effective way to determine market prices because consumers determine the selling price. Page Ref: 16 Difficulty: Moderate AACSB: Reflective thinking skills Objective: 1.4 Learning Outcome: Discuss strategies for setting and adjusting prices Skill: Application 47) A new feature on the the latest Acme smart phone has made it wildly popular. Which action should Acme take to maximize profits? A) Increase the supply of the smart phone. B) Decrease the supply of the smart phone. C) Reduce the price of the smart phone. D) Maintain the current supply until demand falls. E) Create a shortage of the smart phone. Answer: A Explanation: A) Acme will want to have more phones available when demand is high. Page Ref: 12 Difficulty: Moderate AACSB: Reflective thinking skills Objective: 1.4 Learning Outcome: Discuss strategies for setting and adjusting prices Skill: Application 48) Which of the following is NOT a likely reason why a small business in a free enterprise system should seek the ideal combination of price charged and quantity supplied? A) to maintain goodwill among customers B) to maximize profits C) to avoid surpluses and shortages D) to avoid government regulation E) to discourage competition Answer: D Explanation: D) A small business is unlikely to be a monopoly so how it sets its prices will not attract the attention of government regulators. Page Ref: 12 Difficulty: Moderate AACSB: Reflective thinking skills Objective: 1.4 Learning Outcome: Discuss strategies for setting and adjusting prices Skill: Application 49) In a small town of 3,000 people, the number of car washes has grown from two to five in the last two months. Which result will likely occur? A) The increase in supply will have no effect on the equilibrium price. B) The increase in supply will lead to a decrease in demand. C) The increase in supply will lead to a decrease in the equilibrium price. D) The increase in supply will lead to an increase in demand. E) The increase in supply will lead to an increase in equilibrium price. Answer: C Explanation: C) In general, greater availability of goods and services leads to a decrease in cost. Page Ref: 12 Difficulty: Moderate AACSB: Reflective thinking skills Objective: 1.4 Learning Outcome: Discuss strategies for setting and adjusting prices Skill: Application 50) Ernesto's Pizza is about to offer customers a new variety of a personal pizza. Which of the following actions by Ernesto's Pizza would most likely prevent a surplus or a shortage of the personal pizzas in its first week? A) limit the supply of the personal pizza B) lower demand for the personal pizza C) set the price of substitute goods D) estimate the market price of the personal pizza E) lower the price of the pizza throughout the week Answer: D Explanation: D) The market price occurs where there is neither a surplus nor a shortage of supply. Page Ref: 12 Difficulty: Moderate AACSB: Reflective thinking skills Objective: 1.4 Learning Outcome: Describe the major components of effective distribution Skill: Application 51) Which of the following is one of the elements required in private enterprise? A) adequate representation in the government B) freedom from foreign competition C) the opportunity for market leadership D) the right to ownership of property E) numerous regulatory agencies Answer: D Explanation: D) Necessary elements of capitalism include private property rights, freedom of choice, profits, and competition. Page Ref: 14 Difficulty: Moderate AACSB: Analytic skills Objective: 1.4 Learning Outcome: Compare and contrast different economic systems Skill: Application 52) In perfect competition, which of the following conditions must prevail? A) All firms must adhere to cost and pricing standards. B) The number of firms in the industry must be limited. C) A single firm is able to influence the price of its product. D) It is relatively difficult to enter the industry. E) All firms in a given industry must be small. Answer: E Explanation: E) In perfect competition, all firms in a given industry must be small, the number of firms in the industry must be large, and no single firm is powerful enough to influence the price of its product; therefore, prices are determined completely by supply and demand. Page Ref: 14 Difficulty: Moderate Objective: 1.4 Learning Outcome: Compare and contrast different economic systems Skill: Concept 53) How does monopolistic competition differ from perfect competition? A) There are more sellers in a market characterized by monopolistic competition. B) It is easier for sellers to enter a market or industry characterized by monopolistic competition. C) In a perfectly competitive market, products are more dissimilar. D) In a market characterized by monopolistic competition, individual firms have some control over price. E) In a perfectly competitive market, the size of the firms must be large. Answer: D Explanation: D) Individual firms have some control over price in monopolistic competition; in perfect competition price is determined by supply and demand. Page Ref: 14 Difficulty: Moderate AACSB: Analytic skills Objective: 1.4 Learning Outcome: Compare and contrast different economic systems Skill: Concept 54) Which of the following is characterized by having few sellers, similar prices among sellers, and difficult market entry? A) oligopoly B) pure competition C) monopoly D) monopolistic competition E) state socialism Answer: A Explanation: A) An oligopoly has few sellers. Because there is competition between those sellers, they will offer similar prices. Because those sellers will be large and well-established, it will be difficult for new companies to enter the market. Page Ref: 15 Difficulty: Moderate Objective: 1.4 Learning Outcome: Compare and contrast different economic systems Skill: Concept 55) In an oligopoly, when one firm reduces its prices, how do other sellers react? A) There is no reaction from other firms to change their prices. B) Other firms reduce their prices also, usually quite quickly. C) Other firms may reduce their prices, but usually gradually. D) Other firms are not usually aware that price has changed. E) There is a combined response from other firms to stabilize prices. Answer: B Explanation: B) Because there are few firms competing in an oligopoly, competition between firms tends to be close, and firms thus respond quickly to price reductions by one firm. Therefore, other firms will likely quickly reduce their prices as well. For example, when an airline announces new fare discounts, others adopt the same strategy. Page Ref: 15 Difficulty: Moderate AACSB: Analytic skills Objective: 1.4 Learning Outcome: Compare and contrast different economic systems Skill: Application 56) Which of the following is the most likely pricing approach for a monopoly? A) Charge customers whatever you please. B) Give customers a real bargain on price. C) Charge a price that will not cause consumer demand to drop. D) Undercut the prices of the competition. E) Give customers a fair price that cannot be matched by competitors. Answer: C Explanation: C) A monopoly happens when one firm dominates the market. The monopolizing firm does not have to factor competitors' prices when determining its own prices, so it only needs to charge a price that will not be so high that it causes demand to drop. Page Ref: 16 Difficulty: Moderate Objective: 1.4 Learning Outcome: Compare and contrast different economic systems Skill: Concept 57) What term do economists use for the pattern of short-term expansions and contractions in the economy? A) business cycle B) aggregate output C) standard of living D) consumer price index E) purchasing power parity Answer: A Explanation: A) Short-term ups and downs are referred to as the business cycle. Page Ref: 17 Difficulty: Easy Objective: 1.5 Learning Outcome: Compare and contrast different economic systems Skill: Concept 58) Which indicator refers to the total quantity of goods and services produced by an economic system during a given period? A) the business cycle B) aggregate output C) gross domestic product D) the consumer price index E) gross national product Answer: B Explanation: B) Aggregate output is the primary measure of growth in the business cycle. Page Ref: 17 Difficulty: Moderate Objective: 1.5 Learning Outcome: Explain how economic performance is monitored Skill: Concept 59) Azucks is a socially responsible American company engaged in the manufacture of sports shoes. The CEO cautions that there is a disadvantage associated with the company planning increased globalization of production. Which of the following is the most likely disadvantage he is referring to? A) Heavy job losses can ensue in the domestic market. B) Greater disparities in living standards will emerge. C) Trade barriers will be reduced between countries. D) Substantial job losses will occur in developing markets. E) Increased competition will emerge between companies. Answer: A Explanation: A) Developed countries usually have higher labor costs than developing countries so some companies will choose to outsource production to a developing country in order to lower costs. This results in the loss of jobs in the developed countries where the goods were originally produced. Page Ref: 19 Difficulty: Moderate AACSB: Reflective thinking skills Objective: 1.5 Learning Outcome: Compare and contrast different economic systems Skill: Application 60) A financial analyst has noticed a serious rise in inflation. What course of action might the financial analyst advise the government to take in order to reduce inflation? A) The government can lower taxes. B) The government can raise taxes. C) Encourage the government to increase spending. D) Encourage consumers to increase spending. E) Encourage the government to increase spending and lower taxes. Answer: B Explanation: B) Raising taxes encourages consumers to spend less, thereby curbing inflation. Page Ref: 21 Difficulty: Moderate AACSB: Reflective thinking skills Objective: 1.5 Learning Outcome: Compare and contrast different economic systems Skill: Application 61) Which course of action should the Federal Reserve take if it wishes to reduce inflation and curb consumer spending? A) The Federal Reserve should buy securities. B) The Federal Reserve should sell securities. C) The Federal Reserve should decrease the reserve requirement. D) The Federal Reserve should lower the discount rate. E) The Federal Reserve should sell securities and lower the discount rate. Answer: B Explanation: B) Selling securities reduces the money supply, thus curbing consumer spending and bringing prices down. Page Ref: 23 Difficulty: Moderate AACSB: Reflective thinking skills Objective: 1.5 Learning Outcome: Compare and contrast different economic systems Skill: Application 62) The Fed has opted to buy securities and lower the reserve requirement. What will most likely happen as a result of this course of action? A) The money supply will increase thereby stimulating the economy. B) The money supply will increase thereby reducing inflation. C) The money supply will increase thereby causing disinflation. D) The money supply will decrease thereby reducing the possibility of a recession. E) The money supply will decrease thereby reducing deflation. Answer: A Explanation: A) Buying securities and lowering the reserve requirement will increase the money supply, which in turn stimulates the economy. Page Ref: 23 Difficulty: Moderate AACSB: Reflective thinking skills Objective: 1.5 Learning Outcome: Compare and contrast different economic systems Skill: Application 63) If the Fed is concerned with a potential recession, what course of action should it take? A) Lower the reserve requirement and sell securities. B) Lower the discount rate and increase the reserve requirement. C) Sell securities and lower the discount rate. D) Buy securities and lower the discount rate. E) Buy securities and increase the reserve requirement. Answer: D Explanation: D) Buying securities and lowering the discount rate increases the money supply, which encourages spending. Page Ref: 23 Difficulty: Moderate AACSB: Reflective thinking skills Objective: 1.5 Learning Outcome: Compare and contrast different economic systems Skill: Application 64) The global business environment encompasses many international forces that influence the outcomes of companies doing business in foreign countries. Which of the following scenarios would best count as evidence of globalization? A) Diamonds mined in Country X are sold on the markets in Country Y. The proceeds are then used to fund a coup against the government in Country X. B) A natural disaster disables factories in Country X that produce computer chips for Company Y. Company Y reduces the number of employee shifts and raises the price of its products. C) Farmers in Country X have lost most of their wheat crops due to a drought. The market price for wheat triples and the government pays the farmers for a percentage of the lost wheat. D) Country X has almost mined all of its available natural energy sources. The government has been investing in alternative energy research for years and now an efficient synthetic energy source has been developed. E) Country X has raised the tax on oil products. As a result more people are taking public transportation and reducing unnecessary car travel. Answer: B Explanation: B) Globalization is the movement toward a more interconnected and interdependent world market. One consequence of this is that countries and companies become more reliant on each other and events in one part of the world can directly affect conditions in another part of the world. Page Ref: 6 Difficulty: Difficult AACSB: Reflective thinking skills Objective: 1.1 Learning Outcome: Compare and contrast different economic systems Skill: Application 65) The CEOs of Company A and B are investigating expansion into new markets, which will provide greater trade volume. Which scenario describes an economic environment that is most suitable for an importing-exporting opportunity? A) Country X invests primarily in its textile industry while Country Y focuses on producing high-yield crops. B) Company A is willing to provide parts used in the production of equipment by Company B in exchange for discounts on manufacturing equipment. C) The southern region of Country X over-produces a wide range of produce, while few crops can be efficiently grown in the northern region. D) Company A and Company B are exploring the possibility of a joint venture to provide goods to Country X. E) Country X is looking to invest in foreign markets and Country Y currently has a large trade deficit. Answer: A Explanation: A) The economic environment refers to relevant conditions, such as import and export opportunities, that exist in the economic system in which a company operates. In the above case, importing and exporting both refer to the movement of goods between different countries. Countries X and Y focus on a particular industry, which implies that these are areas in which they have an import-export opportunity. Page Ref: 6 Difficulty: Difficult AACSB: Reflective thinking skills Objective: 1.1 Learning Outcome: Compare and contrast different economic systems Skill: Application 66) Robert has established a start-up business manufacturing a range of outdoor wear for the leisure market. Which possible scenario would best show that Robert is positioning his business for the global business market? A) Robert launched a Web site for his business and hired two new employees to handle national and international shipping. B) Robert launched a Web site and filmed a commercial for television and recorded a radio version for local stations. C) Robert created a page for his business on a networking site and offered store coupons to his first 100 friends. D) Robert had his Web site translated into Korean in order to better serve the local Korean community. E) Robert took a full page ad out in the state's official tourism brochure. Answer: A Explanation: A) By creating a Web site, which can be viewed worldwide, and expanding his business to handle national and international shipping, Robert is actively engaging in market globalization. Page Ref: 6 Difficulty: Difficult AACSB: Reflective thinking skills Objective: 1.1 Learning Outcome: Compare and contrast different economic systems Skill: Application 67) What is a chief goal of an economic system? A) economic stability B) income equality C) money supply growth D) national debt reduction E) product innovation Answer: A Explanation: A) With stability, the amount of money available in an economic system and the quantity of goods and services produced in it are growing at about the same rate. Page Ref: 21 Difficulty: Moderate Objective: 1.5 Learning Outcome: Compare and contrast different economic systems Skill: Concept 68) Which of the following best describes economic stability? A) the measure of economic growth that compares how much a system produces with the resources needed to produce it B) a condition in an economic system in which the amount of money available and the number of goods and services produced are growing at about the same rate C) the economic condition in which a country's exports exceed its imports D) a relative equality between what the government owes its creditors and the revenue it gets from imports E) the economic value of all the products that a country exports minus the economic value of all the products it imports Answer: B Explanation: B) Economic stability is evident when the amount of money available and the number of goods and services produced are growing at about the same rate; inflation and unemployment, in particular, threaten economic stability. Page Ref: 21 Difficulty: Moderate AACSB: Analytic skills Objective: 1.5 Learning Outcome: Compare and contrast different economic systems Skill: Concept 69) Which of the following statements about inflation is true? A) The consumer price index is a good way of measuring inflation. B) Productivity decreases the purchasing power of consumer dollars. C) Inflation increases the purchasing power of consumer dollars. D) Localized price increases can cause inflation across the economic system. E) The amount of money tends to be distributed evenly during inflationary periods. Answer: A Explanation: A) The consumer price index is a good measurement of inflation; inflation occurs with widespread price increases. Page Ref: 22 Difficulty: Moderate Objective: 1.5 Learning Outcome: Explain how economic performance is monitored Skill: Concept 70) How would supply affect the aggregate output of an economy? A) Supply would decrease aggregate output. B) Increases in both would cause inflation. C) Aggregate output would grow with supply. D) Supply would have no direct effect on aggregate output. E) Aggregate output would increase the money supply. Answer: C Explanation: C) Supply is the amount of goods available, and aggregate output is the production of goods, so increased supply would increase aggregate output. Page Ref: 17 Difficulty: Moderate Objective: 1.4 Learning Outcome: Explain how economic performance is monitored Skill: Synthesis 71) Which of the following best describes what the consumer price index is used to indicate? A) the average wage rate for a given region B) the economic performance of key consumer product companies C) the changes in the cost of consumer products over time D) the percentage of consumers in the total population looking for work E) the number of price increases within a given period of time Answer: C Explanation: C) The consumer price index is a measure of the prices of typical products purchased by consumers living in urban areas. It is expressed as a percentage of prices as compared to a base period. Page Ref: 22 Difficulty: Moderate Objective: 1.5 Learning Outcome: Explain how economic performance is monitored Skill: Concept 72) Which of the following is a likely cause of increased prices for products, decreased purchasing power, and decreased profit margins? A) low unemployment B) stagnant wages C) declining living standards D) cyclical inflation E) limited credit Answer: A Explanation: A) Potential results of low unemployment include increased prices, decreased purchasing power, and decreased profits. Page Ref: 22 Difficulty: Difficult Objective: 1.5 Learning Outcome: Explain how economic performance is monitored Skill: Synthesis 73) What type of policy is made up of fiscal and monetary policy? A) trade B) employment C) stabilization D) tax E) foreign Answer: C Explanation: C) A stabilization policy is a government goal to smooth out fluctuations in output and unemployment and to even out prices. Page Ref: 23 Difficulty: Moderate Objective: 1.5 Learning Outcome: Compare and contrast different economic systems Skill: Concept 74) In the city of Westminster, Christmas trees are being sold for $25. At this price, buyers are able to satisfy their demand, though some trees remain unsold. What is the term used to describe this market condition? A) surplus B) stoppage C) shortage D) demand deficit E) equilibrium price Answer: A Explanation: A) A surplus is defined as a situation in which quantity supplied exceeds quantity demanded. Page Ref: 13 Difficulty: Moderate Objective: 1.4 Learning Outcome: Explain how economic performance is monitored Skill: Application 75) In the United States, milk is produced on thousands of farms, with no producer dominating the market. Producers sell milk at the going price and are unable to influence this price. Millions of gallons of milk are purchased every day. What type of competition is the market for milk? A) a monopoly B) an oligopoly C) perfect competition D) monopolistic competition E) a cooperative Answer: C Explanation: C) In perfect competition, there are many sellers who are not individually powerful enough to influence price. Page Ref: 14 Difficulty: Difficult Objective: 1.4 Learning Outcome: Compare and contrast different economic systems Skill: Application 76) Which of the following best gives the meaning of the term gross domestic product? A) total quantity of goods and services produced by an economic system B) total quantity of goods that a country's citizens can purchase with the currency used in their economic system C) conditions of the economic system in which an organization operates D) total value of all goods and services produced within a given period by a national economy through domestic factors of production E) indicators of available goods and services produced within a specific domestic market Answer: D Explanation: D) Gross domestic product (GDP) is the total value of all goods and services produced within a given period by a national economy through domestic factors of production; this figure does not include goods and services produced by domestic firms overseas. Page Ref: 17 Difficulty: Difficult Objective: 1.5 Learning Outcome: Explain how economic performance is monitored Skill: Concept 77) Which of the following best gives the meaning of the term business cycle? A) the conditions of the economic system in which an organization operates B) the total quantity and quality of goods and services that a country's citizens can purchase with the currency used in their economic system C) the measure of economic growth that compares how much a system produces with the resources needed to produce it D) the number of businesses started during a given time period E) the pattern of short-term ups and downs in an economy Answer: E Explanation: E) A business cycle is represented in the short-term ups and downs in the economy, as measured by prosperity, recession, and so on. Page Ref: 17 Difficulty: Moderate AACSB: Reflective thinking skills Objective: 1.5 Learning Outcome: Explain how economic performance is monitored Skill: Application 78) Which of the following best gives the meaning of the term gross national product? A) total quantity of goods and services produced by an economic system B) total quantity of goods and services produced within a given period by a national economy, not including foreign production C) total value of all goods and services produced within a given period by a national economy through domestic factors of production D) total value of all goods and services produced within a given period regardless of where the factors of production are located E) total value of all goods and services that are exported Answer: D Explanation: D) Gross national product includes even the value of goods and services produced by domestic companies abroad. Page Ref: 18 Difficulty: Moderate Objective: 1.5 Learning Outcome: Explain how economic performance is monitored Skill: Concept 79) Which of the following best describes nominal GDP? A) total quantity of goods and services produced by an economic system B) total value of all goods and services produced within a given period by a national economy through domestic factors of production measured in current dollars or with all components valued at current prices C) total value of all goods and services produced within a given period by a national economy regardless of where the factors of production are located D) total value of all goods and services produced within a given period by a national economy through domestic factors of production adjusted to account for changes in currency values and price changes E) total quantity of goods and services that are imported within a given period of time Answer: B Explanation: B) Nominal GDP is not adjusted to make up for inflation and/or changes in currency values. Page Ref: 18 Difficulty: Moderate Objective: 1.5 Learning Outcome: Explain how economic performance is monitored Skill: Concept 80) Which of the following would be included in the gross domestic product of Brazil? A) the profits earned by a U.S.-owned automobile plant in Brazil B) the profits earned by a Brazilian coffee company operating in the United States C) the profits earned by a Brazilian coffee company operating in Peru D) the profits earned by U.S. suppliers to Brazil operating in the United States E) the profits earned by Peruvian companies operating outside Brazil Answer: A Explanation: A) Gross domestic product refers to the value of goods and services produced in a given period domestically. Page Ref: 18 Difficulty: Difficult Objective: 1.5 Learning Outcome: Explain how economic performance is monitored Skill: Application 81) Honata Automobiles is a South Korean-owned company. Honata has factories in Indonesia and Brazil. Which of the following is/are affected by the production of Honata automobiles? A) Indonesia, Brazil, and South Korea's GDP B) Indonesia and Brazil's GDP C) South Korea's GDP D) Indonesia and Brazil's GNP E) Indonesia, Brazil, and South Korea's GNP Answer: B Explanation: B) GDP measures goods and services produced within a country regardless of where the companies are based. Page Ref: 18 Difficulty: Difficult AACSB: Reflective thinking skills Objective: 1.5 Learning Outcome: Explain how economic performance is monitored Skill: Application 82) Which of the following best describes purchasing power parity? A) total quantity of goods and services produced by an economic system B) total quantity of goods and services that can be purchased with one paycheck C) total quantity of goods and services that could have been purchased if one's pay rose in the same proportion as inflation D) the principle that exchange rates are set so that prices of similar products in different countries are about the same E) the principle that a market economy determines supply and demand for consumer products Answer: D Explanation: D) Purchasing power parity gives us a good idea of what people can actually buy with the financial resources allocated to them by their respective economic systems. Page Ref: 19 Difficulty: Difficult Objective: 1.5 Learning Outcome: Explain how economic performance is monitored Skill: Concept 83) Which is the best explanation for why standard of living only increases through productivity? A) The more goods that are available, the lower prices will be. B) More goods are available without having to come by additional resources. C) Productivity increases the money supply in an economy. D) Purchasing power increases when there is more consumer choice. E) Prices are determined by the standard of living. Answer: B Explanation: B) Productivity is the amount of goods that can be made using a given amount of resources. If it takes more resources to produce more goods, prices will not decrease as long as demand keeps pace. But if more goods can be made using the same resources, prices will go down and purchasing power will increase. Page Ref: 19 Difficulty: Difficult AACSB: Reflective thinking skills Objective: 1.5 Learning Outcome: Explain how economic performance is monitored Skill: Synthesis 84) Why would a government most likely be concerned about its country carrying a trade deficit? A) A trade deficit means that the country's productivity is low. B) A trade deficit means local companies do not have enough competition. C) A trade deficit ties up money that could be used for economic growth. D) A trade deficit decreases demand for goods overall. E) A trade deficit means that consumers do not have enough purchasing power. Answer: C Explanation: C) A trade deficit implies a carrying of debt; when there is a trade deficit imports are not paid for in full. Page Ref: 20-21 Difficulty: Moderate AACSB: Reflective thinking skills Objective: 1.5 Learning Outcome: Explain how economic performance is monitored Skill: Application 85) If, in the long run, international trade improves the standards of living in participating countries, why might some countries want to place trade barriers such as extra taxes on imported products? A) Imported products compete with domestic products and thus put pressure on local business. B) Imported products are usually not as good quality as domestically made products. C) Generating government revenue is often more economically valuable than free trade. D) This is necessary to keep a country's currency strong. E) International trade tends to lead to political tension. Answer: A Explanation: A) In the course of international trade, jobs are often initially lost in those industries in which a country does not have a comparative advantage. Trade barriers are used as a way to protect these industries from strenuous competition. Page Ref: 20 Difficulty: Difficult AACSB: Reflective thinking skills Objective: 1.5 Learning Outcome: Explain the benefits and challenges of engaging in international business Skill: Synthesis 86) Through various stimulus programs, the U.S. government injected money into the economy during the 2008-2009 recession. What was the concern that this type of action would lead to? A) increased labor costs B) increased spending C) increased unemployment D) increased inflation E) increased trade imbalance Answer: D Explanation: D) Some experts feared that increasing the money supply might also lead to inflation. Page Ref: 23 Difficulty: Moderate Objective: 1.5 Learning Outcome: Explain how economic performance is monitored Skill: Application 87) The government is concerned that the economy is being stifled by low consumer spending. Which course of action should the Federal Reserve take if it wishes to improve the economy? A) The Federal Reserve should buy securities. B) The Federal Reserve should sell securities. C) The Federal Reserve should increase the reserve requirement. D) The Federal Reserve should increase the discount rate. E) The Federal Reserve should sell securities and lower the discount rate. Answer: A Explanation: A) Buying securities will increase the money supply, which will increase consumer spending, thereby improving the economy. Page Ref: 23 Difficulty: Difficult AACSB: Reflective thinking skills Objective: 1.5 Learning Outcome: Explain how economic performance is monitored Skill: Application Chapter 2 Business Ethics and Social Responsibility 27) What are beliefs about what is right and wrong or good and bad called? A) motivators B) rules C) cultures D) ethics E) laws Answer: D Explanation: D) Ethics are beliefs about what is right and wrong or good and bad in actions that affect others. Page Ref: 32 Difficulty: Easy AACSB: Ethical understanding and reasoning abilities Objective: 2.1 Learning Outcome: Discuss the roles of ethics and corporate responsibility in business Skill: Concept 28) Which of the following should be the first step in assessing ethical behavior in a certain situation? A) Seek the advice of managers. B) Make a judgment based on the outcome of the activity. C) Consider appropriate moral values. D) Collect facts related to the situation. E) Gather a range of opinions on the effectiveness of the policy. Answer: D Explanation: D) The first step in assessing ethical behavior is to gather the correct factual information. Page Ref: 35 Difficulty: Easy AACSB: Ethical understanding and reasoning abilities Objective: 2.1 Learning Outcome: Discuss the roles of ethics and corporate responsibility in business Skill: Concept 29) What is likely the single most effective step that a company can take to set ethical standards? A) Offer large cash awards for whistle-blowing. B) Suggest that employees take ethics training. C) Post ethical rules on bulletin boards. D) State that the workplace values diversity. E) Demonstrate support from top management. Answer: E Explanation: E) Perhaps the single most effective step a company can take in setting ethical standards is to demonstrate top management support of ethical standards. Page Ref: 36 Difficulty: Easy AACSB: Ethical understanding and reasoning abilities Objective: 2.1 Learning Outcome: Discuss the roles of ethics and corporate responsibility in business Skill: Concept 30) How do top managers best demonstrate a commitment to ethical business practices? A) by adopting written codes of ethics B) by reading employees' e-mails C) by decentralized decision-making practices D) by cooperating with other companies E) by monitoring employees' Web searches Answer: A Explanation: A) A common approach to formalizing top management commitment to ethical business practices is the adoption of a written code of ethics. Page Ref: 36 Difficulty: Easy AACSB: Ethical understanding and reasoning abilities Objective: 2.1 Learning Outcome: Discuss the roles of ethics and corporate responsibility in business Skill: Concept 31) Which statement about codes of ethics is FALSE? A) Fewer and fewer companies are adopting codes of ethics. B) Codes of ethics are determined by company managers. C) Some ethical responses can be learned through experience. D) Social responsibility involves providing quality products and pricing products fairly. E) Core principles and values should remain a constant characteristic of a company. Answer: A Explanation: A) The number of companies adopting an ethical code has risen sharply in the last thirty years. Page Ref: 37 Difficulty: Easy AACSB: Ethical understanding and reasoning abilities Objective: 2.1 Learning Outcome: Discuss the roles of ethics and corporate responsibility in business Skill: Application 32) Which of the following statements is most likely to be included in a company's code of ethics? A) We reward high achievers. B) We strive for domination in the market. C) We achieve our objective by raising capital. D) We believe in our company brand. E) We reward all valuable contributions. Answer: E Explanation: E) This statement reflects a strong corporate value and a high ethical standard. Page Ref: 37-38 Difficulty: Easy AACSB: Ethical understanding and reasoning abilities Objective: 2.1 Learning Outcome: Discuss the roles of ethics and corporate responsibility in business Skill: Application 33) What term refers to the way in which a business tries to balance its commitments to groups and individuals in its social environment? A) business ethics B) socialization C) stakeholder consciousness D) social responsibility E) corporatization Answer: D Explanation: D) Social responsibility refers to the way in which a business tries to work responsibly within its social environment by benefiting all stakeholders. Page Ref: 38 Difficulty: Easy AACSB: Ethical understanding and reasoning abilities Objective: 2.2 Learning Outcome: Discuss the roles of ethics and corporate responsibility in business Skill: Concept 34) How often should an organization change its objectives? A) constantly B) infrequently C) every year D) every decade E) never Answer: B Explanation: B) The objectives of a business should change only occasionally. Page Ref: 38 Difficulty: Easy AACSB: Ethical understanding and reasoning abilities Objective: 2.2 Learning Outcome: Discuss the factors that influence decisions about organizational structure Skill: Concept 35) What is the term given to the groups, individuals, and organizations that are directly affected by the practices of an organization? A) primary agents of interest B) social auditors C) senior managers D) organizational stakeholders E) local communities Answer: D Explanation: D) Organizational stakeholders are the groups, individuals, and organizations that are directly affected by the practices of an organization. Page Ref: 39 Difficulty: Easy AACSB: Ethical understanding and reasoning abilities Objective: 2.2 Learning Outcome: Discuss the factors that influence decisions about organizational structure Skill: Concept 36) Advertising for which of the following products is most likely to have ethical ramifications? A) swimwear B) sporting equipment C) decaffeinated beverages D) books E) alcohol Answer: E Explanation: E) Laws regulate the advertisement of such items as underwear, condoms, alcohol, and firearms. Page Ref: 45 Difficulty: Easy AACSB: Ethical understanding and reasoning abilities Objective: 2.3 Learning Outcome: Discuss the roles of ethics and corporate responsibility in business Skill: Application 37) What must every corporation do in maintaining and reporting its financial status? A) purchase stocks on margin B) use creative accounting practices C) support increased stock prices D) aim to maximize predicted profit margins E) conform to generally accepted accounting principles Answer: E Explanation: E) Every corporation must conform to generally accepted accounting practices. This ensures that all corporations are held to the same standards for assessing their financial information. Page Ref: 47 Difficulty: Easy AACSB: Ethical understanding and reasoning abilities Objective: 2.3 Learning Outcome: Identify the basic concepts of and tools used for business accounting Skill: Concept 38) Which of the following occurs when someone uses confidential information to gain from the purchase or sale of stocks? A) identity theft B) a margin purchase C) insider trading D) collusion E) a guarantee Answer: C Explanation: C) Insider trading is the illegal practice of using special knowledge about a firm for profit or gain. Page Ref: 47 Difficulty: Easy AACSB: Ethical understanding and reasoning abilities Objective: 2.3 Learning Outcome: Identify the basic concepts of and tools used for business accounting Skill: Concept 39) What does the Sarbanes-Oxley Act of 2002 require a chief financial officer to do? A) prevent insider trading B) engage in strategic management C) guarantee the accuracy of financial reporting D) hire a forensic accountant E) oversee the implementation of ethical standards Answer: C Explanation: C) The Sarbanes-Oxley Act of 2002 requires that the chief financial officer personally guarantee the accuracy of financial reporting. Page Ref: 47 Difficulty: Easy AACSB: Ethical understanding and reasoning abilities Objective: 2.4 Learning Outcome: Identify the basic concepts of and tools used for business accounting Skill: Concept 40) If a firm hides responsibility for wrongdoing, what type of stance are they taking toward social responsibility? A) responsible B) obstructionist C) accommodative D) proactive E) moderate Answer: B Explanation: B) An obstructionist stance toward social responsibility means a company will do as little as possible to solve social or environmental problems. Page Ref: 48 Difficulty: Easy AACSB: Ethical understanding and reasoning abilities Objective: 2.4 Learning Outcome: Discuss the roles of ethics and corporate responsibility in business Skill: Concept 41) Which of the following is a way to tell how an organization is doing in meeting its social responsibility goals? A) governmental mandate B) executive summary C) social audit D) government initiative E) financial report Answer: C Explanation: C) A social audit is a systematic analysis of an organization's success in using funds earmarked for its social responsibility goals. Page Ref: 50 Difficulty: Moderate AACSB: Ethical understanding and reasoning abilities Objective: 2.4 Learning Outcome: Discuss the roles of ethics and corporate responsibility in business Skill: Concept 42) What is the term for an activity that may benefit the individual to the detriment of his or her employer? A) agency dispute B) leadership challenge C) conflict of interest D) proactive stance E) ethical standard Answer: C Explanation: C) For example, many companies have policies that forbid buyers from accepting gifts from suppliers since such gifts might be construed as a bribe or an attempt to induce favoritism. Page Ref: 34 Difficulty: Moderate AACSB: Ethical understanding and reasoning abilities Objective: 2.1 Learning Outcome: Discuss the roles of ethics and corporate responsibility in business Skill: Concept 43) Which of the following would NOT be included among the primary agents of interest with regard to a company's ethical behavior? A) stockholders B) unions C) bloggers D) customers E) competitors Answer: C Explanation: C) Primary agents of interest include customers, competitors, stockholders, suppliers, unions, and dealers. A business and its employees must consider ethical behaviors in its relationship with these groups. Page Ref: 34 Difficulty: Moderate AACSB: Ethical understanding and reasoning abilities Objective: 2.1 Learning Outcome: Discuss the factors that influence decisions about organizational structure Skill: Concept 44) Which ethical norm considers whether a particular act optimizes the benefits to those who are affected by it? A) utility B) rights C) justice D) caring E) respect Answer: A Explanation: A) The ethical norm of utility considers whether a particular act optimizes the benefits to those who are affected. Page Ref: 36 Difficulty: Moderate AACSB: Ethical understanding and reasoning abilities Objective: 2.1 Learning Outcome: Discuss the roles of ethics and corporate responsibility in business Skill: Concept 45) Which of the following is the best description of organizational stakeholders? A) competitors B) government regulators C) the officers and key employees of an organization D) individuals and businesses that own stock in a company E) individuals and groups that are directly affected by the practices of a company Answer: E Explanation: E) Individuals and groups that are directly affected by the practices of an organization are considered to have a "stake" in the company's performance. Page Ref: 39 Difficulty: Moderate AACSB: Ethical understanding and reasoning abilities Objective: 2.2 Learning Outcome: Discuss the factors that influence decisions about organizational structure Skill: Application 46) The CEO of Perry's Pizzeria, a rapidly growing restaurant chain, is committed to a high level of corporate social responsibility along with continued business success. Which action by the CEO would most enhance the company's reputation for social responsibility? A) using more nutritious, but more expensive, ingredients B) cutting employees' wages and benefits to boost the company's stock price C) using marketing or advertising methods that would allow the company to discreetly reduce pizza size without also reducing the price D) packaging products in material that costs less, even if customers object to the change E) taking steps that would cause competitors to close their doors Answer: A Explanation: A) Although Perry's Pizzeria has a responsibility to make the best business decisions for its stakeholders (i.e., bring in a high profit for investors), being socially responsible requires making the best decision for the community as well. Purchasing nutritious ingredients that are expensive would benefit the company's reputation for social responsibility. Its effect on investor profits would also be an issue of contention for the company's CEO. Page Ref: 40 Difficulty: Moderate AACSB: Reflective thinking skills Objective: 2.2 Learning Outcome: Discuss the roles of ethics and corporate responsibility in business Skill: Application 47) Dara's Dress Shop was honored last year for its strong commitment to corporate social responsibility. Which of the following would MOST likely decrease the shop's chances of being recognized again this year? A) The shop has laid off two employees during the past six months. B) Fewer clearance sales have been offered to the public this year. C) An experienced ethics professional has been added to the Board of Directors. D) Many of the dresses are made from fabrics spun by unprotected child laborers. E) The shop has reduced its line of designer merchandise. Answer: D Explanation: D) A socially responsible company makes decisions in the area of human rights and employment standards in the workplace. Using unprotected child laborers for production goes against this tenet. Page Ref: 41 Difficulty: Moderate AACSB: Reflective thinking skills Objective: 2.2 Learning Outcome: Discuss the roles of ethics and corporate responsibility in business Skill: Application 48) What role does accountability play in business life? A) It encourages businesses to strive for increased revenue. B) It fosters laissez-faire attitudes to economic growth. C) It protects and enhances the general welfare of society. D) It restricts health warnings on harmful products. E) It helps to maximize profits for shareholders. Answer: C Explanation: C) New laws that dictate an expanded role for business in protecting and enhancing the general welfare of society have improved business accountability. Page Ref: 41 Difficulty: Moderate AACSB: Ethical understanding and reasoning abilities Objective: 2.1 Learning Outcome: Discuss the roles of ethics and corporate responsibility in business Skill: Concept 49) Jana and Kate are the owners of an upscale jewelry boutique that offers unique and much desired accessories in an affluent neighborhood. Jana and Kate are concerned, however, that the company's commitment to social responsibility has begun to diminish in various respects. How would this MOST likely be evident? A) Other local businesses may be forced to close their doors. B) The health of employees and their children is starting to suffer. C) Local investors and stakeholders seek to invest their money elsewhere. D) The boutique's customer base may decrease sharply due to moderate increases in price. E) Employees may begin making fewer charitable personal donations. Answer: B Explanation: B) If this trend is seen across the board for the company's employees, it likely indicates that working conditions for them are not good, or that they have insufficient health benefits with their job. Page Ref: 41 Difficulty: Moderate AACSB: Reflective thinking skills Objective: 2.2 Learning Outcome: Discuss the roles of ethics and corporate responsibility in business Skill: Synthesis 50) Jordan is a recent college graduate with a degree in computer programming. He has just begun his first full-time job and is excited to apply the skills that he worked so hard to acquire. Though he excelled as a student, Jordan is anxious about how successful he will be as an employee. When an opportunity unexpectedly presents itself to take credit for a brilliant software program that someone else wrote, Jordan does not resist and claims the program as his own. Which of the following statements must be true about Jordan's decision? A) It was made with careful thought. B) It does not reveal anything about his beliefs or character. C) It raises questions about his personal ethics. D) It will benefit him professionally. E) It will not benefit him professionally. Answer: C Explanation: C) Personal ethics are the guiding principles that help you make ethical decisions in your life. Jordan's decision to take credit for, and essentially steal, someone else's invention reflects questionable personal ethics. Page Ref: 41 Difficulty: Moderate Objective: 2.2 Learning Outcome: Discuss the roles of ethics and corporate responsibility in business Skill: Application 51) Jordan is a recent college graduate with a degree in computer programming. He has just begun his first full-time job and is excited to apply the skills that he worked so hard to acquire. Though he excelled as a student, Jordan is anxious about how successful he will be as an employee. When an opportunity unexpectedly presents itself to take credit for a brilliant software program that someone else wrote, Jordan does not resist and claims the program as his own. Which of the following would LEAST likely also happen along with Jordan's decision? A) Other recently hired employees who learn of Jordan's dishonesty are hesitant to report him because they think he is the boss's favorite new employee. B) After thinking deeply about his behavior, Jordan becomes better equipped to act ethically in the future. C) Without recognizing his behavior as unethical, Jordan steals office supplies and reports more hours on his time sheet than he has actually worked. D) The company's ethics committee agrees that employees should not be expected to tell the truth if honesty is not part of their personal ethics. E) An employee who reports Jordan's dishonesty consequently suffers from psychological and emotional stress. Answer: D Explanation: D) Personal ethics are the guiding principles that help you make ethical decisions in your life. A company with an ethics committee holds ethical behavior and good character as a priority, not something to be overlooked. Page Ref: 41 Difficulty: Moderate AACSB: Ethical understanding and reasoning abilities Objective: 2.2 Learning Outcome: Discuss the roles of ethics and corporate responsibility in business Skill: Application 52) Why has climate change become a major issue for both business and government? A) The body of scientific evidence is inconclusive. B) The solutions to the problems are unanimous. C) The weather patterns are becoming increasingly uniform. D) The change in climate patterns affects the entire planet. E) The effects are confined to national boundaries. Answer: D Explanation: D) Although climate change is occurring at a relatively mild pace at the moment, one possible outcome in the future of increased environmental damage is more rapid climate change. Page Ref: 42 Difficulty: Moderate AACSB: Reflective thinking skills Objective: 2.3 Learning Outcome: Discuss the roles of ethics and corporate responsibility in business Skill: Concept 53) SeaBreeze Sailing Company manufactures sailboats for personal pleasure use. Which of the following would suggest that SeaBreeze Sailing Company is NOT committed to corporate social responsibility? A) Sailboats are available for customer rental only when weather conditions are safe. B) The boats are constructed from scarce or nonrenewable natural resources. C) Employees are provided with excellent vacation packages. D) The company engages in practices that benefit local investors. E) Advertisements for sailboat rentals contain correct pricing information. Answer: B Explanation: B) A company that is socially responsible makes decisions based on environmental concerns and would not use scarce or nonrenewable natural resources for product construction. Page Ref: 43 Difficulty: Moderate AACSB: Reflective thinking skills Objective: 2.3 Learning Outcome: Discuss the roles of ethics and corporate responsibility in business Skill: Application 54) Which one of the following organizations has as its primary function the regulation of advertising and pricing practices? A) Food and Drug Administration (FDA) B) Federal Trade Commission (FTC) C) Federal Communication Commission (FCC) D) Securities and Exchange Commission (SEC) E) Small Business Administration (SBA) Answer: B Explanation: B) The government controls and regulates many business activities; irresponsible practices toward customers can result in government-imposed penalties and expensive civil litigation. Page Ref: 43 Difficulty: Moderate AACSB: Ethical understanding and reasoning abilities Objective: 2.3 Learning Outcome: Discuss the roles of ethics and corporate responsibility in business Skill: Concept 55) What is it called when two or more firms agree to collaborate on wrongful acts? A) monopolizing B) consumerism C) collusion D) brokering E) business as usual Answer: C Explanation: C) A recent example of collusion occurred when the U.S. Justice Department charged three pharmaceutical firms with illegally controlling worldwide supplies and prices of vitamins. Page Ref: 44 Difficulty: Moderate AACSB: Ethical understanding and reasoning abilities Objective: 2.3 Learning Outcome: Discuss the roles of ethics and corporate responsibility in business Skill: Concept 56) What is the term used to describe a situation when increased demand leads to overly steep and often unwarranted price increases? A) price fixing B) price matching C) price stabilization D) price gouging E) price comparison Answer: D Explanation: D) For example, when residents of a coastal area are warned about a possible hurricane, they often flock to retailers to stock up on bottled water and batteries; unfortunately, some retailers take advantage of this pattern by vastly marking up their prices. Page Ref: 44 Difficulty: Moderate AACSB: Ethical understanding and reasoning abilities Objective: 2.3 Learning Outcome: Discuss strategies for setting and adjusting prices Skill: Concept 57) Which of the following is NOT an element of green marketing? A) profitability B) sustainability C) production processes D) product modification E) packaging reduction Answer: A Explanation: A) The elements of green marketing include production processes, product modification, carbon offsets, packaging reduction, and sustainability. Page Ref: 44 Difficulty: Moderate AACSB: Ethical understanding and reasoning abilities Objective: 2.3 Learning Outcome: Discuss the roles of ethics and corporate responsibility in business Skill: Concept 58) Which of the following has NOT been identified as a basic consumer right? A) a right to safe products B) a right to low prices C) a right to be heard D) a right to choose what they buy E) a right to be informed about a product Answer: B Explanation: B) Although low prices are a boon to the consumer and many companies are motivated to offer their products at low prices, low prices are not considered a basic consumer right. Page Ref: 45 Difficulty: Moderate AACSB: Ethical understanding and reasoning abilities Objective: 2.3 Learning Outcome: Discuss the roles of ethics and corporate responsibility in business Skill: Concept 59) What is the term for an employee who detects and tries to put an end to a company's unethical, illegal, and/or socially irresponsible actions by publicizing them? A) fast-track manager B) whistle-blower C) corporate lookout D) government inspector E) blue sky thinker Answer: B Explanation: B) As whistle-blowers, employees generally contact the media or a particular regulatory agency. Page Ref: 46 Difficulty: Moderate AACSB: Ethical understanding and reasoning abilities Objective: 2.3 Learning Outcome: Discuss the roles of ethics and corporate responsibility in business Skill: Concept 60) Chang is the CEO of a local furniture business. He is very committed to treating his customers with honesty and fairness. At the end of a long day, Chang knowingly accepts a $20 bill that a customer accidentally hands him instead of a $1 bill. Which of the following is most likely to be true of Chang? A) He has a highly developed set of personal ethics. B) His entire business is based on unethical principles. C) He has acted in a manner that runs counter to his business ethics. D) His personal ethics and business ethics are consistent. E) He learned his business ethics from his employees. Answer: C Explanation: C) It is dishonest and unethical to take more money than what is owed, which is essentially stealing. If Chang is normally committed to treating customers with honesty and fairness, this action runs counter to his ethics. Page Ref: 47 Difficulty: Moderate AACSB: Reflective thinking skills Objective: 2.4 Learning Outcome: Discuss the roles of ethics and corporate responsibility in business Skill: Application 61) What act was passed in 2002 as a response to a number of highly publicized accounting scandals? A) Sarbanes-Oxley Act B) McCain-Feingold Act C) Securities and Exchange Act D) Accounting Standards Act E) Dodd-Frank Act Answer: A Explanation: A) The Sarbanes-Oxley Act requires an organization's chief financial officer to personally guarantee the accuracy of all financial reporting. Page Ref: 47 Difficulty: Easy Objective: 2.3 Learning Outcome: Identify the basic concepts of and tools used for business accounting Skill: Concept 62) What is the full meaning of the acronym GAAP? A) general action accounting points B) gradually accepted activity positions C) generally accepted accounting principles D) greater antagonistic action plans E) general action audit principles Answer: C Explanation: C) In maintaining and reporting its financial status, every corporation must conform to generally accepted accounting principles (GAAP). Page Ref: 47 Difficulty: Easy AACSB: Ethical understanding and reasoning abilities Objective: 2.3 Learning Outcome: Identify the basic concepts of and tools used for business accounting Skill: Concept 63) Moia is a successful entrepreneur who owns several popular pastry shops. She often imports macadamia nuts from countries with poor economic conditions, which has furthered her desire to help improve the lives of the people living there. How can Moia best exhibit corporate social responsibility to the citizens of these nations? A) by including labels on her products indicating where the nuts come from B) by hiring people who have traveled to these nations C) by opening pastry shops in these countries that would create jobs for the citizens there D) by purchasing the nuts more regularly E) by shipping free pastries to these nations Answer: C Explanation: C) A company that is socially responsible makes ethical decisions in the area of ethical sourcing and procurement, and community and "good neighbor" policies. Page Ref: 48 Difficulty: Moderate AACSB: Reflective thinking skills Objective: 2.4 Learning Outcome: Discuss the roles of ethics and corporate responsibility in business Skill: Application 64) Moia is a successful entrepreneur who owns several popular pastry shops. She often imports macadamia nuts from countries with poor economic conditions, which has furthered her desire to help improve the lives of the people living there. Which of the following challenges would Moia likely find to be MOST central in seeking to run both a socially responsible and successful business? A) how to engage in fair marketing practices that are cost-effective B) how to pay her overseas workers fair wages, sell her pastries for less there, and still make the necessary profit C) how to obtain the best quality macadamia nuts at a much lower price than they are worth D) how to decrease employee benefits without experiencing subsequent complaints E) how to perform a social audit without taking up too much valuable work time Answer: B Explanation: B) Being socially responsible involves ethical sourcing and procurement, as well as making ethical decisions in human rights and employment standards in the workplace, but it is a skill to make these ethical decisions and continue to grow as a financially successful business. Page Ref: 48 Difficulty: Moderate Objective: 2.4 Learning Outcome: Discuss the roles of ethics and corporate responsibility in business Skill: Synthesis 65) What is a social audit? A) an examination of a company's financial records to determine their conformance with GAAP B) a program to promote consumer activism C) an analysis by the government of a company's compliance with employment laws D) an analysis of a firm's success in using funds earmarked for meeting its social responsibility goals E) a check on consumer rights Answer: D Explanation: D) Occasional analyses will help businesses determine whether their social responsibility goals are being effectively and efficiently met. Page Ref: 50 Difficulty: Moderate AACSB: Ethical understanding and reasoning abilities Objective: 2.4 Learning Outcome: Discuss the roles of ethics and corporate responsibility in business Skill: Concept 66) A husband and wife have just opened a gymnastics center, with the hope of training future Olympians. They desire to run their center in a socially responsible way. Which of the following should they certainly avoid? A) purchase of substandard gym equipment B) advertising methods intended to profit the center C) initiatives that decrease the satisfaction of investors D) doubling of enrollment fees for the program E) provision of attractive salaries for employees Answer: A Explanation: A) A socially responsible company makes ethical decisions with safety concerns. Substandard gym equipment hinders safety. Page Ref: 50 Difficulty: Moderate Objective: 2.4 Learning Outcome: Discuss the roles of ethics and corporate responsibility in business Skill: Application 67) Tiffany's Taffy Shop is known for its commitment to corporate social responsibility. However, the owner, Mrs. Wu, has learned that an employee has been unknowingly ordering candy wrappers that are harmful to the environment. How could Mrs. Wu best recover from this ethical lapse? A) by firing the employee immediately B) by seeking ways to remedy any harm that may have been caused C) by considering ethics training for all employees D) by ordering candy wrappers that are less expensive E) by decreasing the amount of taffy sold Answer: B Explanation: B) By seeking remedy for harm caused by the environmentally unsound candy wrappers, Tiffany will come back into alignment with the social responsibility tenet of making ethical decisions for the environment. Page Ref: 50 Difficulty: Moderate AACSB: Reflective thinking skills Objective: 2.4 Learning Outcome: Discuss the roles of ethics and corporate responsibility in business Skill: Synthesis 68) Pamela is the manager of a local eyewear store. She wants to ensure that the ethical conduct of her employees is above reproach. Which of the following should Pamela avoid doing? A) underscoring the importance of ethics to all employees B) encouraging and equipping employees to follow the company's ethical standards C) modeling behavior that employees should adopt D) relying on management alone to report ethics violations E) ensuring that a strict code of ethics is in place Answer: D Explanation: D) Management at very high strategic levels needs to have a common vision communicated to the company as a whole of how to enforce a code of ethics. Page Ref: 50 Difficulty: Moderate AACSB: Reflective thinking skills Objective: 2.4 Learning Outcome: Discuss the roles of ethics and corporate responsibility in business Skill: Concept 69) The Omega Group's mission statement conveys a primary commitment to compassionately improving the lives of elderly citizens. What is LEAST likely to be true of the company's ethical code? A) The company values the well-being of others. B) The company believes that all people have worth. C) The company prohibits age discrimination when hiring employees. D) The company expects and requires elderly citizens to contribute to society. E) The company prioritizes fair treatment of all human beings. Answer: D Explanation: D) The Omega Group's stated mission is to improve the lives of elderly citizens, not to expect them to contribute to society. Page Ref: 50 Difficulty: Moderate AACSB: Reflective thinking skills Objective: 2.4 Learning Outcome: Discuss the roles of ethics and corporate responsibility in business Skill: Application 70) Carla's Collegiate Exchange, a popular bookseller among university students, has just drafted its mission statement. The company seeks to sell books on a sliding scale based on a student's available income. What does this MOST likely reveal about the company's ethical code? A) The company believes financially challenged students should not be hindered from succeeding educationally. B) The company believes that poor individuals should be given a greater chance at success than are wealthy individuals. C) The company is indifferent to people's financial well-being. D) The company is not concerned with fostering the success of others. E) The company is most motivated by a desire for greater profits. Answer: A Explanation: A) This statement is in support of Carla's Collegiate Exchange and its mission to provide feasible financial options to low-income students. Page Ref: 50 Difficulty: Moderate AACSB: Reflective thinking skills Objective: 2.4 Learning Outcome: Discuss the roles of ethics and corporate responsibility in business Skill: Application 71) Wei-Yin runs a small, socially responsible business in which she sells her own watercolor paintings. Wei-Yin has a passion to help aspiring artists in her area achieve their dreams, but so far has assisted only a few of them. Which of the following would MOST likely be a reason for this? A) Wei-Yin does not have much time or money to devote to this desire. B) Wei-Yin is consumed with making a large profit in her business. C) Wei-Yin fears that helping more artists will mean more competition for her company. D) Wei-Yin does not have a sound ethical code in place. E) Wei-Yin does not wish to engage in corporate philanthropy. Answer: A Explanation: A) If Wei-Yin has an interest in helping other artists but does not carry it out fully, it is most likely because she is not able to fully pursue it with limited resources. Page Ref: 51 Difficulty: Moderate Objective: 2.5 Learning Outcome: Discuss the roles of ethics and corporate responsibility in business Skill: Synthesis 72) The CEO of Perry's Pizzeria, a rapidly growing restaurant chain, is committed to a high level of corporate social responsibility along with continued business success. Which of the following would likely be a failure in the area of legal compliance for the pizzeria? A) Ingredients used in Perry's pan pizza are not as healthy as those of other companies' pizza. B) A teenager is not hired due to being wheelchair-bound, even though she would be capable of performing her duties. C) An employee is fired after she is caught stealing money from the cash register. D) Persons with certain food allergies are not hired as employees. E) Perry's signature dessert is advertised as being made from a popular chocolate bar. Answer: B Explanation: B) Measures such as the Americans with Disabilities Act of 1990 require companies to make a reasonable accommodation to the known disabilities of an applicant or employee, as long as it doesn't require undue hardship for the employer. Page Ref: 51 Difficulty: Moderate AACSB: Reflective thinking skills Objective: 2.5 Learning Outcome: Discuss the roles of ethics and corporate responsibility in business Skill: Application 73) Which of the following steps would be LEAST likely to help a business develop and maintain an ethical environment? A) Ensure that a mission statement is in place. B) Communicate standards of behavior to all employees. C) Implement ethics training programs. D) Offer orientation programs so that new employees understand the business's ethical standards. E) Trust that management-level employees will act ethically. Answer: E Explanation: E) An ethical environment is created by modeling ethical behavior, and to do this there needs to be clear communication of standards coming from the top down; it is not enough to trust that management will act ethically on its own. Page Ref: 36 Difficulty: Difficult AACSB: Ethical understanding and reasoning abilities Objective: 2.1 Learning Outcome: Discuss the roles of ethics and corporate responsibility in business Skill: Concept 74) When evaluating a decision based on the ethical norm of utility, a manager is most likely to consider which of the following questions? A) Is the decision consistent with what we regard as fair? B) Does the decision respect the rights of the individuals involved? C) Is the decision consistent with people's responsibility to each other? D) Is the decision consistent with best management practice? E) Does the decision optimize the benefits for those who are affected by it? Answer: E Explanation: E) The ethical norm of utility addresses those affected by a decision and whether the decision optimizes what is best for those individuals. Page Ref: 36 Difficulty: Difficult AACSB: Reflective thinking skills Objective: 2.1 Learning Outcome: Discuss the roles of ethics and corporate responsibility in business Skill: Concept 75) When evaluating a decision based on the ethical norm of rights, a manager is most likely to consider which of the following questions? A) Is the decision consistent with what we regard as fair? B) Does the decision respect the individuals involved? C) Is the decision consistent with people's responsibilities to each other? D) Does the decision optimize the benefits for those who are affected by it? E) Does the decision comply with current legislation? Answer: B Explanation: B) The ethical norm of rights addresses the rights of individuals involved with the decision and whether the decision respects those individuals' rights. Page Ref: 36 Difficulty: Difficult AACSB: Reflective thinking skills Objective: 2.1 Learning Outcome: Discuss the roles of ethics and corporate responsibility in business Skill: Concept 76) What is the most effective step that a company can take to promote ethical behavior? A) conduct frequent comprehensive audits of all financial information B) have employees sign a code of conduct upon hiring C) demonstrate top management support of ethical standards D) publicize efforts to become more ethical E) survey employees on their personal ethical codes Answer: C Explanation: C) Such a policy contributes to a corporate culture that values ethical standards and announces that the firm is as concerned with good citizenship as with profits. Page Ref: 36 Difficulty: Moderate AACSB: Ethical understanding and reasoning abilities Objective: 2.1 Learning Outcome: Discuss the roles of ethics and corporate responsibility in business Skill: Concept 77) Which of the following statements about corporate social responsibility is true? A) It cannot legitimately be influenced by a company's competing interests. B) It does not include issues raised by marketing and advertising. C) It must be concerned only with the well-being of employees and consumers. D) Environmental matters must be addressed at the expense of profits. E) It benefits a business by increasing efficiency and sales. Answer: E Explanation: E) Having a strong and clear ethical policy, or taking part in corporate social responsibility, leads companies to use materials efficiently, minimize waste, and increase sales through product innovation and environmentally and ethically conscious labeling. Page Ref: 38 Difficulty: Difficult AACSB: Ethical understanding and reasoning abilities Objective: 2.1 Learning Outcome: Discuss the roles of ethics and corporate responsibility in business Skill: Concept 78) Which of the following steps would be LEAST likely to help a business develop and maintain an ethical environment? A) Ensure that a mission statement is in place. B) Communicate standards of behavior to all employees. C) Implement ethics training programs. D) Offer orientation programs so that new employees understand the business's ethical standards. E) Trust that employees will act ethically. Answer: E Explanation: E) An ethical environment is created by modeling ethical behavior. It is not enough to trust that employees will already have sufficient ethical values in place. Page Ref: 34 Difficulty: Difficult AACSB: Ethical understanding and reasoning abilities Objective: 2.1 Learning Outcome: Discuss the roles of ethics and corporate responsibility in business Skill: Concept 79) Which of the following questions would a company dedicated to increasing its level of corporate social responsibility be LEAST likely to ask? A) "How can we afford to pay our employees more?" B) "Should we offer more sales in order to better serve our customers?" C) "Can we purchase better quality ingredients while still making a profit?" D) "Which charitable donations will make our investors most happy?" E) "If our profits go up, can we lay off some of our employees?" Answer: E Explanation: E) A company's participation in a strong corporate social responsibility plan allows it to serve its local and global communities in the area of community and "good neighbor" polices, but it also ideally benefits the corporation's employees in a direct way too. Page Ref: 34 Difficulty: Difficult AACSB: Reflective thinking skills Objective: 2.1 Learning Outcome: Discuss the roles of ethics and corporate responsibility in business Skill: Application 80) Zach's company has appointed him to boost the business's level of corporate social responsibility. Which of the following would MOST likely be an appropriate step for Zach toward achieving this task? A) extending employees' hours while paying them the same B) placing less priority on the satisfaction of investors C) reducing the company's number of charitable contributions D) offering employees better health benefits while still increasing the company's profit E) suspending standards designed to protect the environment Answer: D Explanation: D) A company's participation in a strong corporate social responsibility plan allows it to serve its local and global communities in the area of human rights and employment standards in the workplace, but it also ideally benefits the corporation in a direct way. Page Ref: 40 Difficulty: Moderate AACSB: Reflective thinking skills Objective: 2.1 Learning Outcome: Discuss the roles of ethics and corporate responsibility in business Skill: Application 81) A university has filed a formal complaint against a local hospital for failing to be socially responsible in its treatment of university students completing internships there. Which of the following would most likely be a valid response by a hospital that was NOT guilty of this charge? A) "The responsibility to take disciplinary action against the supervisor who physically threatened the student interns lies with someone else." B) "Because sanitary conditions are very important, floors are mopped regularly and are sometimes left wet. We cannot be held responsible if student interns unfamiliar with this practice slip and injure themselves." C) "All student interns are clearly instructed to wear gloves and protective covering when handling blood samples that may endanger the interns' health." D) "We expect our student interns to work double shifts after a day of classes so that they will be well prepared for a career in the health professions." E) "Unforeseen budget cuts have forced us to revoke our contracts with student interns regarding hourly compensation." Answer: C Explanation: C) A socially responsible hospital makes decisions that promote health and safety, as well as maintain high employment standards in the workplace. Page Ref: 40 Difficulty: Difficult Objective: 2.2 Learning Outcome: Discuss the roles of ethics and corporate responsibility in business Skill: Application 82) Save-a-Bunch Hardware has tripled its prices for plywood and other building supplies after a tornado strikes the area. What business practice is Save-a-Bunch Hardware demonstrating? A) price gouging B) price wars C) price fixing D) collusion E) buying out competitors Answer: A Explanation: A) When firms respond to increased demand with overly steep prices, they are engaging in price gouging. Page Ref: 44 Difficulty: Moderate AACSB: Reflective thinking skills Objective: 2.3 Learning Outcome: Discuss strategies for setting and adjusting prices Skill: Application 83) IBP, a leading meat-processing firm, has a long record of breaking environmental protection, labor, and food processing laws and then trying to cover up its offenses. What type of stance has IBP adopted to social responsibility? A) accommodative B) obstructionist C) pragmatic D) defensive E) proactive Answer: B Explanation: B) Firms who take this stance do as little as possible to be responsible. Page Ref: 48 Difficulty: Moderate AACSB: Reflective thinking skills Objective: 2.4 Learning Outcome: Discuss the roles of ethics and corporate responsibility in business Skill: Application 84) Tyrone, the president of an architectural firm, was faced with a difficult decision when asked to decrease the portion of his company's budget designated to help construct new homeless shelters. After considering his personal ethics and seeking the advice of coworkers, Tyrone decided he would not cut these funds—even though his decision would mean the company could not immediately move forward on another initiative. Which of the following is MOST likely to be true of Tyrone's decision? A) Concern for the company's profitability did not figure into the decision. B) When Tyrone asked a dozen coworkers what they would do if they were in his shoes, he received mixed counsel. C) Tyrone's values and morals did not help him make the decision. D) If someone asks Tyrone to make a similar budget cut in the future, he will know to act as he did before without thinking through the specific situation. E) Tyrone does not care about his company's success. Answer: B Explanation: B) Tyrone most likely received mixed counsel because some people probably had a strong belief that constructing homeless shelters is the morally correct thing to do for the homeless, while others probably felt advancing the company was the morally correct thing to do for stakeholders. Page Ref: 48 Difficulty: Difficult AACSB: Reflective thinking skills Objective: 2.4 Learning Outcome: Discuss the roles of ethics and corporate responsibility in business Skill: Critical Thinking 85) Tyrone, the president of an architectural firm, was faced with a difficult decision when asked to decrease the portion of his company's budget designated to help construct new homeless shelters. After considering his personal ethics and seeking the advice of coworkers, Tyrone decided he would not cut these funds—even though his decision would mean the company could not immediately move forward on another initiative. Caleb serves on the ethics committee at Tyrone's company. He disagrees with and is upset about Tyrone's decision. Based on this information, which of the following do Caleb and Tyrone most likely disagree about? A) marketing issues B) ethical sourcing C) social auditing D) corporate philanthropy E) environmental protection Answer: D Explanation: D) Companies that participate in corporate philanthropy donate some of their profits or resources to charitable organizations. Tyrone and Caleb most likely degree about whether or not the company should practice corporate philanthropy. Page Ref: 48 Difficulty: Difficult AACSB: Reflective thinking skills Objective: 2.4 Learning Outcome: Discuss the roles of ethics and corporate responsibility in business Skill: Critical Thinking 86) The mission statement of Coco's Chocolate & Confectioners reveals a primary emphasis on ethical sourcing and procurement. Which action below would the company MOST likely seek to avoid? A) neglecting the welfare of the region from which it imports cacao beans B) reducing the size of their marketing department C) adding a new line of fat-free confections D) splurging on a company-wide holiday party E) decreasing health care benefits for employees Answer: A Explanation: A) Ethical sourcing and procurement require finding a source for raw materials and making agreements with suppliers that are morally sound. Page Ref: 48 Difficulty: Moderate Objective: 2.4 Learning Outcome: Discuss the roles of ethics and corporate responsibility in business Skill: Application 87) Acme Inc. meets its legal and ethical requirements. In addition, Acme donates $50,000 per year to local charities when asked. Which of the following approaches is being taken by Acme toward social responsibility? A) obstructionist B) accommodative C) defensive D) proactive E) reactive Answer: B Explanation: B) Firms that adopt an accommodative stance meet legal and ethical requirements but also go further in certain cases. Page Ref: 48 Difficulty: Moderate AACSB: Reflective thinking skills Objective: 2.4 Learning Outcome: Discuss the roles of ethics and corporate responsibility in business Skill: Application 88) Which statement would people in favor of socially responsible business most likely advocate? A) Maximizing profits is the core goal of the corporation. B) The corporation is part of the civic world. C) Other agencies have the skill to develop social programs. D) Vast corporate resources are being overused. E) Civic responsibility needs to be centralized. Answer: B Explanation: B) Proponents of socially responsible business believe that corporations are citizens and should help to improve the lives of fellow citizens. Page Ref: 50 Difficulty: Difficult AACSB: Reflective thinking skills Objective: 2.4 Learning Outcome: Discuss the roles of ethics and corporate responsibility in business Skill: Concept Chapter 3 Entrepreneurship, New Ventures, and Business Ownership 34) Which entity is defined as independent and has little influence on its market? A) corporation B) joint venture C) small business D) partnership E) government agency Answer: C Explanation: C) A small business is defined as independent with marginal influence in its market. Page Ref: 60 Difficulty: Easy Objective: 3.1 Learning Outcome: Describe the role of small businesses in the economy Skill: Concept 35) Entrepreneurs often have more than one goal for starting a business and becoming self-employed. Which of the following reasons is the LEAST common goal of entrepreneurs? A) to seek independence and be their own boss B) to earn a comfortable living for a period of time C) to grow and expand a current business D) to meet community or social obligations E) to provide for their financial needs Answer: D Explanation: D) Some entrepreneurs may start a business with the goal of meeting community or social obligations, but the other goals are far more common. Page Ref: 64 Difficulty: Moderate Objective: 3.2 Learning Outcome: Explain the entrepreneurship process Skill: Concept 36) What is the name of the document in which an entrepreneur summarizes his or her business strategy for a proposed new venture? A) success chart B) director's guideline C) financial blueprint D) business plan E) balance sheet Answer: D Explanation: D) A business plan is a summary of the business strategy for the proposed new venture and how that strategy will be implemented. Page Ref: 65 Difficulty: Easy Objective: 3.3 Learning Outcome: Describe the role of small businesses in the economy Skill: Concept 37) What is the most significant disadvantage of owning a franchise? A) start-up costs B) operational guidelines C) difficulty obtaining loans D) double taxation E) competition Answer: A Explanation: A) Start-up costs are considered one of the most significant disadvantages of owning a franchise. Page Ref: 66 Difficulty: Easy Objective: 3.3 Learning Outcome: Explain the entrepreneurship process Skill: Concept 38) Which of the following is the most significant disadvantage of starting a business from scratch? A) having few employees B) inability to franchise C) having competition D) coming up with new ideas E) the risk of failure Answer: E Explanation: E) There is much greater risk involved in starting a new business from scratch than in acquiring an already-existing business. Page Ref: 67 Difficulty: Easy AACSB: Dynamics of the global economy Objective: 3.3 Learning Outcome: Explain the entrepreneurship process Skill: Concept 39) What is the most important source of money for new business start-ups? A) banks B) the SBA C) personal resources D) government grants E) venture capital Answer: C Explanation: C) Most individuals starting a new business have to rely on personal finances to do so. Page Ref: 67 Difficulty: Easy Objective: 3.3 Learning Outcome: Discuss the functions of different types of financial institutions in the monetary system Skill: Concept 40) Who constitutes the membership of SCORE? A) retired executives B) venture capitalists C) company officers D) commercial lenders E) successful entrepreneurs Answer: A Explanation: A) The Service Corps of Retired Executives is made up of retired executives who volunteer to help entrepreneurs start new businesses. Page Ref: 68 Difficulty: Easy Objective: 3.3 Learning Outcome: Explain the entrepreneurship process Skill: Concept 41) Which of the following represents the fastest-growing segment of minority business ownership? A) African Americans B) Asians C) Hispanics D) Pacific Islanders E) Europeans Answer: A Explanation: A) The number of businesses owned by African Americans increased by 48 percent during the most recent five-year period for which data is available. Page Ref: 69 Difficulty: Easy AACSB: Multicultural and diversity understanding Objective: 3.4 Learning Outcome: Explain the entrepreneurship process Skill: Concept 42) Which of the following types of businesses is most common in the United States? A) sole proprietorships B) general partnerships C) cooperatives D) corporations E) limited partnerships Answer: A Explanation: A) About 72 percent of all U.S. businesses are sole proprietorships. Page Ref: 72 Difficulty: Easy Objective: 3.5 Learning Outcome: Describe the role of small businesses in the economy Skill: Concept 43) What percent of a business is the least amount that a master partner will retain? A) 10 percent B) 25 percent C) 50 percent D) 75 percent E) 80 percent Answer: C Explanation: C) The master partner retains at least 50 percent ownership and runs the business, while minority partners have no management voice. Page Ref: 74 Difficulty: Easy Objective: 3.5 Learning Outcome: Describe the role of small businesses in the economy Skill: Concept 44) In which business sector are cooperatives still important? A) manufacturing B) automotive C) public utilities D) agriculture E) health care Answer: D Explanation: D) Although cooperatives make up only a minor segment of the U.S. economy, the role of cooperatives is important in agriculture. Page Ref: 74 Difficulty: Moderate AACSB: Dynamics of the global economy Objective: 3.5 Learning Outcome: Explain the entrepreneurship process Skill: Concept 45) Which form of business ownership generates the most sales in the United States? A) sole proprietorship B) general partnership C) cooperative D) corporation E) limited partnership Answer: D Explanation: D) Corporations generate up to 85 percent of the total sales revenue of all businesses. Page Ref: 75 Difficulty: Moderate Objective: 3.6 Learning Outcome: Explain the entrepreneurship process Skill: Concept 46) Which type of business is legally considered a separate entity from its owners and is liable for its own debts? A) sole proprietorship B) corporation C) limited partnership D) cooperative E) general partnership Answer: B Explanation: B) A corporation is a business that is legally considered an entity separate from its owners and is liable for its own debts. The liability of the owners extends to the limits of their investments. Page Ref: 75 Difficulty: Easy Objective: 3.6 Learning Outcome: Explain the entrepreneurship process Skill: Concept 47) Who are the owners of a corporation? A) board of directors B) financial officers C) stockholders D) top managers E) entrepreneurs Answer: C Explanation: C) Stockholders are the owners of shares of stock in a corporation. Page Ref: 77 Difficulty: Easy Objective: 3.6 Learning Outcome: Explain the entrepreneurship process Skill: Concept 48) What are shares in a business that the business sells called? A) bonds B) profits C) stocks D) dividends E) loans Answer: C Explanation: C) Shares in a corporation are called stocks. Page Ref: 77 Difficulty: Easy Objective: 3.6 Learning Outcome: Describe the goals and stages of financial management for companies Skill: Concept 49) Suppose BrightLights Online Video purchases another company, QuikFlix. What is this an example of? A) merger B) spin-off C) divestiture D) acquisition E) collapse Answer: D Explanation: D) An acquisition is when one company buys another company outright. Page Ref: 78 Difficulty: Easy Objective: 3.6 Learning Outcome: Describe the goals and stages of financial management for companies Skill: Application 50) The U.S. Department of Commerce considers a business "small" if it has fewer than how many employees? A) 10 B) 50 C) 100 D) 500 E) 1,500 Answer: D Explanation: D) The U.S. Department of Commerce considers a business "small" if it has fewer than 500 employees. Page Ref: 60 Difficulty: Moderate Objective: 3.1 Learning Outcome: Describe the role of small businesses in the economy Skill: Concept 51) Which of the following industry groups is the fastest growing segment of small business enterprise? A) manufacturing B) wholesaling C) retailing D) transportation E) services Answer: E Explanation: E) Services is the fastest growing segment, probably because services require few resources. Page Ref: 62 Difficulty: Moderate AACSB: Dynamics of the global economy Objective: 3.1 Learning Outcome: Describe the role of small businesses in the economy Skill: Concept 52) Which of the following types of venue is favored by small-business retailers? A) superstore B) catalog showroom C) specialty shop D) department store E) studio workroom Answer: C Explanation: C) Usually, people who start small retail businesses favor specialty shops that let them focus limited resources on narrow market segments. Page Ref: 62 Difficulty: Moderate Objective: 3.1 Learning Outcome: Describe the role of small businesses in the economy Skill: Concept 53) What does a start-up company need in order to estimate the required size of a plant, store, or office, to decide how much inventory to carry, and to determine how many employees to hire? A) sales forecast B) tax estimate C) business objective D) stock options E) funding source Answer: A Explanation: A) In addition, a sales forecast helps the entrepreneur demonstrate an understanding of the current market, the strengths and weaknesses of existing firms, and the means by which the new venture will compete. Page Ref: 65 Difficulty: Moderate Objective: 3.3 Learning Outcome: Explain the entrepreneurship process Skill: Concept 54) What are specific locations of Ramada, McDonald's, and Subway examples of? A) government entities B) university sponsored businesses C) franchises D) networking businesses E) cooperatives Answer: C Explanation: C) All of these businesses represent an agreement between a franchisee and a franchiser. The franchisee buys the rights for a location from the franchiser, and runs the business independently. Page Ref: 66 Difficulty: Moderate AACSB: Dynamics of the global economy Objective: 3.3 Learning Outcome: Describe the role of small businesses in the economy Skill: Application 55) Which of the following is the most significant disadvantage of franchising? A) high start-up costs B) risk of failure C) difficulty in obtaining financing D) lengthy contracts E) high cost of operation Answer: A Explanation: A) Start-up franchise prices tend to be high—the start-up fee for a McDonald's franchise is $45,000. Page Ref: 66 Difficulty: Moderate Objective: 3.3 Learning Outcome: Describe the role of small businesses in the economy Skill: Concept 56) What is the major drawback of accepting venture capital? A) sharing company control B) having to pay high interest rates C) limiting expansion D) facing a short repayment period E) having to write a proposal Answer: A Explanation: A) Venture capitalists focus on turning a profit on their investments. In order to better ensure that they do, they typically invest in a business on the condition that they be given some control over the company. Page Ref: 68 Difficulty: Moderate AACSB: Reflective thinking skills Objective: 3.3 Learning Outcome: Explain the entrepreneurship process Skill: Concept 57) Which of the following are groups of small investors seeking to make profits on companies with rapid growth potential? A) community banks B) venture capital companies C) equity managers D) government securities dealers E) stock brokers Answer: B Explanation: B) Most venture capital companies do not lend money; instead, they supply capital in return for partial ownership or representation on boards of directors. Page Ref: 68 Difficulty: Moderate Objective: 3.3 Learning Outcome: Discuss the functions of different types of financial institutions in the monetary system Skill: Concept 58) Which of the following is federally licensed to borrow money from the Small Business Administration to invest in or lend to small businesses? A) SCORE B) SBICs C) SBDC D) FDIC E) SEC Answer: B Explanation: B) In addition, small-business investment companies are themselves investments for their shareholders. Page Ref: 68 Difficulty: Moderate Objective: 3.3 Learning Outcome: Explain the entrepreneurship process Skill: Concept 59) Which of the following helps entrepreneurs gain skills that are essential for running a business? A) ESOP B) SBICs C) SEC D) LLC E) SBA Answer: E Explanation: E) The Small Business Administration helps entrepreneurs improve their management skills. Page Ref: 68 Difficulty: Moderate Objective: 3.3 Learning Outcome: Explain the entrepreneurship process Skill: Concept 60) Which of the following is the most significant recent trend in small-business start-ups? A) entrepreneurs who cross over from big business B) increased opportunities for minorities C) increased opportunities for women D) emergence of e-commerce E) global economic downturn Answer: D Explanation: D) Because the Internet provides fundamentally new ways of doing business, entrepreneurs have created and expanded new businesses faster than ever before. Page Ref: 68 Difficulty: Moderate AACSB: Dynamics of the global economy Objective: 3.4 Learning Outcome: Explain the entrepreneurship process Skill: Concept 61) According to SBA estimates, what percent of all new businesses can expect to survive for at least four years? A) 14 percent B) 24 percent C) 44 percent D) 64 percent E) 84 percent Answer: C Explanation: C) During the 1960s and 1970s, less than half of all new start-ups survived more than 18 months; only one in five lasted 10 years. Now, however, 44 percent can expect to survive for at least four years. Page Ref: 69 Difficulty: Moderate Objective: 3.4 Learning Outcome: Explain the entrepreneurship process Skill: Concept 62) Which of the following factors most contributes to small business failure? A) managerial incompetence or inexperience B) bad product design C) legal problems D) time spent raising capital E) personnel issues Answer: A Explanation: A) The four main reasons for small business failure are managerial incompetence, neglect, weak control systems, and insufficient capital. Page Ref: 70 Difficulty: Moderate Objective: 3.4 Learning Outcome: Describe the role of small businesses in the economy Skill: Concept 63) A new business should have enough capital to operate at least how many months without earning a profit? A) 1 B) 2 C) 4 D) 6 E) 9 Answer: D Explanation: D) In addition, some experts recommend having enough capital to last one year. Page Ref: 70 Difficulty: Moderate Objective: 3.4 Learning Outcome: Explain the entrepreneurship process Skill: Concept 64) Which of the following legal forms of business is owned and usually operated by a person who is responsible for its debts? A) cartel B) corporation C) general partnership D) sole proprietorship E) limited partnership Answer: D Explanation: D) About 72 percent of all businesses are sole proprietorships; however, they account for only about 5 percent of total business revenues. Page Ref: 72 Difficulty: Easy Objective: 3.5 Learning Outcome: Describe the role of small businesses in the economy Skill: Concept 65) What is a major drawback of sole proprietorships? A) limited funding B) work flexibility C) low start-up costs D) unlimited liability E) structured hours Answer: D Explanation: D) A sole proprietor is responsible for all debts incurred by the business, which increases the risk involved in doing business. Page Ref: 72 Difficulty: Moderate Objective: 3.5 Learning Outcome: Describe the role of small businesses in the economy Skill: Concept 66) When can a sole proprietorship legally be dissolved? A) sales exceed $1 million B) the partners reorganize the firm's structure C) earnings are less than $500,000 D) the owner donates profits E) the owner dies Answer: E Explanation: E) Although the business can be reorganized by a successor, executors or heirs must otherwise sell its assets. Page Ref: 72 Difficulty: Moderate Objective: 3.5 Learning Outcome: Describe the role of small businesses in the economy Skill: Concept 67) What resources does a sole proprietorship primarily depend on? A) the corporation B) foreign investors C) capital markets D) those of the owner E) a committed clientele Answer: D Explanation: D) Therefore, sole proprietors often find it hard to borrow money to start up or expand; many bankers fear that they will not be able to recover loans if owners become disabled or insolvent. Page Ref: 72 Difficulty: Moderate Objective: 3.5 Learning Outcome: Describe the role of small businesses in the economy Skill: Concept 68) What type of business has two or more owners who share in the operation of the firm and are financially responsible for its debts? A) corporation B) partnership C) cooperative D) conglomerate E) nonprofit Answer: B Explanation: B) Partners may invest equal or unequal sums of money and may earn profits that bear no relation to their investments. Page Ref: 73 Difficulty: Moderate Objective: 3.5 Learning Outcome: Describe the role of small businesses in the economy Skill: Concept 69) What is the most common type of partnership? A) limited partnership B) corporate partnership C) general partnership D) sole partnership E) cooperative partnership Answer: C Explanation: C) A general partnership is similar to a sole proprietorship but is owned by more than one person. Page Ref: 73 Difficulty: Moderate Objective: 3.5 Learning Outcome: Describe the role of small businesses in the economy Skill: Concept 70) Who invests all of the funds needed for a business but plays no role in its management? A) silent partner B) sole proprietor C) general partner D) principal E) director Answer: A Explanation: A) A silent partner is a partner who invests all of the funds needed for a business and therefore gets a return on the investment if the business earns a profit, but plays no role in its management. Page Ref: 73 Difficulty: Moderate Objective: 3.5 Learning Outcome: Describe the role of small businesses in the economy Skill: Concept 71) What is the most important advantage of general partnerships? A) the unlimited liability of the partnership B) the ability to grow with the addition of new talent and money C) the ease of implementing an effective control system D) the increased role of luck E) the need for minority partners Answer: B Explanation: B) New partners can join a general partnership at any time by investing their time and money. Page Ref: 73 Difficulty: Moderate Objective: 3.5 Learning Outcome: Describe the role of small businesses in the economy Skill: Concept 72) What do both sole proprietorships and partnerships lack? A) trust B) legal standing C) continuity D) shared vision E) adaptable processes Answer: C Explanation: C) When one partner dies or leaves, the original partnership dissolves, even if one or more of the other partners want it to continue. Page Ref: 73 Difficulty: Moderate Objective: 3.5 Learning Outcome: Describe the role of small businesses in the economy Skill: Concept 73) Which of the following is a partner who actively manages a firm and has unlimited liability for its debts? A) limited partner B) general partner C) stockholding partner D) public partner E) silent partner Answer: B Explanation: B) A general partner is also known as an active partner because he or she plays an active role in the management of the partnership. Page Ref: 74 Difficulty: Moderate Objective: 3.5 Learning Outcome: Describe the role of small businesses in the economy Skill: Concept 74) What percent of all businesses in the United States are corporations? A) 15 percent B) 20 percent C) 50 percent D) 75 percent E) 85 percent Answer: B Explanation: B) There are about 4.93 million corporations in the United States; this accounts for about 20 percent of all U.S. businesses. Page Ref: 74 Difficulty: Moderate AACSB: Dynamics of the global economy Objective: 3.6 Learning Outcome: Describe the role of small businesses in the economy Skill: Concept 75) What type of company has stock that is widely held and available for sale to the general public? A) private corporation B) public corporation C) general partnership D) limited liability corporation E) professional corporation Answer: B Explanation: B) When shares are publicly issued, the firm becomes a publicly held (or public) corporation; stock is widely held and available for sale to the public. Page Ref: 76 Difficulty: Moderate Objective: 3.6 Learning Outcome: Describe the goals and stages of financial management for companies Skill: Concept 76) What type of corporations are those comprised of doctors, lawyers, or accountants most likely to be? A) private B) public C) subchapter S D) professional E) multinational Answer: D Explanation: D) While the corporate structure protects members from unlimited financial liability, they are not immune from unlimited liability and are subject to partnership taxation. Page Ref: 77 Difficulty: Moderate Objective: 3.6 Learning Outcome: Explain the entrepreneurship process Skill: Concept 77) Two businesses, Rio Books and Devereaux Holdings, collaborate to create and run a new bookselling business. What is this action called? A) corporatization B) vertical merger C) joint venture D) strategic alliance E) institutional investment Answer: C Explanation: C) Joint ventures abound in numerous industries. Unlike in a strategic alliance, each partner in a joint venture owns a share of the new enterprise. Page Ref: 78 Difficulty: Moderate AACSB: Reflective thinking skills Objective: 3.6 Learning Outcome: Explain the entrepreneurship process Skill: Application 78) Which of the following occurs when two firms combine to create a new company? A) acquisition B) takeover C) venture D) divestiture E) merger Answer: E Explanation: E) A merger occurs when Company A, for example, merges with Company B to create Company C. Page Ref: 78 Difficulty: Easy Objective: 3.6 Learning Outcome: Explain the entrepreneurship process Skill: Concept 79) Why is it difficult to compare relative job growth for different-sized businesses? A) Sourcing up-to-date employment figures is difficult. B) Many small businesses experience rapid unreported growth. C) Spheres of influence overlap between the small and big business sectors. D) Staff size of small businesses tends to change faster than staff size of big businesses. E) It is hard to determine the cutoff point at which a small business becomes a large business. Answer: E Explanation: E) Relative job growth among businesses of different sizes is not easy to determine. There are different parameters governing the size of a small versus a big business and additionally there is no precise point where such a differentiation can be clearly identified. Page Ref: 61 Difficulty: Difficult AACSB: Analytic skills Objective: 3.1 Learning Outcome: Describe the role of small businesses in the economy Skill: Concept 80) Why do attitudes about entrepreneurship vary internationally? A) Decision making in big business can overlook local customs. B) Attitudes towards risk-taking in business are culturally determined. C) Consumer attitudes are influenced by economic cycles. D) Foreign-owned companies can be unresponsive to local markets. E) International business is creating a global village. Answer: B Explanation: B) In many cultures, such as the United States, the entrepreneurial spirit is highly valued. However, in some Asian countries, the entrepreneurial spirit is often tempered by the need for consensus. Page Ref: 63 Difficulty: Difficult AACSB: Multicultural and diversity understanding Objective: 3.1 Learning Outcome: Describe the role of small businesses in the economy Skill: Concept 81) Olivia has found a viable business, a small furniture store, she is interested in purchasing. She has learned that the store is supplied by reliable vendors and has a steady stream of local and online buyers. What should be Olivia's next step at this point? A) Analyze the furniture store's past financial statements. B) Start negotiating a purchase price with the current owner. C) Approach a banking institution about financing her purchase. D) Consult a marketing agency about ways to reach more customers. E) Search for vendors that can provide similar products at lower costs. Answer: A Explanation: A) While what Olivia has learned about the state of the business is positive, it is still in her best interest to continue to do due diligence and more closely examine the business. Page Ref: 66 Difficulty: Difficult AACSB: Reflective thinking skills Objective: 3.3 Learning Outcome: Explain the entrepreneurship process Skill: Application 82) Viola wants to be a business owner, and is contemplating whether to start her own business or open a franchise. Which of the following, if true, would be a compelling reason for Viola to open a franchise rather than start a new business? A) The franchiser requires $3,000 in start-up costs and a 5 percent monthly royalty fee. B) The franchiser tightly controls and monitors the service protocol. C) The franchisee is responsible for developing all marketing materials. D) The company has only recently become a franchise. E) There are several other franchises with the same company in the area. Answer: A Explanation: A) The start-up cost for most franchises is around $50,000 and the royalty fee is usually between 6 and 10 percent. The substantially lower costs of $3,000 and 5 percent would be a compelling reason to take on a franchise if it also has acceptable growth potential. Page Ref: 66 Difficulty: Difficult AACSB: Reflective thinking skills Objective: 3.3 Learning Outcome: Explain the entrepreneurship process Skill: Critical Thinking 83) Approximately what percent of all U.S. businesses employ 20 or fewer people? A) 86 percent B) 66 percent C) 46 percent D) 26 percent E) 16 percent Answer: A Explanation: A) According to 2006 census data, 85.95 percent of all businesses employ 20 or fewer people; this illustrates how important small business is to the U.S. economy. Page Ref: 60 Difficulty: Difficult Objective: 3.4 Learning Outcome: Describe the role of small businesses in the economy Skill: Concept 84) Each year, between 600,000 and 650,000 new businesses are launched in the United States. Approximately how many businesses fail each year? A) 50,000 to 100,000 B) 100,000 to 250,000 C) 250,000 to 500,000 D) 500,000 to 700,000 E) 700,000 to 1 million Answer: D Explanation: D) For example, in 2009, 627,200 new firms started and another 595,600 closed down. Page Ref: 68 Difficulty: Moderate Objective: 3.4 Learning Outcome: Explain the entrepreneurship process Skill: Concept 85) By what percent has the number of African American-owned businesses increased over the past five-year period? A) 18 percent B) 28 percent C) 48 percent D) 68 percent E) 78 percent Answer: C Explanation: C) The number of businesses owned by African Americans increased by 48 percent during the most recent five-year period for which data are available and now totals 2 million. Page Ref: 69 Difficulty: Moderate AACSB: Multicultural and diversity understanding Objective: 3.4 Learning Outcome: Describe the role of small businesses in the economy Skill: Concept 86) Which figures reflect the current size and the growth in Hispanic-owned businesses for the last five years? A) 2.25 million and 31 percent B) 2.75 million and 41 percent C) 3.15 million and 71 percent D) 3.55 million and 81 percent E) 4.25 million and 85 percent Answer: A Explanation: A) Figures of 31 percent totaling about 2.25 million businesses reflect strong growth for this group. Page Ref: 69 Difficulty: Difficult AACSB: Multicultural and diversity understanding Objective: 3.4 Learning Outcome: Describe the role of small businesses in the economy Skill: Application 87) What is the main reason why women start new businesses? A) need to earn more money B) preference for working alone C) opportunities for family members D) control in organizing their time E) desire to employ other women Answer: D Explanation: D) Some 46 percent of woman cite being able to gain control over their schedule as the reason why they are attracted to starting their own businesses. Page Ref: 69 Difficulty: Difficult AACSB: Multicultural and diversity understanding Objective: 3.4 Learning Outcome: Describe the role of small businesses in the economy Skill: Concept 88) What is the most effective way for an entrepreneur to become a competent manager? A) working alone B) relying on luck C) getting a college degree D) being open to opportunities E) working for a successful company first Answer: E Explanation: E) Few successful business owners succeed alone or straight out of college. Most spend time in successful companies or partner with others to bring expertise to a new business. Page Ref: 71 Difficulty: Moderate Objective: 3.3 Learning Outcome: Explain the entrepreneurship process Skill: Concept 89) Lucy has organized her craft shop as a sole proprietorship. Her sister has warned Lucy about the legal principle holding her responsible for paying off all of the debts of the business. What is the name of this principle? A) unlimited liability B) limited liability C) privity D) unrestricted debt E) accountability Answer: A Explanation: A) Sole proprietors face unlimited liability, meaning that they are liable for paying all debts incurred by the business. Page Ref: 72 Difficulty: Difficult Objective: 3.5 Learning Outcome: Describe the role of small businesses in the economy Skill: Application 90) Shirley is opening a flower shop and has decided to operate as a sole proprietorship. Which of the following is NOT an advantage Shirley will have in operating her business? A) freedom B) simplicity of operation C) limited liability D) low start-up costs E) choice of location Answer: C Explanation: C) A major drawback for sole proprietors is unlimited liability, whereby a sole proprietor is personally liable for all debts incurred by the business. Page Ref: 73 Difficulty: Moderate Objective: 3.5 Learning Outcome: Describe the role of small businesses in the economy Skill: Application 91) Kent and Patrick have decided to start a landscaping business. One of the first things that they do is sign a document that describes the investments and responsibilities of the partners and makes provisions for the allocation of profits and dissolution of the business. What is the name of this document? A) articles of incorporation B) partnership charter C) partnership agreement D) corporate charter E) financial proposal Answer: C Explanation: C) The partnership agreement is strictly a private document; no laws require partners to file agreements with any government agency. Page Ref: 73 Difficulty: Moderate AACSB: Reflective thinking skills Objective: 3.5 Learning Outcome: Describe the role of small businesses in the economy Skill: Application 92) Today, what percent of all businesses in the United States are registered as sole proprietors? A) 94 percent B) 72 percent C) 54 percent D) 28 percent E) 10 percent Answer: B Explanation: B) However, sole proprietors account for only 5 percent of total revenues. Page Ref: 75 Difficulty: Moderate Objective: 3.5 Learning Outcome: Describe the role of small businesses in the economy Skill: Concept 93) Why are lenders most willing to loan to corporations? A) Continuity and legal status are assured. B) Links with owners and founders are established. C) Professional management teams can be changed. D) Commitment to expansion is readily available. E) Brand and marketing strategies can be diversified. Answer: A Explanation: A) The biggest advantage of being a corporation is limited liability. Another advantage is continuity, as a corporation can, in theory, continue forever. Page Ref: 75 Difficulty: Difficult Objective: 3.5 Learning Outcome: Describe the role of small businesses in the economy Skill: Concept 94) American Business Machines is organized as a corporation. Its income will be taxed at the corporate level and also on shareholder returns as dividends. What is this situation known as? A) dual costs B) double taxation C) regulatory costs D) double ownership E) financial balancing Answer: B Explanation: B) The biggest disadvantage of incorporation is double taxation. First, a corporation pays income taxes on company profits; in addition, stockholders then pay taxes on income returned by their investments in the corporation. Page Ref: 76 Difficulty: Moderate Objective: 3.6 Learning Outcome: Describe the role of small businesses in the economy Skill: Application Chapter 4 The Global Context of Business 31) What are products that are created abroad and then transported and sold domestically? A) dumped goods B) exports C) imports D) buy backs E) domestic goods Answer: C Explanation: C) Imports are goods that are brought in from other countries. Page Ref: 89 Difficulty: Easy Objective: 4.1 Learning Outcome: Compare and contrast different economic systems Skill: Concept 32) What are products that are created domestically and transported for sale abroad? A) dumped goods B) exports C) imports D) buy backs E) domestic goods Answer: B Explanation: B) Exports are goods that are made in a country and sold in another country. Page Ref: 89 Difficulty: Easy Objective: 4.1 Learning Outcome: Compare and contrast different economic systems Skill: Concept 33) Which country is the world's largest marketplace and most stable economy? A) the United States B) Canada C) Germany D) Japan E) China Answer: A Explanation: A) The United States is the dominant trading partner in North America. Page Ref: 90 Difficulty: Easy AACSB: Dynamics of the global economy Objective: 4.1 Learning Outcome: Compare and contrast different economic systems Skill: Application 34) Which country dominates the Pacific Asia region economically? A) South Korea B) China C) Indonesia D) Japan E) Taiwan Answer: D Explanation: D) Japan's large automakers and industry help it dominate the Pacific Asia region. Page Ref: 91 Difficulty: Easy AACSB: Dynamics of the global economy Objective: 4.1 Learning Outcome: Compare and contrast different economic systems Skill: Application 35) Which organization most likely has the greatest impact on Asian countries? A) ASEAN B) UAS C) EU D) NAFTA E) ECOWAS Answer: A Explanation: A) The Association of South East Asian Nations is the economic association of nations in that region. Page Ref: 92 Difficulty: Easy AACSB: Dynamics of the global economy Objective: 4.1 Learning Outcome: Compare and contrast different economic systems Skill: Application 36) Which of the following is the name for the economic value of all of the products that a country exports minus the economic value of its imports? A) balance of foreign competition B) balance of domestic competition C) balance of trade D) balance of payments E) balance of supply and demand Answer: C Explanation: C) This is the definition of balance of trade. Page Ref: 93 Difficulty: Easy AACSB: Dynamics of the global economy Objective: 4.2 Learning Outcome: Explain how economic performance is monitored Skill: Concept 37) Which of the following refers to the situation when a country's imports exceed its exports? A) balance of payments B) balance of trade C) trade deficit D) trade surplus E) trade advantage Answer: C Explanation: C) A country operates a trade deficit if it imports more goods than it exports. Page Ref: 94 Difficulty: Easy AACSB: Dynamics of the global economy Objective: 4.2 Learning Outcome: Explain how economic performance is monitored Skill: Concept 38) Which of the following is the name for the overall flow of money into or out of a country? A) exchange rate B) trade rate C) balance of exchange D) balance of payments E) balance of trade Answer: D Explanation: D) The balance of payments determines whether a country is running a trade deficit or surplus. Page Ref: 94 Difficulty: Easy AACSB: Dynamics of the global economy Objective: 4.2 Learning Outcome: Explain how economic performance is monitored Skill: Concept 39) With what type of exchange rate does the value of a country's currency remain constant relative to that of another country? A) floating B) indexed C) fixed D) adjusted E) deflated Answer: C Explanation: C) A fixed exchange rate indexes a country's currency directly to that of another country. Page Ref: 96 Difficulty: Easy AACSB: Dynamics of the global economy Objective: 4.2 Learning Outcome: Explain how economic performance is monitored Skill: Concept 40) When the value of one country's currency relative to that of another varies with market conditions, which of the following exists? A) floating exchange rate B) fixed exchange rate C) indexed exchange rate D) flat exchange rate E) balanced exchange rate Answer: A Explanation: A) A floating exchange rate is one that varies as a function of market conditions among other countries. Page Ref: 96 Difficulty: Easy AACSB: Dynamics of the global economy Objective: 4.2 Learning Outcome: Explain how economic performance is monitored Skill: Concept 41) Which of the following organizations is a firm that makes products in one country and then distributes and sells them in others? A) importer B) exporter C) multinational firm D) international firm E) direct foreign investor Answer: B Explanation: B) An exporter sells and ships its products to other countries, who import them. Page Ref: 100 Difficulty: Easy AACSB: Dynamics of the global economy Objective: 4.3 Learning Outcome: Discuss the factors that influence decisions about organizational structure Skill: Concept 42) Which of the following buys products in foreign markets and then sells them for resale in its home country? A) importer B) exporter C) multinational firm D) international firm E) direct foreign investor Answer: A Explanation: A) An importer buys products from companies in other countries to sell in the importer's country. Page Ref: 100 Difficulty: Easy AACSB: Dynamics of the global economy Objective: 4.3 Learning Outcome: Discuss the factors that influence decisions about organizational structure Skill: Concept 43) ABC Company in the United States has given exclusive rights to XYZ Company in Switzerland to manufacture its products. What is this is an example of? A) a foreign direct investment B) a licensing arrangement C) a branch office setup D) a strategic alliance E) an import-export relationship Answer: B Explanation: B) A licensing arrangement is one in which a company gives rights to a foreign company to manufacture its products. Page Ref: 102 Difficulty: Easy AACSB: Dynamics of the global economy Objective: 4.3 Learning Outcome: Discuss the factors that influence decisions about organizational structure Skill: Application 44) The ABC Paint Company in the United States has joined forces with the 1-2-3 Plastics Company in Italy to create a nonpeeling, plastics-based paint. Both companies intend to share the research costs and any profits. What is this an example of? A) a licensing arrangement B) a branch office setup C) a strategic alliance D) a foreign direct investment E) an import-export relationship Answer: C Explanation: C) In a strategic alliance, different companies combine resources in an endeavor that can be accomplished more efficiently with them both. Page Ref: 102 Difficulty: Easy AACSB: Dynamics of the global economy Objective: 4.3 Learning Outcome: Discuss the factors that influence decisions about organizational structure Skill: Application 45) What is it called when a company sends some of its own managers overseas to conduct business in a local office? A) developing a strategic alliance B) outsourcing C) licensing products D) investing directly in a foreign venture E) opening a branch office Answer: E Explanation: E) A company doing business overseas will often open a branch office in the host country out of which to do business. Page Ref: 102 Difficulty: Easy AACSB: Dynamics of the global economy Objective: 4.3 Learning Outcome: Discuss the factors that influence decisions about organizational structure Skill: Concept 46) Which of the following denotes the international organization strategy of finding a partner in the country in which a company would like to conduct business? A) opening a branch office B) developing a strategic alliance C) forming an independent agency D) outsourcing E) investing directly in a foreign venture Answer: B Explanation: B) In a strategic alliance, different companies combine resources in an endeavor that can be accomplished more efficiently with them both. Page Ref: 102 Difficulty: Easy AACSB: Dynamics of the global economy Objective: 4.3 Learning Outcome: Discuss the factors that influence decisions about organizational structure Skill: Concept 47) Ford, an American car company, purchased Volvo, which is based in Sweden. What is this an example of? A) a multinational firm B) a independent agency C) a licensing arrangement D) a foreign direct investment E) a strategic alliance Answer: D Explanation: D) In a foreign direct investment, a company in another country invests, by partial or full ownership, in a company in the host country. Page Ref: 103 Difficulty: Easy AACSB: Reflective thinking skills Objective: 4.3 Learning Outcome: Discuss the factors that influence decisions about organizational structure Skill: Application 48) Which of the following denotes a government order forbidding exportation and/or importation of a particular product from a particular country? A) tariff B) embargo C) subsidy D) local content law E) business practice law Answer: B Explanation: B) An embargo blocks importation or exportation of a particular product. Page Ref: 104 Difficulty: Easy AACSB: Dynamics of the global economy Objective: 4.4 Learning Outcome: Explain the benefits and challenges of engaging in international business Skill: Concept 49) Which of the following is a government payment to help a domestic business compete with foreign firms? A) revenue tariff B) protectionist tariff C) import buy-back D) export rebate E) subsidy Answer: E Explanation: E) Subsidies are government aid to an industry to help it keep prices competitive. Page Ref: 104 Difficulty: Easy AACSB: Dynamics of the global economy Objective: 4.4 Learning Outcome: Explain the benefits and challenges of engaging in international business Skill: Concept 50) Which of the following denotes the practice of protecting domestic business at the expense of free market competition? A) fair trade B) balanced trade C) free trade D) market liberalism E) protectionism Answer: E Explanation: E) Protectionism is a policy of discouraging imports from other countries to protect domestic industries from foreign competition. Page Ref: 104-105 Difficulty: Easy AACSB: Dynamics of the global economy Objective: 4.4 Learning Outcome: Explain the benefits and challenges of engaging in international business Skill: Concept 51) Many governments require that products sold in their particular country be at least partly made there. This policy involves what type of laws? A) quota laws B) local content laws C) business practice laws D) free trade laws E) subsidy laws Answer: B Explanation: B) Local content laws support local industries by requiring that some products be at least partly made there. Page Ref: 105 Difficulty: Easy AACSB: Dynamics of the global economy Objective: 4.4 Learning Outcome: Explain the benefits and challenges of engaging in international business Skill: Concept 52) Which of the following refers to associations of producers that control supply and prices? A) independent agencies B) cartels C) trade alliances D) multinational firms E) trade monopolies Answer: B Explanation: B) A cartel is a group of companies that coordinate their production so as to maintain collective control over prices. Page Ref: 105 Difficulty: Easy AACSB: Dynamics of the global economy Objective: 4.4 Learning Outcome: Explain the benefits and challenges of engaging in international business Skill: Concept 53) Which of the following refers to the practice of selling a product abroad for less than the cost of production? A) exporting B) importing C) dumping D) safeguarding E) offshoring Answer: C Explanation: C) Dumping hurts companies where it occurs because local companies cannot compete with below-cost prices. Page Ref: 105 Difficulty: Easy AACSB: Dynamics of the global economy Objective: 4.4 Learning Outcome: Explain the benefits and challenges of engaging in international business Skill: Concept 54) Which organization created the classification of countries based on per capita income? A) the World Bank B) the World Trade Organization C) the International Monetary Fund D) the European Union E) the United Nations Economic and Social Council Answer: A Explanation: A) The World Bank, an agency of the United Nations, uses per capita income to make distinctions among countries. Page Ref: 90 Difficulty: Moderate AACSB: Dynamics of the global economy Objective: 4.1 Learning Outcome: Compare and contrast different economic systems Skill: Concept 55) Which of the following nations is NOT contained within a major world economic marketplace? A) Canada B) Greece C) Taiwan D) Singapore E) Honduras Answer: E Explanation: E) Major marketplaces include North America, Europe, and Pacific Asia. Page Ref: 90-91 Difficulty: Moderate AACSB: Dynamics of the global economy Objective: 4.1 Learning Outcome: Compare and contrast different economic systems Skill: Application 56) Which of the following countries is the world's third largest economy, behind that of the United States and Japan? A) China B) India C) Brazil D) Germany E) Russia Answer: A Explanation: A) China is the world's most densely populated country and has emerged as the world's third-largest economy. Page Ref: 91 Difficulty: Moderate AACSB: Dynamics of the global economy Objective: 4.1 Learning Outcome: Compare and contrast different economic systems Skill: Concept 57) Under which of the following treaties are Canada, the United States, and Mexico gradually eliminating tariffs and all other trade barriers? A) the EURO Agreement B) the Pan-American Agreement C) the North American Free Trade Agreement D) the General Agreement on Tariffs and Trade E) the G-3 Free Trade Agreement Answer: C Explanation: C) Some barriers came down in 1994; others were removed in 1999, 2004, and 2009. Page Ref: 91 Difficulty: Moderate AACSB: Dynamics of the global economy Objective: 4.1 Learning Outcome: Compare and contrast different economic systems Skill: Concept 58) What organization was founded in Pacific Asia in 1967 for economic, political, social, and cultural cooperation? A) SEATO B) ASEAN C) CENTO D) OAS E) WTO Answer: B Explanation: B) Today, the Association of Southeast Asian Nations (ASEAN) encompasses a population of 500 million and a GDP of $800 billion. Page Ref: 92 Difficulty: Moderate AACSB: Dynamics of the global economy Objective: 4.1 Learning Outcome: Compare and contrast different economic systems Skill: Concept 59) Which of the following treaties sought to eliminate trade barriers such as tariffs and quotas? A) United Nations Development Assistant Plan B) Fair Labor Standards Act C) General Agreement on Tariffs and Trade D) General Agreement on Trade in Services E) European Union Stability and Growth Pact Answer: C Explanation: C) GATT's purpose is to reduce or eliminate trade barriers by encouraging nations to protect domestic industries within agreed-upon limits and to engage in multilateral negotiations. Page Ref: 92 Difficulty: Moderate AACSB: Dynamics of the global economy Objective: 4.1 Learning Outcome: Compare and contrast different economic systems Skill: Concept 60) Which of the following best describes a positive trade balance? A) the economic condition in which a country's exports exceed its imports B) the economic condition in which a country's imports exceed its exports C) the economic condition in which a country's inflow of money exceeds its outflow D) the economic condition in which a country's outflow of money exceeds its inflow E) the economic condition in which a country's natural resources exceed its human resources Answer: A Explanation: A) This is also known as a trade surplus. Page Ref: 93 Difficulty: Moderate AACSB: Dynamics of the global economy Objective: 4.2 Learning Outcome: Explain how economic performance is monitored Skill: Concept 61) The value of the U.S. dollar relative to the value of the British pound fluctuates with market conditions. What is this type of exchange rate? A) floating B) fixed C) indexed D) adjusted E) monitored Answer: A Explanation: A) Floating exchange rates fluctuate according to changes in the market. Page Ref: 96 Difficulty: Moderate AACSB: Dynamics of the global economy Objective: 4.2 Learning Outcome: Explain how economic performance is monitored Skill: Application 62) Country Z can produce pig iron more efficiently than any other goods. Which type of advantage in pig iron does Country Z have? A) absolute advantage B) national competitive advantage C) comparative advantage D) monopolistic advantage E) resource advantage Answer: C Explanation: C) When a country can produce a good more efficiently than it can produce any other good, that country is said to have a comparative advantage in that efficiently produced good. Page Ref: 97-98 Difficulty: Moderate AACSB: Reflective thinking skills Objective: 4.2 Learning Outcome: Explain how economic performance is monitored Skill: Application 63) Which of the following exists when a country can produce something more cheaply and/or of higher quality than any other country can? A) monopolistic advantage B) national competitive advantage C) resource advantage D) absolute advantage E) comparative advantage Answer: D Explanation: D) An absolute advantage, though rare, exists when a country can produce something more cheaply and/or of higher quality than any other country can. Page Ref: 97 Difficulty: Moderate AACSB: Dynamics of the global economy Objective: 4.2 Learning Outcome: Explain how economic performance is monitored Skill: Concept 64) National competitive advantage derives from four conditions, one of which is factor conditions. What do factor conditions include? A) customers, suppliers, and labor B) government quotas and subsidies C) product quality, productivity, and profits D) labor, capital, entrepreneurs, and physical resources E) strategies, structures, and rivalries Answer: D Explanation: D) Factor conditions, or factors of production, include labor, capital, and materials. Page Ref: 98 Difficulty: Moderate AACSB: Dynamics of the global economy Objective: 4.2 Learning Outcome: Explain how economic performance is monitored Skill: Concept 65) Which of the following types of firms design, produce, and manufacture products in many nations? A) multinational firms B) importing firms C) domestic firms D) international firms E) exporting firms Answer: A Explanation: A) Multinational firms design, produce and market products in many countries; in addition, headquarters locations are almost irrelevant. Page Ref: 100 Difficulty: Moderate AACSB: Dynamics of the global economy Objective: 4.3 Learning Outcome: Discuss the factors that influence decisions about organizational structure Skill: Concept 66) Which of the following is NOT considered an international organizational structure? A) independent agents B) licensing arrangements C) foreign direct investments D) branch office E) exporters Answer: E Explanation: E) International organizational structures include independent agents, licensing arrangements, branch offices, strategic alliances, and foreign direct investment. Page Ref: 100 Difficulty: Moderate AACSB: Dynamics of the global economy Objective: 4.3 Learning Outcome: Discuss the factors that influence decisions about organizational structure Skill: Concept 67) Which statement is true regarding independent agents? A) They represent one firm at a time. B) They usually specialize in a particular product. C) They act as sales representatives. D) They are usually based at domestic offices. E) They do not monitor or address customer satisfaction. Answer: C Explanation: C) Independent agents act as sales representatives, representing many exporters in foreign markets. Page Ref: 101 Difficulty: Moderate AACSB: Dynamics of the global economy Objective: 4.3 Learning Outcome: Discuss the factors that influence decisions about organizational structure Skill: Concept 68) To avoid transportation costs and tariffs, Apics Enterprises gave exclusive rights to Svenson AB in Finland to produce its product. What is this an example of? A) licensing arrangement B) strategic alliance C) foreign direct investment D) independent agent E) branch office Answer: A Explanation: A) In licensing arrangements, firms give foreign companies exclusive rights to manufacture or market their products in that market. Page Ref: 102 Difficulty: Moderate AACSB: Dynamics of the global economy Objective: 4.3 Learning Outcome: Discuss the factors that influence decisions about organizational structure Skill: Application 69) Which of the following types of payments are usually received by an exporter as an ongoing payment calculated as a percentage of the license holder's sales? A) flat fees B) royalties C) agent fees D) subsidies E) direct investments Answer: B Explanation: B) The exporter receives a fee plus ongoing payments, or royalties, calculated as a percentage of the license holder's sales. Page Ref: 102 Difficulty: Moderate AACSB: Dynamics of the global economy Objective: 4.3 Learning Outcome: Discuss the factors that influence decisions about organizational structure Skill: Concept 70) In which international organization structure does a firm have greatest control over foreign activities? A) exporting arrangements B) strategic alliances C) branch offices D) licensing arrangements E) independent agents Answer: B Explanation: B) Strategic alliances give firms greater control over foreign activities than agents and licensees. Page Ref: 102 Difficulty: Moderate AACSB: Dynamics of the global economy Objective: 4.3 Learning Outcome: Discuss the factors that influence decisions about organizational structure Skill: Concept 71) U.S. imports of Canadian softwood timber are limited to 14.7 billion board feet per year. What is this practice called? A) tariff B) embargo C) subsidy D) dumping E) quota Answer: E Explanation: E) A quota restricts the number or amount of certain types of products that can be imported. Page Ref: 104 Difficulty: Moderate AACSB: Dynamics of the global economy Objective: 4.4 Learning Outcome: Explain the benefits and challenges of engaging in international business Skill: Application 72) The U.S. government forbids the importation of cigars from Cuba. What is this practice called? A) embargo B) tariff C) subsidy D) business practice law E) local content law Answer: A Explanation: A) An embargo is a government order forbidding exportation/importation of certain products. Page Ref: 104 Difficulty: Moderate AACSB: Reflective thinking skills Objective: 4.4 Learning Outcome: Explain the benefits and challenges of engaging in international business Skill: Application 73) Which of the following refers to tariffs that are imposed strictly to raise money for the government? A) revenue tariffs B) protectionist tariffs C) quota tariffs D) subsidy tariffs E) labor tariffs Answer: A Explanation: A) Revenue tariffs raise money for the government. Page Ref: 104 Difficulty: Moderate AACSB: Dynamics of the global economy Objective: 4.4 Learning Outcome: Explain the benefits and challenges of engaging in international business Skill: Concept 74) Which of the following types of tariff is meant to discourage the import of particular products? A) revenue tariff B) quota tariff C) labor tariff D) subsidy tariff E) protectionist tariff Answer: E Explanation: E) Protectionist tariffs discourage particular imports. Page Ref: 104 Difficulty: Moderate AACSB: Dynamics of the global economy Objective: 4.4 Learning Outcome: Explain the benefits and challenges of engaging in international business Skill: Concept 75) Suppose that the United States requires that products sold here be at least 51 percent made here. Which of the following denotes this type of policy? A) business practice law B) deregulation law C) business practice law D) strategic alliance E) local content law Answer: E Explanation: E) Local content laws are requirements that products sold in a country be at least partly made there. Page Ref: 105 Difficulty: Moderate AACSB: Reflective thinking skills Objective: 4.4 Learning Outcome: Explain the benefits and challenges of engaging in international business Skill: Application 76) In some South American countries, it is sometimes legal to bribe other businesses and government officials, while this practice is illegal in the United States. Which of the following refers to this type of policy? A) protectionism B) business practice laws C) free trade agreements D) fair trade agreements E) local content laws Answer: B Explanation: B) Every country has its own business practice laws by which they govern business practices. Page Ref: 105 Difficulty: Moderate AACSB: Reflective thinking skills Objective: 4.4 Learning Outcome: Explain the benefits and challenges of engaging in international business Skill: Application 77) Which of the following goals is a primary purpose of tariffs on imports? A) maintain domestic competitiveness B) decrease exports C) increase imports D) subsidize domestic industry E) promote free trade Answer: A Explanation: A) Most tariffs are meant to discourage particular imports in order that domestic businesses may remain competitive. Page Ref: 104 Difficulty: Moderate AACSB: Dynamics of the global economy Objective: 4.4 Learning Outcome: Explain the benefits and challenges of engaging in international business Skill: Concept 78) China and India require that when foreign firms enter into joint ventures with local firms, the local partners must have the controlling ownership stake. What is this an example of? A) local content laws B) business practice laws C) fair trade agreements D) quotas E) subsidies Answer: A Explanation: A) Local content laws are requirements that products sold in a country be at least partly made there. Firms seeking to do business in a country must either invest there directly or take on a domestic partner. Page Ref: 105 Difficulty: Moderate AACSB: Reflective thinking skills Objective: 4.4 Learning Outcome: Explain the benefits and challenges of engaging in international business Skill: Application 79) In Germany, Wal-Mart has been required to buy existing retailers rather than open brand-new stores. What does this illustrate? A) local content laws B) fair trade agreements C) free trade agreements D) licensing arrangements E) business practice laws Answer: E Explanation: E) Host countries govern business practices within their jurisdictions through various business practice laws. Page Ref: 105 Difficulty: Moderate AACSB: Reflective thinking skills Objective: 4.4 Learning Outcome: Explain the benefits and challenges of engaging in international business Skill: Application 80) Which of the following is illegal in the United States? A) dumping but not cartels B) cartels but not dumping C) only certain kinds of dumping and cartels D) neither dumping nor cartels E) both dumping and cartels Answer: E Explanation: E) Dumping and cartels are both illegal in the United States. Page Ref: 105 Difficulty: Moderate AACSB: Dynamics of the global economy Objective: 4.4 Learning Outcome: Explain the benefits and challenges of engaging in international business Skill: Application 81) Which of the following resulted from NAFTA? A) Companies from other countries, such as Japan, have made fewer business investments in North America. B) U.S. exports to Mexico have decreased. C) U.S. exports to Mexico have increased. D) More jobs have been created than what was originally predicted. E) More U.S. jobs were eliminated than what was originally predicted. Answer: C Explanation: C) Many positive predictions regarding what would result from NAFTA have materialized, including the increase in U.S. exports to Mexico. Page Ref: 91 Difficulty: Difficult AACSB: Dynamics of the global economy Objective: 4.1 Learning Outcome: Compare and contrast different economic systems Skill: Synthesis 82) Bridget wants to buy some English tea priced at 10 pounds from an online seller. The current exchange rate is 1.5 dollars to the pound. How much she will pay for the tea (excluding any upfront exchange fees)? A) $15.00 B) $12.50 C) $11.50 D) $8.50 E) $6.67 Answer: A Explanation: A) $1.50 × 10 pounds = $15.00. Page Ref: 96 Difficulty: Moderate AACSB: Analytic skills Objective: 4.2 Learning Outcome: Explain the benefits and challenges of engaging in international business Skill: Application 83) What typically happens to a country's balance of trade as the value of its currency falls? A) It improves. B) It declines. C) It remains the same. D) It fluctuates drastically. E) It fluctuates moderately. Answer: A Explanation: A) As the value of its currency falls relative to that of other countries, other countries are more likely to buy goods from that country. Page Ref: 97 Difficulty: Difficult AACSB: Dynamics of the global economy Objective: 4.2 Learning Outcome: Explain how economic performance is monitored Skill: Concept 84) What is a difference between an international firm and a multinational firm? A) Multinational firms design, produce, and market products in many nations, whereas international firms are based primarily in one nation. B) International firms design, produce, and market products in many nations, whereas multinational firms are based primarily in one nation. C) International and multinational firms are the same thing. D) Multinational firms are concerned primarily with foreign markets, whereas international firms are concerned primarily with the domestic market. E) International firms are concerned primarily with foreign markets, whereas multinational firms are concerned primarily with the domestic market. Answer: A Explanation: A) International firms conduct much of their business abroad and may maintain overseas manufacturing facilities; multinational firms design, produce, and market products in many nations. Page Ref: 100 Difficulty: Difficult AACSB: Dynamics of the global economy Objective: 4.3 Learning Outcome: Explain the benefits and challenges of engaging in international business Skill: Concept 85) Which of the following statements best describes the effects of subsidies? A) A subsidy essentially lowers the prices of foreign goods rather than raising the prices of domestic goods. B) A subsidy essentially raises the prices of domestic goods rather than lowering the prices of foreign goods. C) A subsidy essentially lowers the prices of domestic goods rather than raising the prices of foreign goods. D) A subsidy essentially raises the prices of foreign goods while also raising the prices of domestic goods. E) A subsidy essentially lowers the prices of foreign goods while also lowering the prices of domestic goods. Answer: C Explanation: C) A subsidy is a government payment to help a domestic business compete with foreign firms; the subsidy lowers the prices of domestic goods rather than raising the prices of foreign goods. Page Ref: 104 Difficulty: Difficult AACSB: Dynamics of the global economy Objective: 4.4 Learning Outcome: Explain the benefits and challenges of engaging in international business Skill: Concept Chapter 5 Business Management 24) Which business constituents analyze their competitive environments and plan, organize, direct, and control the operations of their organizations? A) stockholders B) managers C) regulators D) agents E) customers Answer: B Explanation: B) Management itself is the process of planning, organizing, leading, and controlling an organization's financial, physical, human, and information resources to achieve its goals. Managers oversee the use of all these resources in their respective firms. Page Ref: 114 Difficulty: Easy Objective: 5.1 Learning Outcome: Describe the skills and functions of management Skill: Concept 25) Which aspect of the management process involves determining what the organization needs to do and how best to get it done? A) organizing B) planning C) controlling D) processing E) monitoring Answer: B Explanation: B) Planning is determining what the organization needs to do and how best to get it done. Page Ref: 114 Difficulty: Easy Objective: 5.1 Learning Outcome: Describe the skills and functions of management Skill: Concept 26) Which type of manager is responsible for implementing the strategies, policies, and decisions made by top managers? A) first-line manager B) middle manager C) supervisory manager D) office manager E) financial managers Answer: B Explanation: B) Just below top managers are middle managers, who implement strategies, policies, and decisions made by top managers. Page Ref: 118 Difficulty: Easy Objective: 5.2 Learning Outcome: Describe the skills and functions of management Skill: Concept 27) Which type of manager spends most of their time working with and supervising the employees who report to them? A) first-line managers B) information managers C) top managers D) chief executive officers E) operations managers Answer: A Explanation: A) Supervisors and office managers are the first-line managers who work with and supervise the employees who report to them. Page Ref: 118 Difficulty: Easy Objective: 5.2 Learning Outcome: Describe the skills and functions of management Skill: Concept 28) What type of skills are needed to understand and work well with people in the workplace? A) human relations skills B) technical skills C) conceptual skills D) motor skills E) decision-making skills Answer: A Explanation: A) Human relations skills are skills in understanding and getting along with other people. Page Ref: 119 Difficulty: Easy Objective: 5.3 Learning Outcome: Describe the skills and functions of management Skill: Concept 29) Which of the following refers to a person's ability to think in the abstract, to diagnose and analyze different situations, and to see beyond the present situation? A) human relations skills B) decision-making skills C) conceptual skills D) production skills E) technical skills Answer: C Explanation: C) Conceptual skills refer to the ability to think abstractly as well as diagnose and analyze different situations. Page Ref: 120 Difficulty: Easy Objective: 5.3 Learning Outcome: Describe the skills and functions of management Skill: Concept 30) What type of skills requires defining problems and selecting the best course of action to take? A) human relations skills B) production skills C) conceptual skills D) decision-making skills E) technical skills Answer: D Explanation: D) Decision-making skills include the ability to define problems and select the best courses of action. Page Ref: 120 Difficulty: Easy Objective: 5.3 Learning Outcome: Describe the skills and functions of management Skill: Concept 31) What identifies performance targets used by organizations and their managers to measure success or failure at every level? A) tactics B) goals C) strategies D) mission statements E) promotions Answer: B Explanation: B) Goals are objectives that a business hopes (and plans) to achieve. Page Ref: 122 Difficulty: Easy Objective: 5.4 Learning Outcome: Explain how economic performance is monitored Skill: Concept 32) If Kodak sets out to increase its share of the 35-mm film market by 10 percent during the next eight years, what kind of goal is the organization setting? A) a short-term goal B) an intermediate goal C) a tactical goal D) a long-term goal E) an organizational goal Answer: D Explanation: D) A long-term goal is one set for an extended time, typically five years or more into the future. Page Ref: 124 Difficulty: Easy AACSB: Reflective thinking skills Objective: 5.4 Learning Outcome: Explain how economic performance is monitored Skill: Application 33) When the finance department aims for a 3 percent increase in return on investment in three years, what type of goal is the organization setting? A) a long-term goal B) a tactical goal C) a short-term goal D) an intermediate goal E) an organizational goal Answer: D Explanation: D) An intermediate goal is one set for a period of one to five years into the future. Page Ref: 124 Difficulty: Easy AACSB: Reflective thinking skills Objective: 5.4 Learning Outcome: Explain how economic performance is monitored Skill: Application 34) What does e-partnering involve? A) buying shares of small companies that provide technology unavailable to the "purchasing" company B) buying shares of small companies that are similar to the "purchasing" firm C) linking up to the inventory records of another company D) sharing a Web server with another company E) advertising another company on the "purchasing" company's Web site Answer: A Explanation: A) E-partnering is buying shares of small companies that can provide technology that the buyer itself does not have. It is a corporate strategy for growth. Page Ref: 125 Difficulty: Easy Objective: 5.4 Learning Outcome: Describe the skills and functions of management Skill: Concept 35) Which kind of strategy takes place at the level of the product line and focuses on a firm's competitive position? A) functional B) business C) local D) fundamental E) environmental Answer: B Explanation: B) Business (or competitive) strategy takes place at the level of the business unit or product line and focuses on improving the company's competitive position. Page Ref: 125 Difficulty: Easy Objective: 5.4 Learning Outcome: Describe the skills and functions of management Skill: Concept 36) Which element of an organization is defined by shared experiences, stories, beliefs, and norms? A) its human resources B) its product philosophy C) its corporate culture D) its mission statement E) its organizational structure Answer: C Explanation: C) Every company has a unique identity, a corporate culture: the shared experiences, stories, beliefs, and norms that characterize an organization. This culture helps define the work and business climate that exists in an organization. Page Ref: 130 Difficulty: Easy Objective: 5.6 Learning Outcome: Discuss the factors that affect motivation and behavior in the workplace Skill: Concept 37) Which of the following is TRUE about corporate culture? A) It can direct employees' efforts toward goals. B) It is usually determined by outside forces. C) It cannot be changed. D) It is easy to change. E) It rarely affects newcomers in an organization. Answer: A Explanation: A) Every company has a unique identity, a corporate culture: the shared experiences, stories, beliefs, and norms that characterize an organization. This culture helps define the work and business climate that exists in an organization. Page Ref: 130 Difficulty: Easy Objective: 5.6 Learning Outcome: Discuss the factors that affect motivation and behavior in the workplace Skill: Concept 38) Which description defines the planning function of the management process? A) guiding and motivating employees to meet organizational objectives B) discussing how best to arrange an organization's resources into a coherent structure C) scanning the business environment for threats and opportunities D) determining what an organization needs to do and how best to get it done E) monitoring a firm's performance to make sure that it is meeting its goals Answer: D Explanation: D) Further, planning has three components. First, it begins when managers determine the firm's goals. Next, they develop a strategy for achieving those goals. Finally, they design tactical and operational plans for implementing the strategy. Page Ref: 114 Difficulty: Moderate Objective: 5.1 Learning Outcome: Describe the skills and functions of management Skill: Concept 39) Which description identifies the organizing function of the management process? A) guiding and motivating employees to meet organizational objectives B) determining how best to arrange an organization's resources into a coherent structure C) scanning the business environment for threats and opportunities D) monitoring an organization's performance to ensure that it is meeting its goals E) determining what an organization needs to do and how best to get it done Answer: B Explanation: B) This function is sometimes illustrated through the use of organization charts, which help everyone understand roles and reporting relationships. Page Ref: 115 Difficulty: Moderate Objective: 5.1 Learning Outcome: Describe the skills and functions of management Skill: Concept 40) Hewlett-Packard decides to move toward a more centralized structure. This is an example of which function in the management process? A) planning B) leading C) organizing D) controlling E) monitoring Answer: C Explanation: C) Organizing is determining the best way to arrange a business's resources and activities into a coherent structure. Page Ref: 115 Difficulty: Moderate AACSB: Reflective thinking skills Objective: 5.1 Learning Outcome: Discuss the factors that influence decisions about organizational structure Skill: Application 41) Which description identifies the controlling function of the management process? A) guiding and motivating employees to meet organizational objectives B) determining how to arrange a firm's resources into a coherent structure C) scanning the business environment for threats and opportunities D) monitoring a firm's performance to ensure that it is meeting its goals E) determining what an organization needs to do and how best to get it done Answer: D Explanation: D) The control process begins when managers establish performance standards; they then compare results with those identified standards. Page Ref: 116 Difficulty: Moderate Objective: 5.1 Learning Outcome: Explain how economic performance is monitored Skill: Concept 42) Comparing actual performance against standards is an example of which function of the management process? A) planning B) organizing C) leading D) controlling E) strategizing Answer: D Explanation: D) Controlling is the process of monitoring a firm's performance to make sure that it is meeting its goals. Page Ref: 116 Difficulty: Moderate Objective: 5.1 Learning Outcome: Explain how economic performance is monitored Skill: Concept 43) Steve motivates employees by rewarding them with additional vacation when standards are achieved. This is an example of which function in the management process? A) leading B) planning C) organizing D) marketing E) strategizing Answer: A Explanation: A) When leading, a manager works to guide and motivate employees to meet the firm's objectives. Page Ref: 117 Difficulty: Moderate AACSB: Reflective thinking skills Objective: 5.1 Learning Outcome: Describe the skills and functions of management Skill: Application 44) Which type of business constituent has titles such as President and CEO? A) first-line managers B) top managers C) middle managers D) power brokers E) stockholders Answer: B Explanation: B) These are titles given to top managers, who are responsible for the overall performance of the organization. Page Ref: 117 Difficulty: Moderate Objective: 5.2 Learning Outcome: Describe the skills and functions of management Skill: Concept 45) Which level of management is responsible for the overall performance and effectiveness of the firm? A) top managers B) first-line managers C) middle managers D) supervisory managers E) human resource managers Answer: A Explanation: A) Top managers set general policies, formulate strategies, approve all significant decisions, and represent the company in deals with other firms and with government bodies. Page Ref: 118 Difficulty: Moderate Objective: 5.2 Learning Outcome: Describe the skills and functions of management Skill: Concept 46) Which level of management sets general policies, formulates strategies, approves all significant decisions, and represents the company in dealings with other firms and with government bodies? A) operations managers B) production managers C) top managers D) middle managers E) first-line managers Answer: C Explanation: C) Top managers, through these types of activities, are responsible for the overall performance and effectiveness of the firm. Page Ref: 118 Difficulty: Moderate Objective: 5.2 Learning Outcome: Describe the skills and functions of management Skill: Concept 47) Titles such as plant manager, operations manager, and division manager designate which level of management? A) first-line managers B) middle managers C) top managers D) supervisors E) technology managers Answer: B Explanation: B) Middle managers are responsible for implementing the strategies and working toward the goals set by top managers. Page Ref: 119 Difficulty: Moderate Objective: 5.2 Learning Outcome: Describe the skills and functions of management Skill: Concept 48) The titles supervisor, office manager, and group leader are examples of which level of management? A) top management B) middle management C) first-line management D) operations management E) human resource management Answer: C Explanation: C) First-line managers spend most (but not all) of their time working with and supervising the employees who report to them. Page Ref: 119 Difficulty: Moderate Objective: 5.2 Learning Outcome: Describe the skills and functions of management Skill: Concept 49) Which of the following are responsible for production and quality control? A) information managers B) human resource managers C) first-line managers D) operations managers E) marketing managers Answer: D Explanation: D) The term operations refers to the systems by which a firm produces goods and services. Page Ref: 119 Difficulty: Moderate Objective: 5.2 Learning Outcome: Describe the skills and functions of management Skill: Concept 50) Which of the following managers are directly responsible for getting products from producers to consumers? A) human resource managers B) public relations managers C) information managers D) marketing managers E) financial managers Answer: D Explanation: D) Marketing encompasses the development, pricing, promotion, and distribution of goods and services. Page Ref: 119 Difficulty: Moderate Objective: 5.2 Learning Outcome: Describe the skills and functions of management Skill: Concept 51) Who is responsible for designing and implementing systems to gather, organize, and distribute facts and data essential for the running of the company? A) marketing managers B) information managers C) production managers D) human resource managers E) research and development managers Answer: B Explanation: B) Huge increases in both the sheer volume of information and the ability to manage it have led to the emergence of this important function. Page Ref: 119 Difficulty: Moderate Objective: 5.2 Learning Outcome: Describe the skills and functions of management Skill: Concept 52) Which of the following managers are responsible for planning and overseeing the firm's accounting functions? A) financial managers B) information managers C) production managers D) human resource managers E) marketing managers Answer: A Explanation: A) Levels of financial management include chief financial officer, division controller, and accounting supervisor. Page Ref: 119 Difficulty: Moderate Objective: 5.2 Learning Outcome: Describe the skills and functions of management Skill: Concept 53) An accountant's ability to audit a company's records is an example of what type of skills? A) motor skills B) technical skills C) human relations skills D) conceptual skills E) decision-making skills Answer: B Explanation: B) The skills needed to perform specialized tasks are called technical skills. Page Ref: 120 Difficulty: Moderate Objective: 5.3 Learning Outcome: Identify the basic concepts of and tools used for business accounting Skill: Concept 54) Camille Graham is a manager who works well with people and makes them feel excited about their work. What type of skills is she illustrating? A) human relations skills B) technical skills C) accounting skills D) conceptual skills E) marketing skills Answer: A Explanation: A) Human relations skills enable managers to understand and get along with other people. Page Ref: 120 Difficulty: Moderate Objective: 5.3 Learning Outcome: Discuss the factors that affect motivation and behavior in the workplace Skill: Application 55) How might an effective manager choose the best plan of action when making a decision? A) making a self-generated list of pros and cons and choosing based on the list B) using the results of a survey that was sent to key customers who will be affected C) sending a feedback form to customers after the plan has been executed D) creating only one plan with no alternatives so it has to work E) reviewing what other managers have done in the same situation and doing the same Answer: B Explanation: B) An effective manager will consider the effects on all stakeholders before taking a certain action. Page Ref: 120 Difficulty: Moderate AACSB: Analytic skills Objective: 5.3 Learning Outcome: Describe the skills and functions of management Skill: Synthesis 56) Which of the following managers depend most on conceptual skills? A) first-line managers B) top managers C) middle managers D) marketing managers E) financial managers Answer: B Explanation: B) Conceptual skills, closely associated with top managers, refer to a person's ability to think in the abstract, to diagnose and analyze different situations, and to see beyond the present situation. Page Ref: 120 Difficulty: Moderate Objective: 5.3 Learning Outcome: Describe the skills and functions of management Skill: Concept 57) What activity do company managers undertake in order to measure success or failure at every organizational level? A) planning tactics B) setting goals C) devising strategies D) auditing skills E) publicizing promotions Answer: B Explanation: B) The starting point in effective strategic management is setting goals—objectives that a business hopes and plans to achieve. Page Ref: 123 Difficulty: Moderate Objective: 5.4 Learning Outcome: Describe the skills and functions of management Skill: Concept 58) Which of the following best describes a mission? A) an objective that a business hopes and plans to achieve B) a broad set of organizational plans for implementing decisions made for achieving organizational goals C) the shared experiences, stories, beliefs, and norms that characterize an organization D) an organization's statement of how it will achieve its purpose in the environment in which it conducts business E) an organization's competitive standing in the marketplace Answer: D Explanation: D) That purpose can be profits, the discovery and transmission of new knowledge, market share, the enforcement of public policies, etc. Page Ref: 124 Difficulty: Moderate Objective: 5.4 Learning Outcome: Describe the skills and functions of management Skill: Concept 59) What do a company's vision and mission statement have in common? A) They are both used to generate profits. B) They are both directed toward employees. C) They are both used to keep management on track. D) They are both directed toward customers. E) They are both used to set profit objectives. Answer: C Explanation: C) A company's vision and mission statement are used to keep management moving toward defined goals. Page Ref: 124 Difficulty: Moderate AACSB: Analytic skills Objective: 5.4 Learning Outcome: Describe the skills and functions of management Skill: Synthesis 60) Sometimes a corporation buys and operates multiple businesses in compatible industries as part of its corporate strategy. This is an example of what type of strategy? A) customer-driven diversification B) unrelated diversification C) e-partnering D) strategic diversification E) complex collaborations Answer: D Explanation: D) When businesses a corporation operates are related to one another, the strategy is related diversification. If the businesses are not similar, the strategy is unrelated diversification. Page Ref: 125 Difficulty: Moderate Objective: 5.4 Learning Outcome: Describe the skills and functions of management Skill: Concept 61) Which of the following best describes business strategy? A) a strategy for determining the firm's overall attitude toward growth and the way it will manage its businesses or product lines B) a strategy at the business-unit or product-line level that focuses on improving a firm's competitive position C) a strategy by which managers in specific areas decide how best to achieve corporate goals through productivity D) a strategy that assists first-line managers in making day-to-day decisions about motivating employees E) a strategy that integrates an organization's marketing goals into a cohesive whole by focusing on the ideal product mix to achieve maximum profit potential Answer: B Explanation: B) Further, a strategy outlines how the business intends to meet its goals and includes the organization's responsiveness to new challenges and new needs. Page Ref: 125 Difficulty: Moderate Objective: 5.4 Learning Outcome: Describe the skills and functions of management Skill: Concept 62) Which of the following best describes functional strategy? A) a strategy for determining the firm's overall attitude toward growth and the way it will manage its businesses or product lines B) a strategy at the business-unit or product-line level that focuses on improving a firm's competitive position C) a strategy by which managers in specific areas decide how best to achieve corporate goals through productivity D) a strategy that assists first-line managers in making day-to-day decisions about motivating employees E) a strategy that integrates an organization's marketing goals into a cohesive whole by focusing on the ideal product mix to achieve maximum profit potential Answer: C Explanation: C) At the level of functional strategy, managers in specific areas such as marketing, finance, and operations decide how best to achieve corporate goals by performing their functional activities most effectively. Page Ref: 126 Difficulty: Moderate Objective: 5.4 Learning Outcome: Describe the skills and functions of management Skill: Concept 63) Which of the following is NOT a characteristic of a solid contingency plan? A) contact information for an on-site crisis manager B) information systems backup software C) an evacuation plan D) a prioritized list of critical operations E) a summary of common types of emergencies Answer: E Explanation: E) A summary of common types of emergencies would not be of much assistance in a crisis. However, a good contingency plan will provide all of the other information listed in anticipation of an emergency situation. Page Ref: 130 Difficulty: Moderate AACSB: Analytic skills Objective: 5.5 Learning Outcome: Describe the skills and functions of management Skill: Synthesis 64) Doreen Madison has been tracking product sales for McCoy Industries. She notices that sales of surgical supplies have fallen off in the past six months and she is considering strategies to reverse this trend. This is an example of what function of the management process? A) planning B) organizing C) leading D) controlling E) budgeting Answer: D Explanation: D) Controlling is the process of monitoring a firm's performance to make sure that it is meeting its goals. Page Ref: 116 Difficulty: Difficult AACSB: Reflective thinking skills Objective: 5.1 Learning Outcome: Describe the skills and functions of management Skill: Application 65) CFO and controller are titles associated with what type of manager? A) financial B) information C) human resource D) marketing E) operations Answer: A Explanation: A) Nearly every company has financial managers to plan and oversee its accounting functions and financial resources. Page Ref: 118 Difficulty: Difficult Objective: 5.2 Learning Outcome: Describe the skills and functions of management Skill: Concept 66) Terrell is being considered for promotion from a supervisory role to top management. He is unsure whether he is qualified for the position. He wants to ensure that he is an effective manager, and not simply having success in the workplace because of his multiple promotions in the last few years. Although he's never been a top manager, there are some things he does in his personal life other members of the management team think qualify him for the position. What personal activity does management most likely think is a good indicator of whether or not Terrell will be an effective top manager? A) He is part of a theatrical group that gives performances to hundreds of people, showing that he is not afraid to speak in front of a crowd. B) He juggles being a parent, working as a middle manager, and having personal hobbies, showing an ability to do well in all aspects of his life without going overboard in one or neglecting another. C) He has many siblings, showing that he knows how to share space with a large number of people. D) He quickly learns how to play new games by following the rules explained to him. E) He was a member of the debate team in high school and consequently is well-versed in being able to argue both sides of an issue. Answer: B Explanation: B) This scenario shows Terrell's personal skills of time-management and interpersonal skills will contribute to his being an effective manager. Page Ref: 118 Difficulty: Difficult AACSB: Reflective thinking skills Objective: 5.2 Learning Outcome: Describe the skills and functions of management Skill: Critical Thinking 67) Starbucks has established itself as the premier marketer of the finest coffee in the world while maintaining its uncompromising principles and continuing to grow. This statement illustrates what aspect of Starbucks' organization? A) its strategy B) its tactic C) its mission D) its diversity E) its profitability Answer: C Explanation: C) This is the mission statement for Starbucks. Page Ref: 124 Difficulty: Difficult Objective: 5.4 Learning Outcome: Describe the skills and functions of management Skill: Application 68) "Teach For America provides a critical source of well-trained teachers who are helping break the cycle of educational inequity. These teachers, called corps members, commit to teaching for two years in one of 39 urban and rural regions across the country, going above and beyond traditional expectations to help their students achieve at high levels. "Teach For America's 20,000 alumni are playing critical leadership roles in the effort to improve the quality of public education in low-income communities. Armed with the experience, conviction, and insight that come from leading children to fulfill their potential, our alumni are working from all sectors to shape our schools, policies, and investments in low-income communities." What objective might Teach For America have, based on its company mission? A) Teach For America will transition from a nonprofit to a for-profit organization in 2015. B) Teach For America will open a food bank in low-income communities in each of its 39 regional offices. C) Teach For America will increase the number of its alumni in leadership positions within school systems across the United States by 10 percent each year. D) Teach For America will change the corps member service commitment to four years, versus two. E) Teach For America will open ten new regional offices by the year 2020. Answer: C Explanation: C) The mission emphasizes an importance of moving alumni toward leadership positions in public school systems throughout the United States. Page Ref: 124 Difficulty: Difficult AACSB: Reflective thinking skills Objective: 5.4 Learning Outcome: Describe the skills and functions of management Skill: Critical Thinking 69) Carlos wants to complete his M.B.A. in three years. This is an example of what kind of goal? A) long-term B) intermediate C) short-term D) objective E) problematic Answer: B Explanation: B) Intermediate goals are set for a period of one to five years. Page Ref: 124 Difficulty: Difficult AACSB: Reflective thinking skills Objective: 5.4 Learning Outcome: Explain how economic performance is monitored Skill: Application 70) Increasing sales by 4 percent in the next eight months is an example of what kind of goal? A) tactical B) strategic C) short-term D) intermediate E) long-term Answer: C Explanation: C) Short-term goals are set for a period of one year or less. Page Ref: 125 Difficulty: Difficult AACSB: Reflective thinking skills Objective: 5.4 Learning Outcome: Explain how economic performance is monitored Skill: Application 71) The marketing manager wants to increase regional sales by 2 percent each quarter over the next year. This is an example of what kind of goal? A) long-term B) intermediate C) short-term D) strategic E) tactical Answer: C Explanation: C) Short-term goals are set for one year or less. Page Ref: 125 Difficulty: Difficult AACSB: Reflective thinking skills Objective: 5.4 Learning Outcome: Explain how economic performance is monitored Skill: Application 72) Which of the following is the best example of unrelated diversification? A) Ford Motor Company merging with General Motors B) Northwest Airlines merging with Procter & Gamble C) McDonald's merging with Burger King D) Tootsie Roll merging with Hershey E) Bath and Body Works merging with The Body Store Answer: B Explanation: B) Northwest Airlines provides air travel for the public, whereas Procter & Gamble manufactures consumer goods. Page Ref: 125 Difficulty: Difficult AACSB: Reflective thinking skills Objective: 5.4 Learning Outcome: Describe the skills and functions of management Skill: Application 73) The first step in the planning process is determining goals. What is the next step? A) developing a strategic mission B) developing a comprehensive strategy C) developing an array of tactics D) developing an operational plan E) developing a functioning budget Answer: B Explanation: B) Planning begins when managers determine the firm's goals; next, a comprehensive strategy is developed. After that, tactical and operational plans for implementing the strategy are set. Page Ref: 126 Difficulty: Difficult Objective: 5.1 Learning Outcome: Describe the skills and functions of management Skill: Concept 74) Which variable of SWOT analysis identifies changing consumer tastes, hostile takeover offers, and new government regulations? A) threats B) opportunities C) strategies D) internal factors E) weaknesses Answer: A Explanation: A) Organizational threats are external variables that may adversely affect the organization; they can be outlined through SWOT analysis. Page Ref: 127 Difficulty: Difficult Objective: 5.4 Learning Outcome: Describe the skills and functions of management Skill: Concept 75) When conducting a SWOT analysis of an organization, which question can help management brainstorm for the strengths section of the analysis? A) What external changes present interesting possibilities? B) What aspect of the organization's vision can be altered? C) What necessary skills do the organization's employees currently lack? D) What does the organization offer that makes it stand out from other organizations? E) What external economic forces can affect the organization's bottom line? Answer: D Explanation: D) This question will help management brainstorm an organization's internal strengths. Page Ref: 127 Difficulty: Difficult AACSB: Reflective thinking skills Objective: 5.4 Learning Outcome: Describe the skills and functions of management Skill: Concept 76) When conducting a SWOT analysis of an organization, which question can help management brainstorm for the weaknesses section of the analysis? A) What external changes present interesting possibilities? B) What aspect of the organization's vision can be altered? C) What necessary skills do the organization's employees currently lack? D) What does the organization offer that makes it stand out from other organizations? E) What external economic forces can affect the organization's bottom line? Answer: C Explanation: C) This question will help management brainstorm an organization's internal weaknesses. Page Ref: 127 Difficulty: Difficult AACSB: Reflective thinking skills Objective: 5.4 Learning Outcome: Describe the skills and functions of management Skill: Concept 77) When conducting a SWOT analysis of an organization, which question can help management brainstorm for the opportunities section of the analysis? A) What external changes present interesting possibilities? B) What aspect of the organization's vision can be altered? C) What necessary skills do the organization's employees currently lack? D) What does the organization offer that makes it stand out from other organizations? E) What external economic forces can affect the organization's bottom line? Answer: A Explanation: A) This question will help management brainstorm opportunities that external constituents may offer. Page Ref: 127 Difficulty: Difficult AACSB: Reflective thinking skills Objective: 5.4 Learning Outcome: Describe the skills and functions of management Skill: Concept 78) When conducting a SWOT analysis of an organization, which question can help management brainstorm for the threats section of the analysis? A) What external changes present interesting possibilities? B) What aspect of the organization's vision can be altered? C) What necessary skills do the organization's employees currently lack? D) What does the organization offer that makes it stand out from other organizations? E) What external economic forces can affect the organization's bottom line? Answer: E Explanation: E) This question will help management evaluate outside factors that may pose a threat to the company. Page Ref: 127 Difficulty: Difficult AACSB: Reflective thinking skills Objective: 5.4 Learning Outcome: Describe the skills and functions of management Skill: Concept 79) Which of the following would be classified as opportunities for an organization? A) government regulations B) highly unionized workers C) new competitors D) possible company expansion E) economy fluctuations Answer: D Explanation: D) The other factors would typically be considered threats; possible expansion of the firm is an opportunity that could possibly lead to enhanced sales, increased market share, and a stronger market image. Page Ref: 127 Difficulty: Difficult Objective: 5.4 Learning Outcome: Describe the skills and functions of management Skill: Concept 80) What type of analysis focuses on external factors? A) organizational analysis B) governmental analysis C) environmental analysis D) functional analysis E) financial analysis Answer: C Explanation: C) External threats and opportunities are revealed through environmental analysis. Page Ref: 127 Difficulty: Difficult Objective: 5.4 Learning Outcome: Describe the skills and functions of management Skill: Concept 81) Which of the following is NOT considered an organizational strength? A) strong market position B) dedicated work force C) changing consumer tastes D) technical expertise E) recognized customer relation management Answer: C Explanation: C) Organizational strengths are any number of positive attributes within an organization from which the organization might benefit. Page Ref: 127 Difficulty: Difficult Objective: 5.4 Learning Outcome: Explain how economic performance is monitored Skill: Concept 82) Marty uses computer forecasts to determine possible outcomes of an intended change in his division. What element of management is Marty utilizing? A) strategy formulation B) goal setting C) crisis management D) contingency planning E) related diversification Answer: D Explanation: D) Contingency planning is planning for change: It seeks to identify in advance important aspects of a business or its market that might change. Page Ref: 130 Difficulty: Difficult AACSB: Reflective thinking skills Objective: 5.5 Learning Outcome: Describe the skills and functions of management Skill: Application 83) According to the CEOs of several companies that have undergone radical change in the last decade, what does NOT happen within such organizations? A) At the highest level, analysis of the company's environment highlights extensive change as the most effective response to its problems. B) Lower-level employees develop informal groups to plan for change. C) The firm sets up new systems for appraising and compensating employees who enforce the firm's new values. D) Top management begins to formulate a vision of a new company. E) Change is characterized by much conflict and resistance. Answer: B Explanation: B) Conflict and resistance typically characterize this period. The purpose is to give the new culture solid shape from within the firm. Whatever the new vision, it must include renewed focus on the activities of competitors and the needs of customers. Page Ref: 131 Difficulty: Difficult Objective: 5.6 Learning Outcome: Describe the skills and functions of management Skill: Application Chapter 6 Organizing the Business 37) Which term refers to the specification of jobs to be done within an organization and the ways in which those jobs relate to one another? A) delegation B) a mission statement C) job specialization D) role responsibility E) organizational structure Answer: E Explanation: E) Organizational structure refers to the jobs undertaken within an organization and how these jobs are related to one another. Page Ref: 141 Difficulty: Easy Objective: 6.1 Learning Outcome: Discuss the factors that influence decisions about organizational structure Skill: Concept 38) Which of the following clarifies structure and shows employees where they fit into a firm's operations? A) delegation B) departmentalization C) organization charts D) mission statement E) job descriptions Answer: C Explanation: C) An organization chart is a diagram showing the structure and employee placement in terms of operations. Page Ref: 141 Difficulty: Easy Objective: 6.1 Learning Outcome: Discuss the factors that influence decisions about organizational structure Skill: Concept 39) What do the solid lines on an organization chart show in reporting relationships within a company? A) mission statement B) chain of command C) job specialization D) grapevine activity E) market share Answer: B Explanation: B) The chain of command is shown on an organization chart as a solid line. Page Ref: 141 Difficulty: Easy Objective: 6.1 Learning Outcome: Discuss the factors that influence decisions about organizational structure Skill: Concept 40) What is the process of designing specific jobs that need to be done and identifying the people who will perform them called? A) departmentalization B) job specialization C) group tasks D) segmentation E) team cohesion Answer: B Explanation: B) Job specialization is a result of identifying specific roles that need to be undertaken within the organization. Page Ref: 142 Difficulty: Easy Objective: 6.2 Learning Outcome: Discuss the factors that influence decisions about organizational structure Skill: Concept 41) Which term covers grouping jobs into logical units? A) specialization B) accountability C) authorization D) departmentalization E) process management Answer: D Explanation: D) Departmentalization is the process of grouping jobs into logical units. Page Ref: 143 Difficulty: Easy Objective: 6.2 Learning Outcome: Discuss the factors that influence decisions about organizational structure Skill: Concept 42) What kind of departmentalization happens when an organization is divided into operations related to different products? A) functional departmentalization B) process departmentalization C) customer departmentalization D) geographic departmentalization E) regional departmentalization Answer: B Explanation: B) Process departmentalization refers to organizational divisions based on product process. Page Ref: 143 Difficulty: Easy Objective: 6.2 Learning Outcome: Discuss the factors that influence decisions about organizational structure Skill: Concept 43) What is the main organizational characteristic of decentralized firms? A) flat B) tall C) diverse D) multi-layered E) matrix Answer: A Explanation: A) Decentralized firms share the characteristic of a flat organizational structure. Page Ref: 146 Difficulty: Easy Objective: 6.3 Learning Outcome: Discuss the factors that influence decisions about organizational structure Skill: Concept 44) Which feature characterizes the organization of centralized firms? A) flat B) tall C) very large D) responsive to change E) well capitalized Answer: B Explanation: B) The main feature of centralized firms is their tall organizational structure. Page Ref: 146 Difficulty: Easy Objective: 6.3 Learning Outcome: Discuss the factors that influence decisions about organizational structure Skill: Concept 45) What is an employee's duty to perform an assigned task? A) delegation B) authority C) responsibility D) accountability E) specialization Answer: C Explanation: C) Within an organization, the duty to perform an assigned task is called responsibility. Page Ref: 148 Difficulty: Easy Objective: 6.3 Learning Outcome: Discuss the factors that influence decisions about organizational structure Skill: Concept 46) What is the power to make the decisions necessary to complete a task called? A) delegation B) specification C) authority D) responsibility E) accountability Answer: C Explanation: C) Authority is defined as the power to make decisions that are necessary to ensure the completion of a task. Page Ref: 148 Difficulty: Easy Objective: 6.3 Learning Outcome: Discuss the factors that influence decisions about organizational structure Skill: Concept 47) What is authority that flows up and down the chain of command called? A) staff authority B) departmental authority C) delegated authority D) divisional authority E) line authority Answer: E Explanation: E) Line authority is the term that describes the flow of authority along the chain of command. Page Ref: 148 Difficulty: Easy Objective: 6.3 Learning Outcome: Discuss the factors that influence decisions about organizational structure Skill: Concept 48) What type of departments are linked to the production and sales of specific products? A) staff departments B) matrix departments C) line departments D) organizational departments E) entrepreneurial departments Answer: C Explanation: C) Line departments are defined as departments linked to the production and sales of specific departments. Page Ref: 148 Difficulty: Easy Objective: 6.3 Learning Outcome: Discuss the factors that influence decisions about organizational structure Skill: Concept 49) What is authority given to a special group of employees that play central roles in the firm's daily operations called? A) risk aversion B) committee and team C) line D) matrix E) product and labor Answer: B Explanation: B) Committee and team authority is the term given to the authority exercised by a special group of employees. Page Ref: 149 Difficulty: Easy Objective: 6.3 Learning Outcome: Discuss the factors that influence decisions about organizational structure Skill: Concept 50) What are everyday social interactions among employees that transcend formal jobs and job interrelationships called? A) traditional structure B) informal organization C) networking opportunities D) production organization E) community structure Answer: B Explanation: B) The informal organization of a company includes the everyday social interactions among employees that go beyond the formal job-related interactions. Page Ref: 155 Difficulty: Easy Objective: 6.5 Learning Outcome: Discuss the factors that influence decisions about organizational structure Skill: Concept 51) Which term describes creating and maintaining the innovation and flexibility of a small business environment within the confines of a large, bureaucratic structure? A) basic structuring B) process organization C) intrapreneuring D) extrapolating E) product innovation Answer: C Explanation: C) Intrapreneuring is the creation and maintenance of the innovation and flexibility of a small business environment within the confines of a large, bureaucratic structure. Page Ref: 156 Difficulty: Easy Objective: 6.5 Learning Outcome: Discuss the factors that influence decisions about organizational structure Skill: Concept 52) Which factor plays a key role in determining an organization's structure? A) competition and technology B) location and life-cycle stage C) profitability and leverage D) research and development E) mission and strategy Answer: E Explanation: E) Many factors determine an organization's optimal structure, including mission, strategy, company size, and the organization's environment. Page Ref: 141 Difficulty: Moderate Objective: 6.1 Learning Outcome: Discuss the factors that influence decisions about organizational structure Skill: Concept 53) What do managers do when the "job" of a firm is broken down into smaller jobs? A) fragment the task B) specialize the job C) respond to the job D) assign the task E) control the job Answer: B Explanation: B) In a sense, all firms have one major job; to perform this one overall job, managers actually break it down, or specialize it, into several smaller jobs. Page Ref: 142 Difficulty: Moderate Objective: 6.2 Learning Outcome: Discuss the factors that influence decisions about organizational structure Skill: Concept 54) At Chaparral Steel, some employees transport scrap steel while others operate shredding equipment. What does this example demonstrate? A) departmentalization B) specialization C) functionalization D) responsibility E) innovation Answer: B Explanation: B) When the overall job of the organization is broken down like this, workers can develop real expertise in their jobs, and employees can better coordinate their work with that done by others. Page Ref: 142 Difficulty: Moderate Objective: 6.2 Learning Outcome: Discuss the factors that influence decisions about organizational structure Skill: Application 55) Which of the following is a natural part of organizational growth? A) retrenchment B) entrepreneurship C) job specialization D) outsourcing E) team consolidation Answer: C Explanation: C) As an organization grows, the need to specialize jobs so that others can perform them also becomes a priority. Page Ref: 142 Difficulty: Moderate Objective: 6.2 Learning Outcome: Discuss the factors that influence decisions about organizational structure Skill: Concept 56) Determining how people performing certain tasks can best be grouped together is called what? A) departmentalization B) specialization C) controlling D) leadership E) directing Answer: A Explanation: A) Departmentalized companies benefit from this division of activities; control and coordination are narrowed and made easier, and top managers can see more easily how various units are performing. Page Ref: 143 Difficulty: Moderate Objective: 6.2 Learning Outcome: Discuss the factors that influence decisions about organizational structure Skill: Concept 57) What type of departmentalization does Kraft Foods use by having separate divisions for different types of foods? A) customer B) functional C) geographic D) product E) brand Answer: D Explanation: D) In addition, a department can be treated as a profit center. Page Ref: 143 Difficulty: Moderate Objective: 6.2 Learning Outcome: Discuss the factors that influence decisions about organizational structure Skill: Application 58) Vlasic has separate departments to transform cucumbers into fresh-packed pickles, pickles cured in brine, and relishes. What is this an example of? A) functional departmentalization B) process departmentalization C) customer departmentalization D) geographic departmentalization E) centralized departmentalization Answer: B Explanation: B) With process departmentalization, the organization is divided according to production processes used to create a good or service. Page Ref: 143 Difficulty: Moderate Objective: 6.2 Learning Outcome: Discuss the factors that influence decisions about organizational structure Skill: Application 59) What is dividing a store into a men's department, a women's department, and a luggage department an example of? A) product departmentalization B) customer departmentalization C) process departmentalization D) geographic departmentalization E) international departmentalization Answer: B Explanation: B) Customer departmentalization is a form of organization in which departments are created to offer products and meet needs for identifiable customer groups. Page Ref: 144 Difficulty: Moderate Objective: 6.2 Learning Outcome: Discuss the factors that influence decisions about organizational structure Skill: Application 60) A snack food firm has one division for the United States, one for Europe, and another for Asia. How is this firm departmentalized? A) by customer B) by process C) by geography D) by function E) by productivity Answer: C Explanation: C) With geographic departmentalization, an organization is divided according to the areas of the country or the world that they serve. Page Ref: 144 Difficulty: Moderate Objective: 6.2 Learning Outcome: Discuss the factors that influence decisions about organizational structure Skill: Application 61) In what type of organization do most lower-level decisions need to be approved by upper management before they can be implemented? A) line B) staff C) decentralized D) centralized E) learning Answer: D Explanation: D) In a centralized organization, most decision-making authority is held by upper-level managers. Page Ref: 146 Difficulty: Moderate Objective: 6.3 Learning Outcome: Discuss the factors that influence decisions about organizational structure Skill: Concept 62) At McDonald's, most advertising is handled at the corporate level, and any local advertising must be approved by a regional manager. What kind of organization is McDonald's demonstrating? A) authoritarian B) committee C) decentralized D) centralized E) team Answer: D Explanation: D) McDonald's practices centralization as a way to maintain standardization. Page Ref: 146 Difficulty: Moderate Objective: 6.3 Learning Outcome: Discuss the factors that influence decisions about organizational structure Skill: Application 63) In what type of organization is decision-making authority delegated to levels of management at various points below the top? A) centralized B) decentralized C) point-of-sale D) geographic E) autocratic Answer: B Explanation: B) As a company gets larger, decision making tends to become more decentralized; decentralization is typical in firms that have complex and rapidly changing environmental conditions. Page Ref: 146 Difficulty: Moderate Objective: 6.3 Learning Outcome: Discuss the factors that influence decisions about organizational structure Skill: Concept 64) What kind of organization becomes more responsive to its environment by breaking the company into more manageable units? A) centralized B) decentralized C) geographic D) process-oriented E) brand conscious Answer: B Explanation: B) In doing so, the organization can operate more efficiently in complex, rapidly changing environmental conditions. Page Ref: 146 Difficulty: Moderate Objective: 6.3 Learning Outcome: Discuss the factors that influence decisions about organizational structure Skill: Concept 65) With relatively few layers of managers, what do decentralized organizations tend to reflect? A) tall organizational structure B) short organizational structure C) flat organizational structure D) elliptical organizational structure E) triangular organizational structure Answer: C Explanation: C) Delegation and authority are pushed downward in the organizational hierarchy. Page Ref: 146 Difficulty: Moderate Objective: 6.3 Learning Outcome: Discuss the factors that influence decisions about organizational structure Skill: Concept 66) Companies with centralized authority systems typically require multiple layers of management. What type of organizational structures are these companies utilizing? A) tall B) short C) flat D) broad E) oblique Answer: A Explanation: A) Decisions are made mostly by upper-level managers. Page Ref: 146 Difficulty: Moderate Objective: 6.3 Learning Outcome: Discuss the factors that influence decisions about organizational structure Skill: Concept 67) With its many organizational layers, what type of organizational structure does the U.S. Army demonstrate? A) tall B) step C) flat D) broad E) triangular Answer: A Explanation: A) The U.S. Army has a centralized, and tall, authority system. Page Ref: 146 Difficulty: Moderate Objective: 6.3 Learning Outcome: Discuss the factors that influence decisions about organizational structure Skill: Application 68) In tall organizations, which feature describes the span of control? A) short B) tall C) wide D) narrow E) vertical Answer: D Explanation: D) Due to multiple layers of employees, different levels of employee skill levels, job dissimilarities and job difficulties, tall organizations tend to lead to a narrower managerial span of control. Page Ref: 146 Difficulty: Moderate Objective: 6.3 Learning Outcome: Discuss the factors that influence decisions about organizational structure Skill: Concept 69) What is the preferred span of control when jobs are more diversified or prone to change? A) wide B) narrow C) tall D) deep E) authoritative Answer: B Explanation: B) Job difficulty and job dissimilarity can lead to a narrower span of control for managers. Page Ref: 146 Difficulty: Moderate Objective: 6.3 Learning Outcome: Discuss the factors that influence decisions about organizational structure Skill: Concept 70) Which of the following begins when a manager assigns responsibility to a subordinate? A) centralization B) delegation C) liability D) authority E) training Answer: B Explanation: B) The delegation process involves three steps: assigning responsibility, granting authority, and creating accountability. Page Ref: 148 Difficulty: Moderate Objective: 6.3 Learning Outcome: Discuss the factors that influence decisions about organizational structure Skill: Concept 71) After responsibility for a task is assigned, which of the following comes next? A) the centralization of the task B) the liability of the task C) the span of control is decided D) the work team is created E) the granting of authority Answer: E Explanation: E) The delegation process involves three steps: assigning responsibility, granting authority, and creating accountability. Page Ref: 148 Difficulty: Moderate Objective: 6.3 Learning Outcome: Discuss the factors that influence decisions about organizational structure Skill: Concept 72) Which of the following best describes responsibility? A) the power to make the decisions necessary to complete a task B) the liability of subordinates for accomplishing tasks assigned by managers C) the number of people supervised by one manager D) the duty to perform an assigned task E) the need to prove satisfactory work performance Answer: D Explanation: D) In contrast, authority is the power to make the decisions necessary to complete the task. Page Ref: 148 Difficulty: Moderate Objective: 6.3 Learning Outcome: Discuss the factors that influence decisions about organizational structure Skill: Concept 73) Which of the following best describes authority? A) the power to make the decisions necessary to complete a task B) the liability of subordinates for accomplishing tasks assigned by managers C) the number of people supervised by one manager D) the ability to take on new responsibility E) the duty to perform an assigned task Answer: A Explanation: A) In contrast, responsibility is the duty to perform an assigned task. Page Ref: 148 Difficulty: Moderate Objective: 6.3 Learning Outcome: Discuss the factors that influence decisions about organizational structure Skill: Concept 74) Which of the following best describes accountability? A) the power to make the decisions necessary to complete a task B) the obligation employees have to their manager for the successful completion of an assigned task C) the decision to communicate clear expectations related to assigned tasks D) the number of people supervised by one manager E) the duty to perform a financial task Answer: B Explanation: B) Creating accountability is the final step of the delegation process; for the process to work smoothly, authority and responsibility must be equivalent. Page Ref: 148 Difficulty: Moderate Objective: 6.3 Learning Outcome: Discuss the factors that influence decisions about organizational structure Skill: Concept 75) What authority is based on special expertise and usually involves counseling and advising? A) line authority B) matrix authority C) delegated authority D) staff authority E) divisional authority Answer: D Explanation: D) For example, a corporate attorney may be asked to advise the marketing department as it prepares a new contract with the firm's advertising agency. Page Ref: 149 Difficulty: Moderate Objective: 6.3 Learning Outcome: Discuss the factors that influence decisions about organizational structure Skill: Concept 76) What are specialists in law, accounting, and human resource management commonly called? A) profit centers B) cost centers C) staff members D) line members E) unit managers Answer: C Explanation: C) Staff members help departments in making decisions but do not usually have the authority to make final decisions. Page Ref: 149 Difficulty: Moderate Objective: 6.3 Learning Outcome: Discuss the factors that influence decisions about organizational structure Skill: Concept 77) What are groups of operating employees called who are empowered to plan and organize their tasks and to perform those tasks with a minimum of supervision? A) task forces B) standing committees C) work teams D) division committees E) board members Answer: C Explanation: C) The organization will usually find it beneficial to grant special authority to work teams so that they may function more effectively. Page Ref: 149 Difficulty: Moderate Objective: 6.3 Learning Outcome: Discuss the factors that influence decisions about organizational structure Skill: Concept 78) Which form of organization is used by most small to medium-sized firms? A) divisional B) functional C) matrix D) international E) stratified Answer: B Explanation: B) Functional structure is a form of organization in which authority is determined by the relationships between group functions and activities; it is based on the use of functional departmentalization at the highest level of the business. Page Ref: 150 Difficulty: Moderate Objective: 6.4 Learning Outcome: Discuss the factors that influence decisions about organizational structure Skill: Concept 79) Firms with what structure are structured around departments for each basic business action? A) divisional structure B) matrix structure C) functional structure D) international structure E) regional structure Answer: C Explanation: C) Benefits of this structure include specialization within functional areas and smoother coordination among them. Page Ref: 150 Difficulty: Moderate Objective: 6.4 Learning Outcome: Discuss the factors that influence decisions about organizational structure Skill: Concept 80) Which of the following structures relies on product departmentalization? A) matrix B) functional C) divisional D) international E) regional Answer: C Explanation: C) The firm organizes itself around product-based divisions, each of which may then be managed as a separate enterprise. Page Ref: 150 Difficulty: Moderate Objective: 6.4 Learning Outcome: Discuss the factors that influence decisions about organizational structure Skill: Concept 81) What are departments called that resemble separate businesses in that they produce and market their own products? A) regions B) divisions C) function centers D) cost centers E) work hubs Answer: B Explanation: B) Further, the head of each division may be a corporate vice president or, if the organization is large enough, a divisional president. Page Ref: 150 Difficulty: Moderate Objective: 6.4 Learning Outcome: Discuss the factors that influence decisions about organizational structure Skill: Concept 82) The organization of the health care producer Johnson & Johnson includes consumer health care products, medical devices and diagnostics, and pharmaceuticals. What kind of organizational structure does this company use? A) departments B) regions C) units D) divisions E) committees Answer: D Explanation: D) Johnson & Johnson can thus evaluate the performance of each division independently. Page Ref: 150 Difficulty: Moderate Objective: 6.4 Learning Outcome: Discuss the factors that influence decisions about organizational structure Skill: Application 83) What style of organization relies almost entirely on employees working on a common project, with little or no underlying functional hierarchy? A) team B) virtual C) learning D) flexible E) stratified Answer: A Explanation: A) People float from project to project as dictated by their skills and the demands of those projects. Page Ref: 154 Difficulty: Moderate Objective: 6.4 Learning Outcome: Discuss the factors that influence decisions about organizational structure Skill: Concept 84) What structure does an organization with little or no formal structure and with only a handful of permanent employees use? A) team structure B) virtual structure C) learning structure D) flexible structure E) balanced structure Answer: B Explanation: B) Typically, a virtual organization has only a handful of permanent employees, a very small staff, and a modest administrative facility. Page Ref: 154 Difficulty: Moderate Objective: 6.4 Learning Outcome: Discuss the factors that influence decisions about organizational structure Skill: Concept 85) What type of organization seeks to integrate continuous improvements with continuous employee development? A) team B) virtual C) learning D) flexible E) evolving Answer: C Explanation: C) A learning organization works to facilitate the learning and personal development of all of its employees while continually transforming itself to respond to changing demands and needs. Page Ref: 155 Difficulty: Moderate Objective: 6.4 Learning Outcome: Discuss the factors that influence decisions about organizational structure Skill: Concept 86) Which of the following organizational systems is the part that can be seen and represented in chart form? A) networking B) formal organization C) informal organization D) grapevine E) sales forecasting Answer: B Explanation: B) The structure of a company, however, is by no means limited to the organization chart and the formal assignment of authority. Page Ref: 155 Difficulty: Moderate Objective: 6.5 Learning Outcome: Discuss the factors that influence decisions about organizational structure Skill: Concept 87) What has Rubbermaid used to create and maintain the innovation and flexibility of a small business environment within the large, bureaucratic structure? A) formal organization B) informal organization C) intrapreneuring D) entrepreneurship E) capitalization Answer: C Explanation: C) The firm has one major division, the New Business Group. When a manager or engineer has an idea for a new product or product application, the individual takes it to the New Business Group and "sells" it. The managers in the group then help the innovator develop the idea for testing. Page Ref: 156 Difficulty: Moderate Objective: 6.5 Learning Outcome: Discuss the factors that influence decisions about organizational structure Skill: Application 88) Which of the following is an advantage of job specialization? A) Workers can develop expertise in their jobs. B) Workers will not need to be trained. C) Jobs become more interesting. D) Workers derive more satisfaction from their jobs. E) Jobs do not need to be redefined. Answer: A Explanation: A) However, jobs at lower levels of the organization are especially susceptible to overspecialization. If such jobs become too narrowly defined, employees may become bored and careless, derive less satisfaction from their jobs, and lose sight of their roles in the organization. Page Ref: 142 Difficulty: Moderate Objective: 6.2 Learning Outcome: Discuss the factors that influence decisions about organizational structure Skill: Synthesis 89) Procter & Gamble has Baby Care, Beauty Care, Health Care, and Home and Fabric Care divisions in its U.S. operations. What is this an example of? A) customer departmentalization B) process departmentalization C) geographic departmentalization D) functional departmentalization E) product departmentalization Answer: E Explanation: E) With product departmentalization, the organization is divided according to the specific product or service being created. Page Ref: 143 Difficulty: Difficult AACSB: Reflective thinking skills Objective: 6.2 Learning Outcome: Discuss the factors that influence decisions about organizational structure Skill: Application 90) American Wood Products has three divisions. The Lumberjack division is involved in harvesting trees and transporting them to lumber mills. The Milling division transforms cut timber into dimensional lumber for building. The Paper Products division makes paper and cardboard from the remnants of the milling process. What type of departmentalization is this an example of? A) functional departmentalization B) process departmentalization C) customer departmentalization D) geographic departmentalization E) brand departmentalization Answer: B Explanation: B) With process departmentalization, the organization is divided according to production processes used to create a good or service. Page Ref: 143 Difficulty: Difficult AACSB: Reflective thinking skills Objective: 6.2 Learning Outcome: Discuss the factors that influence decisions about organizational structure Skill: Application 91) Silver Designs is a high-end clothing manufacturer. They have three divisions producing clothes for men, women, and teens. What is this an example of? A) functional departmentalization B) process departmentalization C) customer departmentalization D) product departmentalization E) geographic departmentalization Answer: C Explanation: C) Customer departmentalization follows specialization for certain types of customers. Page Ref: 144 Difficulty: Moderate AACSB: Reflective thinking skills Objective: 6.2 Learning Outcome: Discuss the factors that influence decisions about organizational structure Skill: Application 92) Which statement about organizational structure is TRUE? A) Decentralized firms tend to have many layers of management. B) As a company gets larger, it tends to adopt a decentralized structure. C) The purpose of centralization is to make a company more responsive to a changing, complex environment. D) In a centralized organization, most decision-making authority is held by lower-level managers. E) Decentralized companies must change to centralized structures in response to unpredictable environments. Answer: B Explanation: B) In a decentralized structure, much decision-making authority is delegated to levels of management at various points below the top. Page Ref: 146 Difficulty: Difficult Objective: 6.3 Learning Outcome: Discuss the factors that influence decisions about organizational structure Skill: Concept 93) Jack Welch, former CEO of General Electric, stated, "If you don't let managers make their own decisions, you're never going to be anything more than a one-person business." What style of management is Welch proposing? A) authentic management B) linear management C) centralized management D) group management E) decentralized management Answer: E Explanation: E) In a decentralized structure, delegation of authority flows downward. Page Ref: 146 Difficulty: Moderate AACSB: Reflective thinking skills Objective: 6.3 Learning Outcome: Discuss the factors that influence decisions about organizational structure Skill: Application 94) What is the sequence of events in the delegation process? A) assigning responsibility follows granting authority B) granting authority follows creating accountability C) granting authority follows assigning responsibility D) assigning responsibility follows creating accountability E) assigning accountability follows creating authority Answer: C Explanation: C) The steps in the delegation process, in order, include assigning responsibility, granting authority, and creating accountability. Page Ref: 148 Difficulty: Moderate Objective: 6.3 Learning Outcome: Discuss the factors that influence decisions about organizational structure Skill: Concept 95) Which statement about line departments is TRUE? A) They include those with special counseling and advising expertise. B) They include those directly linked to the sales of products. C) They include staff members who provide services to management. D) They have little effect on the production of goods. E) Customer preference has little effect on them. Answer: B Explanation: B) Line departments include those employees directly linked to the production and sales of products. Page Ref: 148 Difficulty: Moderate Objective: 6.3 Learning Outcome: Discuss the factors that influence decisions about organizational structure Skill: Concept 96) Which of the following are the "doers" and producers in a company? A) line employees B) staff employees C) matrix employees D) the legal department E) the finance department Answer: A Explanation: A) Line employees include those related to goods and services production. Page Ref: 148 Difficulty: Difficult Objective: 6.3 Learning Outcome: Discuss the factors that influence decisions about organizational structure Skill: Concept 97) Which statement about staff responsibility is TRUE? A) It is likely to include those employees involved in the production of products. B) It is not likely to include those who provide services to managers. C) It is linked directly to delivery of a product. D) It is clearly different from line authority. E) It is not related to the work of managers. Answer: D Explanation: D) The separation between line authority and staff responsibility is clearly delineated; staff responsibility is typically carried out by those employees who provide services to managers. Page Ref: 148 Difficulty: Difficult Objective: 6.3 Learning Outcome: Discuss the factors that influence decisions about organizational structure Skill: Concept 98) Which feature will a committee have? A) inclusion of only top managers from one specific area B) mandate to change the corporate culture C) some decision-making responsibility D) ability to work together only for a short period of time E) inclusion of employees from all company levels Answer: C Explanation: C) A committee will likely be comprised of top managers from several major areas; the entire committee may be granted decision-making power. Page Ref: 149 Difficulty: Difficult Objective: 6.3 Learning Outcome: Discuss the factors that influence decisions about organizational structure Skill: Concept 99) At Ace Products, three vice presidents report to the CEO, including the VP for Finance, the VP for Marketing, and the VP for Operations. What type of structure does Ace Products have? A) divisional B) matrix C) regional D) functional E) international Answer: D Explanation: D) Functional structure is a form of business organization in which authority is determined by the relationships between group functions and activities. Page Ref: 150 Difficulty: Moderate AACSB: Reflective thinking skills Objective: 6.4 Learning Outcome: Discuss the factors that influence decisions about organizational structure Skill: Application 100) What does functional structure foster? A) competitiveness B) centralization C) decentralization D) diversity E) accountability Answer: B Explanation: B) This, in turn, makes accountability more difficult. Page Ref: 150 Difficulty: Difficult AACSB: Analytic skills Objective: 6.4 Learning Outcome: Discuss the factors that influence decisions about organizational structure Skill: Concept 101) Thomas Corp. has three basic departments: children's books, games, and toys. What type of structure does the company have? A) divisional B) matrix C) functional D) process E) global Answer: A Explanation: A) A divisional structure relies on product departmentalization. Page Ref: 150 Difficulty: Difficult AACSB: Reflective thinking skills Objective: 6.4 Learning Outcome: Discuss the factors that influence decisions about organizational structure Skill: Application 102) Which of the following types of structure entails a second chain of command? A) divisional B) departmental C) committee D) matrix E) team Answer: D Explanation: D) A matrix structure is often referred to as a "combination" structure. Page Ref: 151 Difficulty: Difficult Objective: 6.4 Learning Outcome: Discuss the factors that influence decisions about organizational structure Skill: Concept 103) To provide a state-of-the-art classroom and instructional technology, Shell Oil recently purchased an executive conference center north of Houston. What type of organization does this purchase reflect? A) team B) virtual C) learning D) acquisitive E) traditional Answer: C Explanation: C) The facility boasts state-of-the-art classrooms and instructional technology, lodging facilities, a restaurant, and recreational amenities. Page Ref: 155 Difficulty: Difficult Objective: 6.4 Learning Outcome: Discuss the factors that influence decisions about organizational structure Skill: Application Chapter 7 Operations Management and Quality 36) Which term refers to all the activities involved in making products—goods and services—for customers? A) operations B) processes C) planning D) scheduling E) performance Answer: A Explanation: A) "Operations" and "production" refer to the same activities. Page Ref: 166 Difficulty: Easy Objective: 7.1 Learning Outcome: Discuss the factors that influence decisions about organizational structure Skill: Concept 37) Which of the following is the ability of a product to satisfy a human want or need? A) utility B) relative advantage C) content D) satisfaction E) capacity Answer: A Explanation: A) Production adds customer value by providing utility. Page Ref: 166 Difficulty: Easy Objective: 7.2 Learning Outcome: Discuss the roles of ethics and corporate responsibility in business Skill: Concept 38) Which type of utility is created when a company makes products available where consumers want them? A) time B) place C) possession D) form E) operations Answer: B Explanation: B) An example of place utility is a theater showing a wide selection of movies in a popular shopping mall. Page Ref: 167 Difficulty: Easy Objective: 7.2 Learning Outcome: Discuss the roles of ethics and corporate responsibility in business Skill: Concept 39) Which term refers to a set of methods and technologies used to produce a good or service? A) contingency plan B) production episode C) reduction D) methods plan E) operations process Answer: E Explanation: E) Banks use document shredding and data encryption to protect confidential information. Both are examples of operations processes. Page Ref: 169 Difficulty: Easy Objective: 7.2 Learning Outcome: Discuss the factors that influence decisions about organizational structure Skill: Concept 40) In which of the following is the customer part of the system during service delivery? A) a high-contact system B) a low-contact system C) a customer system D) a quality system E) a utility system Answer: A Explanation: A) A manicure is an example of a high-contact system. Page Ref: 170 Difficulty: Easy Objective: 7.2 Learning Outcome: Discuss the factors that influence decisions about organizational structure Skill: Concept 41) What term describes a special ability that production does especially well to outperform the competition? A) supply chain management B) total quality management C) process engineering D) operations capability E) value-added analysis Answer: D Explanation: D) A company chooses an operation capability that is compatible with its overall business strategy. Page Ref: 170 Difficulty: Easy Objective: 7.3 Learning Outcome: Discuss the factors that influence decisions about organizational structure Skill: Concept 42) Which of the following is the amount of a product that a company can produce under normal working conditions ? A) capacity B) output C) efficiency D) burden E) performance Answer: A Explanation: A) A firm's capacity depends on how many people it employs and the number and size of its facilities. Page Ref: 173 Difficulty: Easy Objective: 7.4 Learning Outcome: Discuss the factors that influence decisions about organizational structure Skill: Concept 43) Equipment and people are grouped according to function in which type of production layout? A) cellular B) same-steps C) custom-products D) supply-chain E) service Answer: C Explanation: C) A custom-products layout is well suited to make-to-order shops. Page Ref: 174 Difficulty: Easy Objective: 7.4 Learning Outcome: Discuss the factors that influence decisions about organizational structure Skill: Concept 44) Thousands of empty jugs move down a conveyor belt at Colgate-Palmolive before stopping to be filled and moving on to be labeled. What type of layout is utilized? A) supply-chain B) custom-products C) hybrid D) fixed-position E) same-steps Answer: E Explanation: E) An assembly line is a same-steps layout. Page Ref: 175 Difficulty: Easy Objective: 7.4 Learning Outcome: Discuss the factors that influence decisions about organizational structure Skill: Application 45) Which term refers to how well a product does what it is supposed to do? A) capacity B) consistency C) reliability D) performance E) quality control Answer: D Explanation: D) The performance of a fast-food meal might be judged by whether it tastes good and is reasonably filling. Page Ref: 175 Difficulty: Easy Objective: 7.4 Learning Outcome: Discuss the roles of ethics and corporate responsibility in business Skill: Concept 46) The sameness of product quality from unit to unit is referred to by which term? A) utility B) reliability C) performance D) consistency E) quality control Answer: D Explanation: D) High consistency at its many locations has made Courtyard by Marriott a leader in the lodging industry. Page Ref: 175 Difficulty: Easy Objective: 7.4 Learning Outcome: Discuss the roles of ethics and corporate responsibility in business Skill: Concept 47) Which of the following is a scheduling tool that breaks down large projects into steps to be performed and specifies the time required to perform each one? A) Myers graphic B) critical path chart C) Gantt chart D) process analysis graph E) master schedule Answer: C Explanation: C) A project manager uses a Gantt chart to keep the project moving on schedule. Page Ref: 178 Difficulty: Easy Objective: 7.5 Learning Outcome: Discuss the factors that influence decisions about organizational structure Skill: Concept 48) Which type of system is designed for smooth production flow to avoid inefficiencies, eliminate unnecessary inventories, and continuously improve production processes? A) quality system B) lean system C) managed system D) production system E) flexible system Answer: B Explanation: B) Lean production systems were pioneered by Toyota. Page Ref: 180 Difficulty: Easy Objective: 7.6 Learning Outcome: Discuss the factors that influence decisions about organizational structure Skill: Concept 49) Which of the following is NOT a major objective of JIT production? A) increasing stop-and-go production B) reducing goods in process C) complying with schedules D) eliminating disruptions by continuous improvement E) reducing inventory levels Answer: A Explanation: A) JIT production replaces stop-and-go production with smooth movement. Page Ref: 180 Difficulty: Easy Objective: 7.6 Learning Outcome: Discuss the factors that influence decisions about organizational structure Skill: Critical Thinking 50) Which term refers to the acquisition of the raw materials a company needs to produce its products? A) development B) purchasing C) goods control D) quality control E) wholesaling Answer: B Explanation: B) Most large firms have purchasing departments to buy proper services and materials in the amounts needed. Page Ref: 180 Difficulty: Easy Objective: 7.6 Learning Outcome: Discuss the factors that influence decisions about organizational structure Skill: Concept 51) Which of the following refers to the receiving, storing, handling, and counting of all raw materials, partly finished goods, and finished goods? A) materials handling B) inventory control C) quality control D) wholesaling E) distribution control Answer: B Explanation: B) Inventory control ensures that enough materials inventories are available to meet production schedules, while at the same time avoiding expensive excess inventories. Page Ref: 180 Difficulty: Easy Objective: 7.6 Learning Outcome: Discuss the factors that influence decisions about organizational structure Skill: Concept 52) Which of the following terms refers to all activities involved in getting quality products into the marketplace? A) quality control B) quality reliability C) total quality management D) performance quality E) quality overview Answer: C Explanation: C) TQM must consider all aspects of a business, including customers, suppliers, and employees. Page Ref: 181 Difficulty: Easy Objective: 7.7 Learning Outcome: Describe the skills and functions of management Skill: Concept 53) Which term describes the idea that quality belongs to each person who creates it while performing a job? A) quality control B) quality ownership C) quality circles D) total quality management E) quality improvement teams Answer: B Explanation: B) A goal of total quality management is to motivate employees to accept quality ownership. Page Ref: 182 Difficulty: Easy Objective: 7.7 Learning Outcome: Discuss the factors that affect motivation and behavior in the workplace Skill: Concept 54) What name is given to the process by which a company analyzes another company's product to identify desirable improvements in its own product? A) quality reliability analysis B) benchmarking C) performance quality analysis D) competitive product analysis E) quality outsourcing Answer: D Explanation: D) Using competitive analysis, for example, Toshiba might take apart a Xerox copier and test each component. Page Ref: 182 Difficulty: Easy Objective: 7.7 Learning Outcome: Discuss the factors that influence decisions about organizational structure Skill: Concept 55) Which of the following is patterned after the successful Japanese concept of quality circles? A) supply chain management B) quality improvement team C) total quality management D) quality ownership E) value-added analysis Answer: B Explanation: B) Quality improvement teams organize their own work, select leaders, and address problems in the workplace. Page Ref: 184 Difficulty: Easy Objective: 7.7 Learning Outcome: Discuss the factors that influence decisions about organizational structure Skill: Concept 56) Which term refers to the flow of information, materials, and services from raw-materials suppliers through stages in the operations process until the product reaches the end customer? A) distribution chain B) supply chain C) service channel D) distribution channel E) performance channel Answer: B Explanation: B) The term supply chain refers to the group of companies and stream of activities that work together to create a product. Page Ref: 185 Difficulty: Easy Objective: 7.8 Learning Outcome: Discuss the factors that influence decisions about organizational structure Skill: Concept 57) Which of the following is another term for the supply chain? A) the distribution chain B) the distribution network C) the value chain D) the supply network E) the wholesale network Answer: C Explanation: C) Each stage in the value chain adds value for the final customer. Page Ref: 185 Difficulty: Easy Objective: 7.8 Learning Outcome: Discuss the factors that influence decisions about organizational structure Skill: Concept 58) Which term refers to the strategy of paying suppliers and distributors to perform certain business processes or to provide needed materials or services? A) outsourcing B) reengineering C) controlling D) scheduling E) warehousing Answer: A Explanation: A) The decision to outsource expands supply chains. Page Ref: 186 Difficulty: Easy Objective: 7.8 Learning Outcome: Discuss the factors that influence decisions about organizational structure Skill: Concept 59) When a company turns out costumes in time for Halloween, it creates which type of utility? A) ownership B) time C) form D) place E) value Answer: B Explanation: B) Time utility is created when products are made available when customers want them. Page Ref: 167 Difficulty: Moderate Objective: 7.2 Learning Outcome: Discuss the roles of ethics and corporate responsibility in business Skill: Application 60) When a company makes products available where they are convenient for consumers, it creates which type of utility? A) time B) form C) place D) possession E) leisure Answer: C Explanation: C) A fast-food restaurant located in a college union is an example of place utility. Page Ref: 167 Difficulty: Moderate Objective: 7.2 Learning Outcome: Discuss the roles of ethics and corporate responsibility in business Skill: Concept 61) In a business, whose job is it to draw up plans to transform resources into products and bring together basic resources, such as knowledge, physical materials, equipment, and labor? A) CEO B) operations manager C) quality manager D) floor foreman E) supply manager Answer: B Explanation: B) Operations managers are responsible for ensuring that operations processes create value and provide benefits to customers. Page Ref: 167 Difficulty: Moderate Objective: 7.2 Learning Outcome: Describe the skills and functions of management Skill: Concept 62) Which term describes services which cannot be produced ahead of time? A) transformed B) low-contact C) unstorable D) intangible E) consistent Answer: C Explanation: C) Unstorable services cannot be used at a later time. Page Ref: 169 Difficulty: Moderate Objective: 7.2 Learning Outcome: Discuss the factors that influence decisions about organizational structure Skill: Concept 63) Which of the following would be considered a low-contact service? A) surgery B) haircut C) massage D) electric power E) piano lesson Answer: D Explanation: D) All of the other choices are considered high-contact services in which the consumer must be present during the service transaction. Page Ref: 170 Difficulty: Moderate Objective: 7.2 Learning Outcome: Discuss the factors that influence decisions about organizational structure Skill: Application 64) The check-processing operations at your bank would be considered to be which type of system? A) high-contact B) low-contact C) conversion D) synthetic E) performance Answer: B Explanation: B) Check-processing operations can be completed without the consumer being present. Page Ref: 170 Difficulty: Moderate Objective: 7.2 Learning Outcome: Discuss the factors that influence decisions about organizational structure Skill: Application 65) Which of the following would be considered a high-contact system? A) gas company B) electric company C) lawn care company D) barber shop E) postal delivery Answer: D Explanation: D) A barber's services require that the consumer be present during the service transaction. Page Ref: 170 Difficulty: Moderate AACSB: Analytic skills Objective: 7.2 Learning Outcome: Discuss the factors that influence decisions about organizational structure Skill: Critical Thinking 66) Save-A-Lot grocery stores use which type of strategy for attracting customers? A) flexibility B) quality C) low-cost D) dependability E) consistency Answer: C Explanation: C) Save-A-Lot offers items at savings up to 40 percent less than conventional food chains. Page Ref: 171 Difficulty: Moderate Objective: 7.4 Learning Outcome: Discuss the factors that influence decisions about organizational structure Skill: Application 67) 3M uses which strategy for attracting customers? A) quality B) low-cost C) dependability D) flexibility E) consistency Answer: D Explanation: D) The flexibility strategy at 3M emphasizes new product development. Page Ref: 171 Difficulty: Moderate Objective: 7.4 Learning Outcome: Discuss the factors that influence decisions about organizational structure Skill: Application 68) Machine, woodworking, and dry cleaning shops typically use which type of layout? A) product B) custom-products C) same-steps D) hybrid E) make-to-stock Answer: B Explanation: B) In a custom-products layout, machines and people are grouped by function in the production facility; custom-products layouts allow for greater flexibility and are well suited to make-to-order shops. Page Ref: 174 Difficulty: Moderate Objective: 7.4 Learning Outcome: Discuss the factors that influence decisions about organizational structure Skill: Application 69) Which type of layout is designed to move resources through a smooth, fixed sequence of steps? A) same-steps B) location C) custom-products D) quality E) make-to-order Answer: A Explanation: A) A same-steps layout is set up to make one type of product in a fixed sequence and is arranged according to its production requirements. Page Ref: 175 Difficulty: Moderate Objective: 7.4 Learning Outcome: Discuss the factors that influence decisions about organizational structure Skill: Concept 70) Automobile, food-processing, and television assembly plants use which type of layout? A) make-to-order B) location C) custom-products D) quality E) same-steps Answer: E Explanation: E) These production processes require a fixed sequence that is arranged according to production requirements. Page Ref: 175 Difficulty: Moderate Objective: 7.4 Learning Outcome: Discuss the factors that influence decisions about organizational structure Skill: Application 71) Which term is defined as the combination of "characteristics of a product or service that bear on its ability to satisfy stated or implied needs"? A) quality B) production C) quantity D) clarification E) capacity Answer: A Explanation: A) Such characteristics can include a reasonable price. Page Ref: 175 Difficulty: Moderate Objective: 7.4 Learning Outcome: Discuss the roles of ethics and corporate responsibility in business Skill: Concept 72) Performance refers to which of the following? A) the principle that quality belongs to each person who creates it while performing a job B) the consistency of product quality from unit to unit C) how well the product does what it is supposed to do D) the process by which a company analyzes a competitor's products to identify desirable improvement E) providing value by making products available when customers want them Answer: C Explanation: C) Planning for quality begins when products are being designed. Early in the process, goals are established for both performance and consistency. Page Ref: 175 Difficulty: Moderate Objective: 7.4 Learning Outcome: Discuss the roles of ethics and corporate responsibility in business Skill: Concept 73) Consistency refers to which of the following? A) the principle that quality belongs to each person who creates it while performing a job B) the sameness of product quality from unit to unit C) the sum of all activities involved in getting high-quality products into the marketplace D) the process by which a company analyzes a competitor's products to identify desirable improvements E) providing value by making products available where customers want them Answer: B Explanation: B) This consistency is achieved by controlling for consistent raw materials, encouraging conscientious work, and maintaining equipment. Page Ref: 175 Difficulty: Moderate Objective: 7.4 Learning Outcome: Discuss the roles of ethics and corporate responsibility in business Skill: Concept 74) Managers can work to reduce waste, inefficiency, and poor performance by examining procedures on a step-by-step basis. Which term describes this process? A) advance planning B) materials management C) methods improvement D) quality planning E) quality ownership Answer: C Explanation: C) A process flowchart is sometimes used to assist in methods improvement. Page Ref: 176 Difficulty: Moderate Objective: 7.4 Learning Outcome: Describe the skills and functions of management Skill: Concept 75) Logan Aluminum makes coils of aluminum that it supplies to customer companies that use it to make beverage cans. Logan uses a schedule that specifies how many tons of each type of coil will be produced each week. What is this schedule called? A) master production schedule B) detailed schedule C) Gantt chart D) staff schedule E) PERT chart Answer: A Explanation: A) The master production schedule shows which products will be produced, and when, in upcoming time periods. Page Ref: 177 Difficulty: Moderate Objective: 7.5 Learning Outcome: Describe the skills and functions of management Skill: Application 76) In operations control, production managers monitor production performance by which method? A) comparing results with detailed plans and schedules B) checking on each worker on the production floor C) observing activities from a platform above the workers D) constantly talking with employees involved E) performing spot checks of worker performance Answer: A Explanation: A) If schedules or quality standards aren't met, managers can take corrective action. Page Ref: 179 Difficulty: Moderate Objective: 7.6 Learning Outcome: Describe the skills and functions of management Skill: Concept 77) Which of the following is NOT one of the areas of materials management? A) transportation B) warehousing C) purchasing D) supplier selection E) customer service Answer: E Explanation: E) Materials management includes supplier selection, purchasing, transportation, warehousing, and inventory control. Page Ref: 180 Difficulty: Moderate Objective: 7.6 Learning Outcome: Describe the skills and functions of management Skill: Concept 78) What is the name for a production system in which all the needed materials and parts arrive at the precise moment they are required for each production stage? A) quality control system B) process control system C) just-in-time production D) standardized production E) custom-products production Answer: C Explanation: C) JIT production brings together all needed materials at the precise moment they are required for each production stage, not before, creating efficient responses to customer orders. Page Ref: 180 Difficulty: Moderate Objective: 7.6 Learning Outcome: Discuss the factors that influence decisions about organizational structure Skill: Concept 79) Which of the following is greatly reduced in a just-in-time production system? A) number of workers needed on the line B) number of goods in process C) number of shifts necessary at the plant D) number of separate operations on the assembly line E) number of forepersons needed at the plant Answer: B Explanation: B) As a result, JIT production minimizes inventory costs and saves money by replacing stop-and-go production with smooth movement. Page Ref: 180 Difficulty: Moderate Objective: 7.6 Learning Outcome: Discuss the factors that influence decisions about organizational structure Skill: Concept 80) Which of the following is a component of materials management? A) promotion B) distribution C) marketing D) warehousing E) sales Answer: D Explanation: D) Materials management includes supplier selection, purchasing, transportation, warehousing, and inventory control. Page Ref: 180 Difficulty: Moderate Objective: 7.6 Learning Outcome: Describe the skills and functions of management Skill: Concept 81) Which of the following best describes total quality management? A) the principle that quality belongs to each person who creates it while performing a job B) the consistency of a product's quality from unit to unit C) the sum of all activities involved in getting high-quality products into the marketplace D) the process by which a company analyzes a competitor's products to identify desirable improvements E) a certification program attesting that an operations process has met rigorous requirements Answer: C Explanation: C) TQM begins with leadership and a desire for continuously improving both processes and products; it must consider all aspects of a business, including customers, suppliers, and employees. Page Ref: 181 Difficulty: Moderate Objective: 7.7 Learning Outcome: Describe the skills and functions of management Skill: Concept 82) Which of the following best describes quality ownership? A) an emphasis on intangible factors in customer satisfaction B) the consistency of a product's quality from unit to unit C) the sum of all activities involved in getting high-quality products into the marketplace D) the process by which a company analyzes a competitor's products to identify desirable improvements E) the principle that quality belongs to each person who creates it while performing a job Answer: E Explanation: E) Quality-focused leaders use various methods to foster a quality focus among the workforce; when these efforts succeed, employees will ultimately accept quality ownership. Page Ref: 182 Difficulty: Moderate Objective: 7.7 Learning Outcome: Discuss the factors that affect motivation and behavior in the workplace Skill: Concept 83) Which of the following best describes competitive product analysis? A) the principle that quality belongs to each person who creates it while performing a job B) the consistency of a product's quality from unit to unit C) the process by which a company analyzes a different company's products to identify desirable improvements D) the sum of all activities involved in getting a high-quality product into the marketplace. E) examining a product to improve its form utility Answer: C Explanation: C) This analysis helps managers decide which product features are satisfactory, which features should be upgraded, and which operations processes need improvement, for example. Page Ref: 182 Difficulty: Moderate Objective: 7.7 Learning Outcome: Discuss the factors that influence decisions about organizational structure Skill: Concept 84) When a worker at Toshiba takes apart a Xerox copier and tests each component, it is engaging in what activity? A) competitive product analysis B) benchmarking C) total quality management D) quality reliability analysis E) value-added analysis Answer: A Explanation: A) Competitive product analysis is the process by which a company analyzes a competitor's products to identify desirable improvements. Page Ref: 182 Difficulty: Moderate AACSB: Reflective thinking skills Objective: 7.7 Learning Outcome: Discuss the factors that influence decisions about organizational structure Skill: Application 85) Which of the following best describes value-added analysis? A) a TQM tool in which groups of employees work together to improve quality B) the process of evaluating all work activities, materials flows, and paperwork to determine the value that they create for customers C) the process by which a company implements the best practices from its own past performance and those of other companies to improve its own products D) the process by which a company analyzes a product to identify possible improvements E) a total company commitment to quality management at every stage of the process Answer: B Explanation: B) Value-added analysis often reveals wasteful or unnecessary activities that can be eliminated without jeopardizing customer service. Page Ref: 183 Difficulty: Moderate Objective: 7.7 Learning Outcome: Discuss the factors that influence decisions about organizational structure Skill: Concept 86) Tootsie Roll Industry's corporate principle, "We run a trim operation and continually strive to eliminate waste, minimize costs, and implement performance improvements," is an example of which of the following? A) just-in-time production B) benchmarking C) a quality circle D) competitive product analysis E) value-added analysis Answer: E Explanation: E) Tootsie Roll Industry evaluates all work activities, material flows, and paperwork to determine the value that they add for customers. Page Ref: 183 Difficulty: Moderate AACSB: Reflective thinking skills Objective: 7.7 Learning Outcome: Discuss the factors that influence decisions about organizational structure Skill: Application 87) Which term refers to collaborative groups of employees from various work areas who meet regularly to define, analyze, and solve common production problems? A) quality improvement teams B) quality assurance teams C) quality control teams D) TQM teams E) quality review teams Answer: A Explanation: A) Quality improvement teams organize their own work, select leaders, and address problems in the workplace. Page Ref: 184 Difficulty: Moderate Objective: 7.7 Learning Outcome: Discuss the factors that influence decisions about organizational structure Skill: Concept 88) Which of the following best describes ISO 9000? A) the concept that all employees are valuable contributors to a firm's business, and should be entrusted with decisions regarding their work B) the redesigning of business processes to improve performance, quality, and productivity C) a program certifying that a factory, laboratory, or office has met the quality management standards of the International Organization for Standardization D) a certification program attesting to the fact that a factory, laboratory, or office has improved environmental performance E) the process of evaluating all work activities to determine the value they add for customers Answer: C Explanation: C) ISO 9000 certifies that a factory, laboratory, or office has met the quality management standards of ISO; these standards are now regarded as a national standard for more than 160 countries. Page Ref: 184 Difficulty: Moderate Objective: 7.7 Learning Outcome: Discuss the roles of ethics and corporate responsibility in business Skill: Concept 89) Which program certifies improvements in environmental performance? A) ISO 9000 B) ISO 9000:2000 C) ISO 14000 D) TQM 2001 E) TQM 2012 Answer: C Explanation: C) Extending the ISO approach into the arena of environmental protection and hazardous waste management, ISO 14000 requires a firm to develop an environmental management system. Page Ref: 184 Difficulty: Moderate Objective: 7.7 Learning Outcome: Discuss the roles of ethics and corporate responsibility in business Skill: Concept 90) Which of the following best describes ISO 14000? A) the concept that all employees are valuable contributors to a firm's business and should be entrusted with decisions regarding their work B) a certification program attesting to the fact that a factory, laboratory, or office has improved environmental performance C) a certification program attesting to the fact that a factory, laboratory, or office has met the quality management standards of the International Organization for Standardization D) a standardized method of evaluating a company's greenhouse gas emissions E) a certification program attesting to the fact that a factory, laboratory, or office is operating under free-trade principles Answer: B Explanation: B) ISO 14000 requires a firm to develop an environmental management system, documenting how the company has acted to improve its performance in using resources and in managing pollution. Page Ref: 184 Difficulty: Moderate Objective: 7.7 Learning Outcome: Discuss the roles of ethics and corporate responsibility in business Skill: Concept 91) Receiving and storing materials, billing patients for treatment, and filling customer orders from Internet sales are examples of which activity? A) function B) technique C) process D) benchmark E) quality ownership Answer: C Explanation: C) Every business consists of processes, which are activities that are performed regularly and routinely in conducting business. Page Ref: 185 Difficulty: Moderate Objective: 7.7 Learning Outcome: Discuss the factors that influence decisions about organizational structure Skill: Application 92) Which term refers to the fundamental rethinking and radical redesign of a business activity to achieve dramatic improvements in performance? A) supply chain management B) business process reengineering C) total quality management D) quality ownership E) quality control Answer: B Explanation: B) Rethinking the production steps by starting from scratch has allowed dramatic improvements in cost, quality, service, and speed. Page Ref: 185 Difficulty: Moderate Objective: 7.7 Learning Outcome: Discuss the factors that influence decisions about organizational structure Skill: Concept 93) Which of the following best describes business process reengineering? A) the redesigning of business activities to improve performance, quality, and productivity B) a program certifying that a factory, laboratory, or office has met the quality management standards of the International Organization for Standardization C) the principle of looking at the supply chain as a whole in order to improve the overall flow through the system D) a program certifying that a factory, laboratory, or office has developed pollution-control standards E) the process by which a company analyzes a different company's products to identify desirable improvements Answer: A Explanation: A) Business process reengineering focuses on improving a business process, rethinking each of its steps from start to finish. Page Ref: 185 Difficulty: Moderate Objective: 7.7 Learning Outcome: Discuss the factors that influence decisions about organizational structure Skill: Concept 94) Which of the following best describes supply chain management? A) the principle of improving the supply chain by focusing on the slowest step in the chain B) a limitation on the number of suppliers allowed to supply a particular company C) complete movement of raw materials throughout a manufacturing or service facility D) a cost-reduction program in which wholesalers and retailers are eliminated in an effort to entice consumers to purchase directly from manufacturers E) the principle of looking at the supply chain as a whole in order to improve the overall flow through the system Answer: E Explanation: E) In addition, because customers ultimately get better value, supply chain management gains competitive advantage for each supply chain member. Page Ref: 186 Difficulty: Moderate Objective: 7.8 Learning Outcome: Describe the skills and functions of management Skill: Concept 95) When Dell Inc. shares information to improve the overall flow through a system composed of companies working together, it is engaging in which of the following? A) distribution chain management B) total quality management C) supply chain management D) service channel analysis E) supplier selection analysis Answer: C Explanation: C) Michael Dell's vision involves the concept of improving performance by sharing information among supply chain members; long-term production plans and up-to-the-minute sales data are available to suppliers via the Internet. Page Ref: 186 Difficulty: Moderate Objective: 7.8 Learning Outcome: Describe the skills and functions of management Skill: Application 96) When Ford assembles parts into a Ford Explorer, it creates which type of utility? A) time B) form C) possession D) place E) ownership Answer: B Explanation: B) Form utility is created when raw materials are converted into finished products. Page Ref: 167 Difficulty: Moderate Objective: 7.2 Learning Outcome: Discuss the factors that influence decisions about organizational structure Skill: Application 97) Which operations competence has Toyota given greatest credit for winning orders in the marketplace? A) quality B) low cost C) flexibility D) dependability E) exclusivity Answer: A Explanation: A) Toyota has focused on creating reliable cars with an appealing fit and finish, and assuring that customer expectations are met or exceeded in order to build the company's quality competence. Page Ref: 171 Difficulty: Moderate Objective: 7.3 Learning Outcome: Discuss the factors that influence decisions about organizational structure Skill: Application 98) Which operations competence has FedEx given greatest credit for winning orders (business) in the marketplace? A) quality B) low cost C) flexibility D) dependability E) accessibility Answer: D Explanation: D) FedEx wants to assure that every delivery is fast and on time, as promised. Page Ref: 171 Difficulty: Moderate Objective: 7.3 Learning Outcome: Discuss the factors that influence decisions about organizational structure Skill: Application 99) Examining step-by-step procedures to reduce inefficiency most centrally involves which approach? A) operations scheduling B) performance analysis C) value-added analysis D) methods improvement E) quality ownership Answer: D Explanation: D) Methods improvement has been used to streamline the traditional checkout method at hotels. Page Ref: 176 Difficulty: Difficult Objective: 7.4 Learning Outcome: Describe the skills and functions of management Skill: Concept 100) Which scheduling tool shows the necessary sequence of activities in a project and identifies the critical path? A) Gantt chart B) process flowchart C) detailed schedule D) master production schedule E) PERT chart Answer: E Explanation: E) The critical path is the most time-consuming set of activities in a project. Page Ref: 177 Difficulty: Difficult Objective: 7.5 Learning Outcome: Discuss the factors that influence decisions about organizational structure Skill: Concept 101) When Hewlett-Packard simplified its contracts and reduced them from 20 pages to as few as 2, it was engaging in which activity? A) value-added analysis B) benchmarking C) getting closer to the customer D) supply chain management E) follow-up Answer: A Explanation: A) Value-added analysis refers to the evaluation of all work activities, materials flows, and paperwork to determine the value that they add for customers. Page Ref: 183 Difficulty: Difficult Objective: 7.7 Learning Outcome: Describe the skills and functions of management Skill: Application Chapter 8 Employee Behavior and Motivation 24) What is the term for behavior that is related to doing a certain job? A) performance behavior B) interpersonal behavior C) disruptive behavior D) potential behavior E) assertive behavior Answer: A Explanation: A) Performance behaviors are the total set of work-related behaviors that an organization expects employees to display. Page Ref: 196 Difficulty: Easy Objective: 8.1 Learning Outcome: Discuss the factors that affect motivation and behavior in the workplace Skill: Concept 25) Which would an employee who is productive but keeps to himself be lacking in? A) organizational citizenship B) workplace aggression C) performance behavior D) personal productivity E) self-esteem Answer: A Explanation: A) Good organizational citizenship includes such factors as being willing to work overtime and to help orient new employees. Page Ref: 197 Difficulty: Moderate Objective: 8.1 Learning Outcome: Discuss the factors that affect motivation and behavior in the workplace Skill: Concept 26) Behaviors that detract from, instead of contribute to, organizational performance are known as what? A) inclusive B) disruptive C) counterproductive D) turnover E) associative Answer: C Explanation: C) The prefix counter- means "against," and so counterproductive behaviors work against production and organizational performance. Page Ref: 197 Difficulty: Easy Objective: 8.1 Learning Outcome: Discuss the factors that affect motivation and behavior in the workplace Skill: Concept 27) Which of the following is NOT a central aspect of organizational citizenship? A) reporting structure B) corporate culture C) workgroup attitude D) business field E) team composition Answer: D Explanation: D) The qualities that make a good organizational citizen, such as willingness to work overtime if asked, are common across business fields. Page Ref: 197 Difficulty: Moderate Objective: 8.1 Learning Outcome: Discuss the factors that affect motivation and behavior in the workplace Skill: Concept 28) When does turnover occur? A) when people quit their jobs B) when managers reprimand staff C) when employers cut down costs D) when employees call in sick E) when benefits are improved Answer: A Explanation: A) Turnover is counterproductive because a business must hire a replacement and, in the meantime, other employees have to complete the work of the employee who has left. Page Ref: 197 Difficulty: Easy Objective: 8.1 Learning Outcome: Discuss the factors that affect motivation and behavior in the workplace Skill: Concept 29) Which of the following would have the lowest cost to an organization? A) absenteeism B) workplace violence C) sexual harassment D) theft E) racism Answer: A Explanation: A) Although absenteeism does have a cost to the organization, its cost is lower than the monetary and emotional cost of workplace violence, the cost of potential lawsuits from harassment (both in dollars and in lost reputation), and the cost of theft, which can run into the millions. Page Ref: 197 Difficulty: Difficult Objective: 8.1 Learning Outcome: Discuss the factors that affect motivation and behavior in the workplace Skill: Concept 30) Which of these behaviors best indicates good organizational citizenship? A) being willing to help new employees B) asking for a pay raise C) meeting job performance standards D) keeping regular hours E) using office supplies for business use Answer: A Explanation: A) Good employee citizenship is more than simply showing up and doing the job, but extends to improving and supporting other workers and the organization. Page Ref: 197 Difficulty: Moderate Objective: 8.1 Learning Outcome: Discuss the factors that affect motivation and behavior in the workplace Skill: Concept 31) Which of these is one of the "big five" personality traits? A) openness B) logic C) creativity D) organization E) musical talent Answer: A Explanation: A) The "big five" personality traits (agreeableness, conscientiousness, negative emotionality, extroversion, and openness) can be viewed as continuums that run from high levels of the trait to low levels of the trait. Page Ref: 197 Difficulty: Moderate AACSB: Multicultural and diversity understanding Objective: 8.2 Learning Outcome: Discuss the factors that affect motivation and behavior in the workplace Skill: Concept 32) What is emotionality? A) the ability of a person to get along with others B) the degree to which someone tends to be positive or negative C) the number of things a person tries to accomplish at once D) the relative rigidity of an individual's beliefs E) the likelihood that a person will react aggressively Answer: B Explanation: B) Emotionality is a continuum of positivity and negativity. Page Ref: 199 Difficulty: Moderate AACSB: Multicultural and diversity understanding Objective: 8.2 Learning Outcome: Discuss the factors that affect motivation and behavior in the workplace Skill: Concept 33) Which of the following might you, as a manager, expect to see in an employee who has a low degree of conscientiousness? A) a tendency to be unprepared at meetings B) an inability to work in a team setting C) a tendency toward mood swings D) a high level of comfort with other people E) an ability to meet deadlines without fail Answer: A Explanation: A) Conscientious employees tend to be thorough and organized and would not arrive at a meeting unprepared. Page Ref: 199 Difficulty: Moderate AACSB: Multicultural and diversity understanding Objective: 8.2 Learning Outcome: Discuss the factors that affect motivation and behavior in the workplace Skill: Application 34) You need to assign a task to one of your employees. The task involves a high level of detail. To whom should you assign the task? A) Amy, who exhibits high agreeableness and low conscientiousness B) David, who exhibits more negative emotionality and low agreeableness C) Sarah, who exhibits high openness and low negative emotionality D) Winston, who exhibits high conscientiousness and low extraversion E) Liza, who exhibits high creativity and low negative emotionality Answer: D Explanation: D) Winston is your most conscientious employee and will probably excel at the task. Page Ref: 199-200 Difficulty: Moderate AACSB: Analytic skills Objective: 8.2 Learning Outcome: Discuss the factors that affect motivation and behavior in the workplace Skill: Application 35) The extent to which people are self-aware is an aspect of which of the following? A) attitude B) emotional intelligence C) negative emotionality D) social skills E) empathy Answer: B Explanation: B) Self-awareness, including recognizing and controlling one's emotions, is an important aspect of emotional intelligence. Page Ref: 200 Difficulty: Moderate Objective: 8.2 Learning Outcome: Discuss the factors that affect motivation and behavior in the workplace Skill: Concept 36) An employee with a low level of social skills would be unlikely to do which of the following? A) be very detail oriented B) circulate a get-well card C) produce creative work D) work well independently E) track milestones effectively Answer: B Explanation: B) People with low levels of social skills tend to have difficulty forming positive relationships. Page Ref: 200 Difficulty: Difficult AACSB: Analytic skills Objective: 8.2 Learning Outcome: Discuss the factors that affect motivation and behavior in the workplace Skill: Application 37) Which of the following is the set of expectations held by employees and the organization regarding what the employee will contribute and what he or she will receive? A) psychological contract B) nondisclosure agreement C) job satisfaction D) mutual commitment E) self expression Answer: A Explanation: A) Although it is not written, the psychological contract is an understanding that each party will provide a benefit to the other. Page Ref: 201 Difficulty: Moderate Objective: 8.3 Learning Outcome: Discuss the factors that affect motivation and behavior in the workplace Skill: Concept 38) Which of the following aspects of the psychological contract is provided by the organization? A) competency B) effort C) loyalty D) status E) talent Answer: D Explanation: D) Status within the organization is one of the contractual aspects that the organization provides to employees, along with pay, benefits, job security, and job and career opportunities. Page Ref: 202 Difficulty: Moderate Objective: 8.3 Learning Outcome: Discuss the factors that affect motivation and behavior in the workplace Skill: Concept 39) Which of the following aspects of the psychological contract is provided by the employee? A) benefits B) pay C) skills D) security E) status Answer: C Explanation: C) The employee brings his or her skill to the position as part of the psychological contract, along with effort, loyalty, ability, time, and competency. Page Ref: 202 Difficulty: Moderate Objective: 8.3 Learning Outcome: Discuss the factors that affect motivation and behavior in the workplace Skill: Concept 40) Michelle has been working at AdCo for five years. The company recently hired a new college graduate to work in a position very similar to Michelle's position. In a casual conversation with the new hire, Michelle discovers that the new employee is making $4,000 more annually than she is. What might Michelle's reaction be? A) Michelle may feel that the psychological contract has been broken. B) Michelle may feel that her job description no longer applies. C) Michelle may feel that her supervisor is not empathetic. D) Michelle may feel that her person-job fit cannot be improved. E) Michelle may feel that she has benefitted from the Hawthorne effect. Answer: A Explanation: A) The pay inequity between herself and a newcomer would make Michelle feel that there has been a violation of the status aspect of the psychological contract. Page Ref: 202 Difficulty: Moderate AACSB: Ethical understanding and reasoning abilities Objective: 8.3 Learning Outcome: Discuss the factors that affect motivation and behavior in the workplace Skill: Application 41) Because job permanence is less likely, which of the following are some companies offering in order to keep the psychological contract in balance? A) additional promotions B) additional vacation time C) fewer training opportunities D) flexible scheduling E) additional sick days Answer: D Explanation: D) Offering a new option to employees when one factor of the psychological contract is uncertain helps keep the relationship solid. Page Ref: 201 Difficulty: Moderate Objective: 8.3 Learning Outcome: Discuss the factors that affect motivation and behavior in the workplace Skill: Concept 42) What is the term for the extent to which a person's contributions and the organization's inducements match? A) employee equity B) person-job fit C) psychological contract D) time-motion study E) team culture Answer: B Explanation: B) If there is a poor match between a person's contributions, such as skills, and the inducements, such as pay, the result will be a dissatisfied and unhappy employee. Page Ref: 202 Difficulty: Moderate Objective: 8.3 Learning Outcome: Discuss the factors that affect motivation and behavior in the workplace Skill: Concept 43) If an employee needs to feel that she is a part of a team and yet her position involves working alone, which of the following may not be right? A) the pay scale B) the person-job fit C) the employee's attitude D) the organization's plan E) the team structure Answer: B Explanation: B) Aspects of a person's personality should be taken into consideration when assigning tasks. Page Ref: 202 Difficulty: Moderate Objective: 8.3 Learning Outcome: Discuss the factors that affect motivation and behavior in the workplace Skill: Concept 44) Jane is a very conscientious worker, but she has poor social skills. Which of the following jobs might be best for her? A) working independently on spreadsheets B) providing customer service C) handling problems in a call center D) training new employees E) working in public relations Answer: A Explanation: A) Asking Jane to work independently on a detailed task makes the best use of her abilities. Page Ref: 199 Difficulty: Moderate Objective: 8.2 Learning Outcome: Discuss the factors that affect motivation and behavior in the workplace Skill: Application 45) Brooke was excited to take a job at Film, Inc. She was looking forward to working with the man who had hired her because of his experience and status within the industry. Eight months after her hire, Brooke's supervisor retired. Which aspect of the psychological contract might Brooke feel has been violated? A) benefits B) career opportunities C) pay D) job security E) talent Answer: B Explanation: B) Brooke may have thought that by working with such a mentor, her own career prospects would be improved, but his retirement changed that. Page Ref: 201 Difficulty: Difficult Objective: 8.3 Learning Outcome: Discuss the factors that affect motivation and behavior in the workplace Skill: Application 46) What is the Hawthorne effect? A) the belief that employees are motivated by money alone B) the conclusion that workers are more productive if management pays attention to them C) the determination that the level of lighting in a workplace has no effect on productivity D) the theory that workers are more productive if their jobs are analyzed closely E) the unspoken agreement between the employee and the employer Answer: B Explanation: B) Almost any management action that workers viewed as special attention caused an increase in productivity in this study at the Hawthorne Works. Page Ref: 203 Difficulty: Moderate Objective: 8.4 Learning Outcome: Discuss the factors that affect motivation and behavior in the workplace Skill: Concept 47) What is MOST useful about Theory X and Theory Y? A) They shed light on managers' attitudes toward employees. B) They provide a useful blueprint for taking action. C) They help managers better understand employees. D) They accurately depict the attitude of employees at work. E) They support management decisions and plans. Answer: A Explanation: A) By understanding his or her own attitudes toward employees, a manager can better understand how he or she relates to employees. Page Ref: 204 Difficulty: Moderate Objective: 8.4 Learning Outcome: Discuss the factors that affect motivation and behavior in the workplace Skill: Concept 48) According to Maslow's model, a set of needs will be a motivator until which of the following occurs? A) The needs are satisfied. B) New needs are identified. C) The person becomes self-motivated. D) A supervisor changes the work. E) The needs are replaced by hygiene factors. Answer: A Explanation: A) Once a set of needs has been satisfied, it is no longer a motivator. Page Ref: 204 Difficulty: Moderate Objective: 8.4 Learning Outcome: Discuss the factors that affect motivation and behavior in the workplace Skill: Concept 49) Which of the following lists Maslow's Hierarchy of Human Needs in order, starting with the most basic? A) physiological, security, social, esteem, self-actualization B) social, security, physiological, esteem, self-actualization C) physiological, self-actualization, security, esteem, social D) physiological, esteem, security, social, self-actualization E) esteem, self-actualization, social, physiological, security Answer: A Explanation: A) The most basic needs relate to survival (physiological), and then proceed to become more complex until they deal with personal satisfaction. Page Ref: 205 Difficulty: Easy Objective: 8.4 Learning Outcome: Discuss the factors that affect motivation and behavior in the workplace Skill: Concept 50) Douglas believes that his employees are self-motivated and growth-oriented. He is what type of manager? A) Theory Y B) Theory X C) classical D) hierarchical Answer: A Explanation: A) According to Theory X, people are lazy and lack ambition, but the view of people (especially employees) in Theory Y is generally much more positive, viewing people as energetic and eager for responsibility. Page Ref: 204 Difficulty: Easy AACSB: Reflective thinking skills Objective: 8.4 Learning Outcome: Discuss the factors that affect motivation and behavior in the workplace Skill: Application 51) Which of the following theories focuses on people evaluating their treatment by the organization relative to the treatment of others? A) equity theory B) hierarchy of needs C) expectancy theory D) goal-setting theory E) X and Y theory Answer: A Explanation: A) Equity theory is one of the contemporary, more complex theories of motivation. Page Ref: 207 Difficulty: Easy Objective: 8.4 Learning Outcome: Discuss the factors that affect motivation and behavior in the workplace Skill: Concept 52) Employees are given a voice in how they do their jobs and how the company is managed according to what theory? A) participative management and empowerment B) job enrichment C) job design D) modified work schedules E) scientific management Answer: A Explanation: A) This allows employees to take more responsibility for their own performance. Page Ref: 210 Difficulty: Moderate Objective: 8.5 Learning Outcome: Discuss the factors that affect motivation and behavior in the workplace Skill: Concept 53) Which of the following works to achieve a more satisfactory fit between workers and their jobs? A) job redesign B) team management C) MBO D) participative management E) person-position fitting Answer: A Explanation: A) Instead of finding a worker to fit a job, the job is tailored to the worker. Page Ref: 211 Difficulty: Easy Objective: 8.5 Learning Outcome: Discuss the factors that affect motivation and behavior in the workplace Skill: Concept 54) Which is an example of a modified work schedule? A) job sharing B) job redesign C) job rotation D) job enrichment E) job elimination Answer: A Explanation: A) Job sharing allows for the accommodation of the needs of employees, increasing the likelihood that they will be retained. Page Ref: 211 Difficulty: Easy Objective: 8.5 Learning Outcome: Discuss the factors that affect motivation and behavior in the workplace Skill: Concept 55) Which of the following allows people to choose their working hours by adjusting a standard work schedule on a daily or weekly basis? A) job enrichment B) flextime programs C) job enlargement D) job redesign E) work sharing Answer: B Explanation: B) Flextime allows workers to accommodate the needs of their personal life, such as family and childcare demands. Page Ref: 211 Difficulty: Easy Objective: 8.5 Learning Outcome: Discuss the factors that affect motivation and behavior in the workplace Skill: Concept 56) What is telecommuting an example of? A) job sharing B) job rotation C) modified work scheduling D) work sharing E) modified workplace planning Answer: C Explanation: C) Telecommuting allows employees to start their days earlier or extend them past the normal closing time, which is especially useful when dealing with a global client base. Page Ref: 211 Difficulty: Easy Objective: 8.4 Learning Outcome: Discuss the factors that affect motivation and behavior in the workplace Skill: Concept 57) Which of the following best describes scientific management? A) a theory of motivation holding that job satisfaction depends on two factors, hygiene and motivation B) a theory of motivation holding that people are motivated to work toward rewards that they want and that they have a reasonable chance of obtaining C) a theory of motivation holding that people evaluate their treatment by the organization relative to the treatment of others D) a theory of motivation holding that managers should analyze jobs to find the most efficient methods and use money as a primary motivator E) a theory of motivation that people will work more productively as long as they receive some kind of attention from management Answer: D Explanation: D) Scientific management led to the application of industrial techniques to each facet of a job to determine how to perform it most efficiently. Page Ref: 204 Difficulty: Moderate Objective: 8.4 Learning Outcome: Discuss the factors that affect motivation and behavior in the workplace Skill: Concept 58) Which of the following helps explain why some people do not work as hard as they can when their salaries are based purely on seniority? A) two-factor theory B) Maslow's hierarchy of human needs C) scientific management D) expectancy theory E) Theory X and Theory Y Answer: D Explanation: D) Paying employees the same whether they work very hard or just hard enough to get by removes the financial incentive for them to work harder. Page Ref: 204 Difficulty: Moderate Objective: 8.4 Learning Outcome: Discuss the factors that affect motivation and behavior in the workplace Skill: Concept 59) Tina is the most productive employee in her department. When her manager retires, Tina doesn't apply for the job, in spite of her excellent record and the pay and prestige associated with the job. She believes that the job will go to someone who has more seniority. Based on expectancy theory, which would explain why Tina doesn't apply for the job? A) the effort-performance issue B) the rewards-personal goals issue C) the performance-reward issue D) her personal goals E) her family demands Answer: C Explanation: C) Expectancy theory suggests that employees work harder toward reaching a goal if they know they have a good chance of reaching it; Tina doesn't feel that her chance of getting the position will be based on her excellent performance. Page Ref: 206 Difficulty: Moderate Objective: 8.4 Learning Outcome: Discuss the factors that affect motivation and behavior in the workplace Skill: Application 60) Sue works very hard and is the most productive worker on her dayshift. When a position opens on the evening shift, Sue is encouraged to apply but does not; she has children and wants to be home in the evening. Based on expectancy theory, which explains why Sue doesn't apply for the job? A) the effort-performance issue B) the rewards-personal goals issue C) the performance-rewards issue D) her personal goals E) her family demands Answer: B Explanation: B) Sue does not feel that the rewards obtained from the new position will outweigh the value she places on being with her family in the evening. Page Ref: 206 Difficulty: Moderate Objective: 8.4 Learning Outcome: Discuss the factors that affect motivation and behavior in the workplace Skill: Application 61) Kent wants to motivate the maintenance staff to be more productive. He starts by providing training and assures employees that high productivity will be rewarded. Employees are asked to identify the rewards that would mean the most to them. Kent likely subscribes to which theory of motivation? A) equity B) expectancy C) two-factor D) hierarchy of needs E) productivity Answer: B Explanation: B) Kent is attempting to convince the maintenance staff that the company's rewards for their work will be worth the increased productivity; expectancy theory suggests that people are motivated when they feel they have a good chance of obtaining the desired rewards. Page Ref: 206 Difficulty: Difficult AACSB: Analytic skills Objective: 8.4 Learning Outcome: Discuss the factors that affect motivation and behavior in the workplace Skill: Application 62) Terry generally likes her job and works hard. However, Terry's level of effort has declined recently. Terry became aware that a new employee, with far less experience, was hired at Terry's current salary. Terry's lack of motivation can best be described by which theory of motivation? A) equity B) expectancy C) two-factor D) scientific management E) planning Answer: A Explanation: A) Equity theory focuses on people evaluating their treatment by the organization relative to the treatment of others. Page Ref: 207 Difficulty: Difficult AACSB: Analytic skills Objective: 8.4 Learning Outcome: Discuss the factors that affect motivation and behavior in the workplace Skill: Application Chapter 9 Leadership and Decision Making 37) Which of the following describes the concept of leadership? A) the processes and behaviors used by someone to determine the optimal decision among several options B) the processes and behaviors used by someone to motivate, inspire, and influence the behaviors of others C) the processes and behaviors used by someone to establish the organizational structure of a firm D) the processes and behaviors used by someone to manage the functioning of an organization E) the processes and behaviors used by someone to achieve tangible outcomes within an organization Answer: B Explanation: B) Leaders set agendas and inspire others to follow their direction. Page Ref: 222 Difficulty: Easy Objective: 9.1 Learning Outcome: Summarize the major theories of and approaches to leadership Skill: Concept 38) Whereas managers focus on plans, results, and goals, what do leaders focus on? A) productivity B) profitability C) people D) sales E) organization Answer: C Explanation: C) Leadership is concerned with motivating and inspiring people, rather than controlling and organizing a company. Page Ref: 222 Difficulty: Easy Objective: 9.1 Learning Outcome: Summarize the major theories of and approaches to leadership Skill: Concept 39) Which tasks do managers typically focus on? A) monitoring organizational goals B) agenda setting C) aligning the organization with its environment D) inspiring employees E) planning and budgeting Answer: E Explanation: E) Management is concerned with the running of an organization. Page Ref: 222 Difficulty: Easy Objective: 9.1 Learning Outcome: Summarize the major theories of and approaches to leadership Skill: Concept 40) When creating an agenda, what do leaders focus on? A) establishing direction B) controlling and problem solving C) organizing and staffing D) outlining procedures E) planning and budgeting Answer: A Explanation: A) This is the prime concern of a leader. Page Ref: 222 Difficulty: Easy Objective: 9.1 Learning Outcome: Summarize the major theories of and approaches to leadership Skill: Concept 41) When developing the human resources for achieving an agenda, what do managers focus on? A) establishing direction B) controlling and problem solving C) organizing and staffing D) aligning people E) motivating and inspiring Answer: C Explanation: C) This is the primary methodology for a manager to achieve his or her goals. Page Ref: 222 Difficulty: Easy Objective: 9.1 Learning Outcome: Summarize the major theories of and approaches to leadership Skill: Concept 42) When developing the human resources necessary for achieving an agenda, what do leaders focus on? A) establishing direction B) controlling and problem solving C) organizing and staffing D) aligning people and goals E) establishing workplace policies Answer: D Explanation: D) Aligning people to work together toward an agenda is one of the leadership responsibilities. Page Ref: 222 Difficulty: Easy Objective: 9.1 Learning Outcome: Summarize the major theories of and approaches to leadership Skill: Concept 43) What area was the earliest research on leadership most concerned with? A) leadership behaviors B) leadership traits C) leadership situations D) leadership integrity E) leadership roles Answer: B Explanation: B) The trait approach to leadership yielded various lists of attributes that successful leaders often have. Page Ref: 223 Difficulty: Easy Objective: 9.2 Learning Outcome: Summarize the major theories of and approaches to leadership Skill: Concept 44) What was a major problem with the trait approach to leadership? A) The results were too innovative and thus were not taken seriously at the time. B) The results were too inconsistent and thus not very practical. C) The results were not very inspiring and thus ignored. D) The results were found to be too pessimistic and thus were dismissed as useless. E) The results were too worshipful of certain leaders and thus were dismissed as irrelevant. Answer: B Explanation: B) These early studies never really managed to agree with each other. Page Ref: 223 Difficulty: Difficult Objective: 9.2 Learning Outcome: Summarize the major theories of and approaches to leadership Skill: Concept 45) Which of the following best describes task-focused leader behavior? A) concentration on how tasks should be performed B) concentration on which tasks should be performed C) concentration on why tasks should be performed D) concentration on who should perform which tasks E) concentration on when tasks should be performed Answer: A Explanation: A) Task-focused leader behavior occurs when a leader focuses on how tasks should be performed in order to meet certain goals and to achieve certain performance standards. Page Ref: 224 Difficulty: Easy Objective: 9.2 Learning Outcome: Summarize the major theories of and approaches to leadership Skill: Concept 46) Which of the following best describes employee-focused leader behavior? A) concentration on both the work quality and work quantity of employees B) concentration on both the well-being and work quality of employees C) concentration on both the satisfaction and work quantity of employees D) concentration on both the satisfaction and the well-being of employees E) concentration on both the work quantity and the well-being of employees Answer: D Explanation: D) Employee-focused leader behavior occurs when a leader focuses on the satisfaction, motivation, and well-being of his or her employees. Page Ref: 224 Difficulty: Easy Objective: 9.2 Learning Outcome: Summarize the major theories of and approaches to leadership Skill: Concept 47) Which of the following describes one of the primary concerns associated with charismatic leadership due to severe devotion to a particular leader? A) the potential lapse of ethical practices B) the potential inability to replace leadership efficiently C) the potential encouragement of micromanaging D) the potential stagnation of corporate vision E) the potential creation of bureaucracy Answer: A Explanation: A) Although many of these reflect hazards of charismatic leadership, overzealousness of supporters has often lead to serious ethical lapses of judgment. Page Ref: 226 Difficulty: Moderate AACSB: Reflective thinking skills Objective: 9.4 Learning Outcome: Summarize the major theories of and approaches to leadership Skill: Concept 48) Bill Lopez spends most of his time performing normal, routine tasks. Which of the following types of leadership is he primarily involved with? A) charismatic leadership B) transformational leadership C) behavioral leadership D) situational leadership E) transactional leadership Answer: E Explanation: E) Transactional leadership is essentially the same as management in that it involves routine, regimented activities. Page Ref: 226 Difficulty: Easy AACSB: Reflective thinking skills Objective: 9.4 Learning Outcome: Summarize the major theories of and approaches to leadership Skill: Application 49) Which of the following accurately describes a primary characteristic of charismatic leaders? A) They tend to set high expectations for themselves and energize others. B) They tend to set high expectations for themselves and establish high standards for others. C) They tend to establish high standards for and energize others. D) They tend to energize others and establish good, stable practices. E) They tend to set high expectations for themselves and establish good, stable practices. Answer: A Explanation: A) High standards are practiced through example in charismatic leadership, more than insisting that everyone follow them. Page Ref: 226 Difficulty: Difficult Objective: 9.4 Learning Outcome: Summarize the major theories of and approaches to leadership Skill: Concept 50) Which of the following organizational aspects allows employees to perform capably without the direction of a leader? A) leadership neutralizers B) leadership substitutes C) leadership incentives D) leadership guidelines E) leadership obstructions Answer: B Explanation: B) Leadership substitutes are individual, task, and organizational characteristics that tend to outweigh the need for a leader to initiate or direct employee performance. Page Ref: 227 Difficulty: Easy Objective: 9.5 Learning Outcome: Summarize the major theories of and approaches to leadership Skill: Concept 51) Which of the following best describes the effects of leadership neutralizers in an organization? A) They tend to make leader behavior ineffective. B) They tend to make leaders more charismatic. C) They tend to outweigh the need for leader behavior. D) They tend to make managers become effective leaders. E) They tend to make leaders more neutral toward employees. Answer: A Explanation: A) Leadership neutralizers are factors that may render leader behaviors ineffective. Page Ref: 227 Difficulty: Easy Objective: 9.5 Learning Outcome: Summarize the major theories of and approaches to leadership Skill: Concept 52) Which of the following may be substitutes for leadership? A) lax individual professionalism B) group divisiveness C) explicit plans and goals D) a loose system of rules and procedures E) a performance-based reward system Answer: C Explanation: C) These may make the need for a leader to plan things out ineffective or unnecessary. Page Ref: 227 Difficulty: Easy Objective: 9.5 Learning Outcome: Summarize the major theories of and approaches to leadership Skill: Concept 53) Bob, Joe, and Larry can all perform their jobs well without supervision. What does this indicate? A) presence of a rigid reward system B) presence of a leadership neutralizer C) presence of a leadership substitute D) presence of a leadership incentive E) presence of a highly automated system Answer: C Explanation: C) This is precisely an example of a situation for which a leader is not needed. Page Ref: 227 Difficulty: Moderate AACSB: Reflective thinking skills Objective: 9.5 Learning Outcome: Summarize the major theories of and approaches to leadership Skill: Application 54) The norms of the engineering group at Acme Incorporated are so strong that there is nothing the new company leader can do to change things. What does this illustrate? A) leadership charisma B) leadership neutralizers C) leadership substitutes D) leadership mandates E) leadership amplifiers Answer: B Explanation: B) This is one example of a leadership neutralizer. Page Ref: 227 Difficulty: Moderate Objective: 9.5 Learning Outcome: Summarize the major theories of and approaches to leadership Skill: Application 55) Which of the following types of leaders has the primary role of helping a less experienced person learn how to function and to advance within an organization? A) enabler B) mentor C) manager D) task master E) motivator Answer: B Explanation: B) Mentors help less experienced employees learn the ropes and advance within the organization. Page Ref: 229 Difficulty: Easy Objective: 9.6 Learning Outcome: Summarize the major theories of and approaches to leadership Skill: Concept 56) Which is the most accurate statement regarding one of the main differences between the Japanese and American cultures? A) Overall, Japanese culture is often characterized as conservative, whereas American culture is often characterized as liberal. B) Overall, Japanese culture is often characterized as rigidly structured, whereas American culture is often characterized as loosely structured. C) Overall, Japanese culture is often characterized as authoritarian, whereas American culture is often characterized as democratic. D) Overall, Japanese culture is often characterized as collectivist, whereas American culture is often characterized as individualist. E) Overall, Japanese culture is often characterized as socialist, whereas American culture is often characterized as capitalist. Answer: D Explanation: D) This is often the primary difference between the structures of Japanese and American firms. Page Ref: 229-230 Difficulty: Easy AACSB: Multicultural and diversity understanding Objective: 9.6 Learning Outcome: Summarize the major theories of and approaches to leadership Skill: Application 57) Which part of the management of an organization would be most involved with strategic leadership? A) the top level of management B) the supervisory level of management C) the advisory level of management D) the auxiliary level of management E) all levels of management Answer: A Explanation: A) Strategic leadership explicitly relates leadership to the role of top management. Page Ref: 230 Difficulty: Easy Objective: 9.7 Learning Outcome: Summarize the major theories of and approaches to leadership Skill: Concept 58) Recent corporate scandals at firms like Enron have brought increased attention to what area of leadership? A) virtual leadership B) ethical leadership C) strategic leadership D) employee-focused leadership E) charismatic leadership Answer: B Explanation: B) This type of scrutiny, had it been present, may have prevented the 2008 economic meltdown. Page Ref: 230 Difficulty: Easy AACSB: Ethical understanding and reasoning abilities Objective: 9.7 Learning Outcome: Summarize the major theories of and approaches to leadership Skill: Application 59) Which of the current business characteristics has brought the rise of virtual leadership? A) more communication within the organization B) little communication among employees C) more face-to-face communication within the organization D) less face-to-face communication within the organization E) little communication between employees and management Answer: D Explanation: D) That is, telecommuting. Page Ref: 230 Difficulty: Easy Objective: 9.7 Learning Outcome: Summarize the major theories of and approaches to leadership Skill: Concept 60) Which of the following is the name for choosing one alternative from among several options? A) decision making B) assessment C) evaluation D) discernment E) arbitration Answer: A Explanation: A) This is the definition of decision making. Page Ref: 231 Difficulty: Easy Objective: 9.8 Learning Outcome: Summarize the major theories of and approaches to leadership Skill: Concept 61) What is the first step in rational decision making? A) understanding who will make the decision B) recognizing that a decision is necessary C) identifying alternatives to the problem D) choosing an alternative E) implementing the alternative Answer: B Explanation: B) The first step is figuring out that a first step is needed. Page Ref: 232 Difficulty: Easy Objective: 9.8 Learning Outcome: Summarize the major theories of and approaches to leadership Skill: Concept 62) What is the definition of a coalition? A) an informal group formed to achieve a common goal B) an institution incorporated in order to achieve a common goal C) a pro-business political party D) the network of management within a firm E) the network of employees within a firm Answer: A Explanation: A) All of the other answers are possible specific examples of a coalition, but not its overall definition. Page Ref: 234 Difficulty: Easy Objective: 9.8 Learning Outcome: Summarize the major theories of and approaches to leadership Skill: Concept 63) Managers at Adidas had a hunch that their sponsorship deal with the Yankees would be a good one. Their feelings were based on what concept? A) escalation of commitment B) risk propensity C) intuition D) perception E) rational decision making Answer: C Explanation: C) Intuition is based on feeling the answer, rather than analyzing the situation. Page Ref: 235 Difficulty: Easy AACSB: Reflective thinking skills Objective: 9.8 Learning Outcome: Summarize the major theories of and approaches to leadership Skill: Application 64) Which of the following reflects the extent to which a leader is willing to gamble when making a decision? A) escalation of commitment B) overconfidence C) caution D) intuition E) risk propensity Answer: E Explanation: E) The organization's culture is a prime ingredient in fostering different levels of risk propensity. Page Ref: 235 Difficulty: Easy Objective: 9.8 Learning Outcome: Summarize the major theories of and approaches to leadership Skill: Concept 65) Anne bought stock in ABC Company in hopes of making a profit. Although the stock's price steadily continued to drop, she refused to sell her stock. What does this illustrate? A) risk propensity B) escalation of commitment C) intuition D) perception E) caution Answer: B Explanation: B) This is an example of committing to a decision and sticking it out. Page Ref: 235 Difficulty: Moderate AACSB: Reflective thinking skills Objective: 9.8 Learning Outcome: Summarize the major theories of and approaches to leadership Skill: Application 66) Which of the following do organizations need in order to be effective? A) more leadership than management B) more management than leadership C) both leadership and management D) either leadership or management E) neither leadership nor management Answer: C Explanation: C) Leadership is necessary to create and direct change and to help the organization get through tough times; management is necessary to achieve coordination and systematic results and to handle administrative activities during times of stability. Page Ref: 223 Difficulty: Moderate Objective: 9.1 Learning Outcome: Summarize the major theories of and approaches to leadership Skill: Concept 67) Which of the following statements is the MOST accurate? A) Effective managers lead for stability whereas visionary leaders lead for change. B) Effective leaders lead for stability whereas visionary managers lead for change. C) Visionary managers lead for stability whereas effective leaders lead for change. D) Strategic leaders lead for stability whereas transforming leaders lead for change. E) Strategic managers lead for stability whereas transforming managers lead for change. Answer: A Explanation: A) Management in conjunction with leadership can help achieve planned change, and leadership in conjunction with management can keep the organization aligned with its environment. Page Ref: 222 Difficulty: Moderate AACSB: Reflective thinking skills Objective: 9.1 Learning Outcome: Summarize the major theories of and approaches to leadership Skill: Concept 68) In interviews with two leadership applicants, Ben tried to determine each person's knowledge of the industry, level of energy, and level of self-esteem. With which of the following concepts of leadership is Ben most concerned here? A) charismatic leadership B) behavioral leadership C) situational leadership D) trait-based leadership E) transformational leadership Answer: D Explanation: D) With trait theory, some people believe that intelligence, drive, motivation, honesty, integrity, self-confidence, cognitive ability, knowledge of the business, charisma, biological factors, and other variables play a role in one's leadership style. Page Ref: 223 Difficulty: Moderate AACSB: Reflective thinking skills Objective: 9.2 Learning Outcome: Summarize the major theories of and approaches to leadership Skill: Application 69) Which of the following statements best describes the assumptions made by researchers of the behavioral approach to leadership? A) Behaviors of effective leaders differ in various situations. B) Behaviors of effective leaders are the same in all situations. C) Behaviors of effective leaders reflect the proper traits for effective leadership. D) Behaviors of effective leaders differ as reflected by different traits for effective leadership. E) Behaviors of effective leaders reflect the proper disposition for effective leadership. Answer: B Explanation: B) Behavioral researchers assumed that the behaviors of effective leaders differed somehow from the behaviors of less effective leaders, and that the behaviors of effective leaders would be the same across all situations. Page Ref: 224 Difficulty: Moderate Objective: 9.2 Learning Outcome: Summarize the major theories of and approaches to leadership Skill: Concept 70) A company leader who often uses the phrase "that was then and this is now" to explain his decisions most likely follows which model of leadership? A) situational leadership B) trait-based leadership C) tranformational leadership D) transactional leadership E) charismatic leadership Answer: A Explanation: A) That leaders sometimes focus on employee-focused behaviors and sometimes on task-focused behaviors led to situational theories. Page Ref: 224 Difficulty: Moderate Objective: 9.3 Learning Outcome: Summarize the major theories of and approaches to leadership Skill: Application 71) According to the situational approach to leadership, the extent to which a leader or subordinate is involved in decision making depends on characteristics of which aspects of an organization? A) the leader, the followers, and the situation B) the leader, the structure of the organization, and the situation C) the leader, the resources at hand, and the situation D) the followers, the resources at hand, and the situation E) the resources, the structure of the organization, and the situation Answer: A Explanation: A) This approach was first proposed as a continuum of leadership behavior, with one extreme of having the leader make decisions alone to the other extreme of having employees make decisions with only minimal guidance from the leader; each point on the continuum is influenced by characteristics of the leader, his or her subordinates, and the situation. Page Ref: 224 Difficulty: Moderate Objective: 9.3 Learning Outcome: Summarize the major theories of and approaches to leadership Skill: Concept 72) What are group effectiveness, worker disposition, and the type of organization examples of? A) cross-cultural traits that may affect a leader's decision making B) behavioral issues that may affect a leader's decision making C) strategic advantages that may affect a leader's decision making D) leadership obstructions that may affect a leader's decision making E) situational characteristics that may affect a leader's decision making Answer: E Explanation: E) These are the things that make a decision-making situation unique and thus affect the outcome of a decision. Page Ref: 224 Difficulty: Moderate Objective: 9.3 Learning Outcome: Summarize the major theories of and approaches to leadership Skill: Concept 73) Which of the following is an example of an approach to leadership as seen through the eyes of followers? A) situational leadership B) ethical leadership C) behavioral leadership D) transformational leadership E) trait-based leadership Answer: D Explanation: D) Both transformational leadership and charismatic leadership are approaches to leadership as seen through the eyes of followers. Page Ref: 225 Difficulty: Moderate Objective: 9.4 Learning Outcome: Summarize the major theories of and approaches to leadership Skill: Concept 74) Which of the following may describe charismatic leadership? A) an approach to leadership emphasizing the perspectives of the leader B) an approach to leadership emphasizing the perspectives of the followers C) an approach to leadership emphasizing the perspectives of the clientele D) an approach to leadership emphasizing the perspectives of the managers E) an approach to leadership emphasizing the perspectives of all parties in the organization Answer: B Explanation: B) Both transformational and charismatic leadership are approaches to leadership as seen through the eyes of followers. Page Ref: 226 Difficulty: Moderate AACSB: Reflective thinking skills Objective: 9.4 Learning Outcome: Summarize the major theories of and approaches to leadership Skill: Concept 75) GDB Technologies has had lackluster performance for the past few years. Choi is hired in an attempt to revivify the company's prospects and give it new direction away from its current situation. Which of the following best describes Choi's leadership role? A) transformational B) charismatic C) transactional D) situational E) cross-cultural Answer: A Explanation: A) Transformational leadership involves using a set of abilities that allows a leader to recognize the need for change, to create a vision to guide that change, and to execute the change. Page Ref: 225 Difficulty: Moderate AACSB: Reflective thinking skills Objective: 9.4 Learning Outcome: Summarize the major theories of and approaches to leadership Skill: Application 76) Carol took over a stagnant firm three years ago. At that time, she clearly laid out her vision of where the firm should go and since that time has worked to energize the employees of the firm toward these goals. Which of the following best describes Carol's leadership role? A) transformational B) charismatic C) transactional D) situational E) cross-cultural Answer: B Explanation: B) This is typical charismatic leadership at work. Page Ref: 226 Difficulty: Moderate AACSB: Reflective thinking skills Objective: 9.4 Learning Outcome: Summarize the major theories of and approaches to leadership Skill: Application 77) At Nordic Enterprises, Johann is most effective in carrying out the management of daily tasks that must be performed. Which of the following best describes Johann's leadership role? A) charismatic B) transactional C) transformational D) behavorial E) situational Answer: B Explanation: B) Transactional leadership is essentially the same as management in that it involves routine, regimented activities. Page Ref: 225 Difficulty: Moderate AACSB: Reflective thinking skills Objective: 9.4 Learning Outcome: Summarize the major theories of and approaches to leadership Skill: Application 78) Which of following describes the activities most involved with transactional leadership? A) the implementation of a corporate vision B) the implementation of organizational transformations C) the implementation of routine activities D) the implementation of employee incentives E) the implementation of situational problem-solving Answer: C Explanation: C) Transactional leadership is essentially the same as management in that it involves routine, regimented activities. Page Ref: 225 Difficulty: Moderate Objective: 9.4 Learning Outcome: Summarize the major theories of and approaches to leadership Skill: Concept 79) Which of the following is a fundamental element of charismatic leadership? A) planning B) controlling C) analyzing D) influencing E) organizing Answer: D Explanation: D) Charismatic leadership is a type of influence based on the leader's personal charm. A highly charismatic supervisor will be more successful in influencing subordinate behavior than a supervisor who lacks charisma. Page Ref: 226 Difficulty: Moderate AACSB: Reflective thinking skills Objective: 9.4 Learning Outcome: Summarize the major theories of and approaches to leadership Skill: Concept 80) Which statement is true regarding charismatic leadership? A) Charismatic leadership ideas are unpopular among contemporary managers. B) Many studies have successfully assessed the impact of charismatic leadership. C) Charismatic leadership is recognized as the most important approach to leadership. D) Dependence on charismatic leadership can raise serious ethical concerns. E) Taking over a leadership role from a charismatic leader is often easier than following a leader lacking in charisma. Answer: D Explanation: D) Charismatic leaders can inspire such blind faith in their followers that they may engage in inappropriate, unethical, or even illegal behaviors just because the leader instructed them to do so. Page Ref: 226 Difficulty: Moderate Objective: 9.4 Learning Outcome: Summarize the major theories of and approaches to leadership Skill: Concept 81) Steven learned that the cohesiveness among his employees was so strong that there was nothing he could do to change things. Which of the following best describes this cohesiveness? A) leadership neutralizer B) leadership substitute C) leadership mandate D) leadership amplifier E) leadership incentive Answer: A Explanation: A) A leader may be powerless to improve things because a group is so cohesive; leadership neutralizers render certain leadership behaviors ineffective regardless of the extent to which they are present. Page Ref: 227 Difficulty: Moderate AACSB: Reflective thinking skills Objective: 9.5 Learning Outcome: Summarize the major theories of and approaches to leadership Skill: Application 82) According to current theories of gender roles in leadership, which of the following best describes the differences in decision making? A) Males tend to be more democratic than females in making decisions. B) Males tend to be more critical than females in making decisions. C) Males tend to be more autocratic than females in making decisions. D) Males tend to be more rational than females in making decisions. E) Males tend to be more charismatic than females in making decisions. Answer: C Explanation: C) One difference that does seem to arise in some cases, according to the text, is that women have a tendency to be slightly more democratic in making decisions, whereas men have a tendency to be more autocratic. Page Ref: 229 Difficulty: Moderate AACSB: Multicultural and diversity understanding Objective: 9.6 Learning Outcome: Summarize the major theories of and approaches to leadership Skill: Concept 83) As ABC Enterprises has grown during the past few years, more and more workers have been hired from the local Ethiopian population. A number of incidents at the firm have arisen during this time regarding certain customs prevalent among these new workers. Which of the following forms of leadership does the management of this firm need to utilize in order to most harmonize this situation? A) autocratic leadership B) democratic leadership C) cross-cultural leadership D) charismatic leadership E) mentoring leadership Answer: C Explanation: C) Some of these other answers may be local solutions. But the wider situation can only be solved by taking the cross-cultural effects into account for any leadership decisions. Page Ref: 230 Difficulty: Moderate AACSB: Multicultural and diversity understanding Objective: 9.6 Learning Outcome: Summarize the major theories of and approaches to leadership Skill: Application 84) Which states the correct definition of collectivism? A) a greater focus on group characteristics than on individual characteristics B) a greater focus on the group than on the individual C) a greater focus on the individual within a group than on the individual without a group D) a greater focus on individuals who are most successful E) a greater focus on groups that have been operative for the longest time Answer: B Explanation: B) All of the other answers are parts of collectivism, but not the whole concept. Page Ref: 229 Difficulty: Moderate Objective: 9.6 Learning Outcome: Summarize the major theories of and approaches to leadership Skill: Concept 85) Which states the correct definition of individualism? A) a greater focus on individual characteristic than on cultural characteristics B) a greater focus on the individual than on the group C) a greater focus on the individual within a group than on the individual without a group D) a greater focus on individuals who have been present for the longest time E) a greater focus on individuals who are in leadership roles Answer: B Explanation: B) All of the other answers are parts of individualism, but not the whole concept. Page Ref: 230 Difficulty: Moderate AACSB: Reflective thinking skills Objective: 9.6 Learning Outcome: Summarize the major theories of and approaches to leadership Skill: Concept 86) In exercising strategic leadership, what is a leader most likely to do? A) focus on company efficiency B) focus on hiring the right people C) align the company with its environment D) avoid environmental impact on the firm E) engage the firm with community organizations Answer: C Explanation: C) Strategic leadership is a leader's ability to understand the complexities of both the firm and its environment and to lead change in the firm so as to achieve and maintain a superior alignment between the organization and its environment. Page Ref: 230 Difficulty: Moderate AACSB: Reflective thinking skills Objective: 9.7 Learning Outcome: Summarize the major theories of and approaches to leadership Skill: Concept 87) An inward belief about something, often without conscious consideration, is often referred to as what? A) intuition B) risk taking C) commitment D) discernment E) rationality Answer: A Explanation: A) Intuition is an innate belief about something, without conscious consideration; managers sometimes decide to do something because it "feels right." Such an inner sense may help managers make an occasional decision without going through a full-blown rational sequence of steps. Page Ref: 235 Difficulty: Moderate Objective: 9.8 Learning Outcome: Summarize the major theories of and approaches to leadership Skill: Concept 88) Bill decides to study a foreign language. All of his friends said it will be easy. After the third week, he realizes the language is very difficult for him to grasp. He continues his studies, hoping that it will eventually get easier. What does this illustrate? A) rational decision making B) risk-taking decision making C) escalation of commitment D) intuitive decision making E) situational commitment Answer: C Explanation: C) With escalation of commitment, decision makers make decisions and then become so committed to the course of action suggested by that decision that they stay with it, even when it appears to have been wrong. Page Ref: 235 Difficulty: Moderate AACSB: Reflective thinking skills Objective: 9.8 Learning Outcome: Summarize the major theories of and approaches to leadership Skill: Application 89) Leadership researchers in the late 1940s believed that leaders should engage in which of the following behaviors? A) mostly employee-focused leader behavior B) mostly task-focused leader behavior C) both employee-focused leader behavior and task-focused leader behavior in equal amounts D) a variable mixture of employee-focused leader behavior and task-focused behavior depending on the task E) a variable mixture of employee-focused leader behavior and task-focused behavior depending on the employees Answer: C Explanation: C) In the late 1940s, most researchers began to shift away from the trait approach and to look at leadership as a set of actual behaviors. Page Ref: 224 Difficulty: Difficult Objective: 9.1 Learning Outcome: Summarize the major theories of and approaches to leadership Skill: Concept 90) Which statement is FALSE regarding gender and leadership? A) Theories regarding gender and leadership are not well-proven. B) Most leadership theories and studies have focused on males. C) There is much still to be learned about the dynamics of gender and leadership. D) There is a growing number of woman leaders in the business world today. E) Men tend to be more democratic than women in decision making. Answer: E Explanation: E) Most theories and studies have focused on males; further, more work is needed in order to better understand the dynamics of gender and leadership. Page Ref: 229 Difficulty: Difficult AACSB: Multicultural and diversity understanding Objective: 9.6 Learning Outcome: Summarize the major theories of and approaches to leadership Skill: Concept 91) In selecting an alternative when making a decision, what should decision makers keep in mind? A) Finding multiple acceptable alternatives is neither possible nor efficient. B) Finding multiple acceptable alternatives should not take very long. C) Selecting one alternative and rejecting all others will be the decision maker's main objective. D) Selecting one alternative and rejecting all others might not be necessary. E) Selecting one alternative and rejecting all others should be a straightforward process. Answer: D Explanation: D) Finding multiple acceptable alternatives may be possible; selecting one alternative and rejecting all the others might not be necessary. Further, selecting alternatives can be a long and difficult task. Page Ref: 233 Difficulty: Moderate Objective: 9.8 Learning Outcome: Summarize the major theories of and approaches to leadership Skill: Concept 92) Which of the following dispositions is typical for leaders with a high degree of risk propensity? A) reliance on intuition B) reliance on rational decisions C) aversion against use of intuition D) aversion against rational decisions E) aversion against use of both intuition and rational decisions Answer: A Explanation: A) Managers with a high degree of risk propensity tend to rely heavily on intuition, reach decisions quickly, and often risk big investments on their decisions. Page Ref: 235 Difficulty: Difficult Objective: 9.8 Learning Outcome: Summarize the major theories of and approaches to leadership Skill: Concept 93) What is a prime ingredient in fostering different levels of risk propensity within an organization? A) organizational culture B) profitability level C) gender differences D) cross-cultural interactions E) company size Answer: A Explanation: A) Risk propensity is the extent to which a decision maker is willing to gamble when making a decision; since some organizations' cultures are more conservative than others, organizational culture is a prime ingredient in fostering different levels of risk propensity. Page Ref: 235 Difficulty: Difficult AACSB: Reflective thinking skills Objective: 9.8 Learning Outcome: Summarize the major theories of and approaches to leadership Skill: Synthesis Chapter 10 Human Resource Management and Labor 24) What is another name given to human resource managers? A) financial managers B) personnel managers C) production managers D) accounting managers E) business development managers Answer: B Explanation: B) Human resource or personnel managers have a substantial impact on a firm's performance as a business. Page Ref: 245 Difficulty: Easy Objective: 10.1 Learning Outcome: Summarize the functions carried out by human resources Skill: Concept 25) Which of the following outlines the duties of a job, its working conditions, and the tools, materials, and equipment used to perform it? A) job advertisement B) job specification C) job description D) job analysis E) job report Answer: C Explanation: C) The job description is part of the job analysis process. Page Ref: 245 Difficulty: Easy Objective: 10.1 Learning Outcome: Summarize the functions carried out by human resources Skill: Concept 26) Which of the following describes the skills, abilities, and qualifications required for a job? A) staffing survey B) advertising budget C) job specification D) employee analysis E) replacement chart Answer: C Explanation: C) A job specification lists the attributes an employee needs in order to perform the job effectively. Page Ref: 245 Difficulty: Easy Objective: 10.1 Learning Outcome: Summarize the functions carried out by human resources Skill: Concept 27) Which of the following contains information on each employee's education, skills, work experiences, and career aspirations? A) managerial grid B) employee information system C) job analysis D) job specification E) labor forecast Answer: B Explanation: B) This system can quickly locate every employee who is qualified to fill a position. Page Ref: 246 Difficulty: Easy Objective: 10.1 Learning Outcome: Summarize the functions carried out by human resources Skill: Concept 28) Jean's paycheck is calculated from the number of hours worked during the week times her hourly rate. What type of system is used to pay Jean? A) gainsharing B) salary C) profit sharing D) wage E) incentive Answer: D Explanation: D) A wage is the dollar amounts paid to employees for their labor. Page Ref: 250 Difficulty: Easy AACSB: Reflective thinking skills Objective: 10.4 Learning Outcome: Summarize the functions carried out by human resources Skill: Application 29) Which of the following links salary increases to performance levels in nonsales jobs? A) bonuses B) collective bargaining C) merit salary systems D) cafeteria benefit plans E) workers' compensation insurance Answer: C Explanation: C) An example is the rewarding of executives with stock options. Page Ref: 251 Difficulty: Easy Objective: 10.4 Learning Outcome: Summarize the functions carried out by human resources Skill: Concept 30) Which plan refers to compensation other than wages and salaries and other incentives offered by a firm to its workers? A) profit-sharing B) gainsharing C) benefits D) pay-for-knowledge E) variable Answer: C Explanation: C) Benefits account for an increasing percentage of most compensation budgets. Page Ref: 251 Difficulty: Easy Objective: 10.4 Learning Outcome: Summarize the functions carried out by human resources Skill: Concept 31) What is the name given to an employee who is hired on something other than a permanent or full-time basis? A) knowledge expert B) contingent worker C) technical employee D) supervisory manager E) managing director Answer: B Explanation: B) Contingent workers include independent contractors, on-call workers, and leased employees. Page Ref: 257 Difficulty: Easy Objective: 10.4 Learning Outcome: Summarize the functions carried out by human resources Skill: Concept 32) What is the name given to a group of individuals, working together to achieve shared job-related goals such as higher pay, shorter working hours, greater benefits, or better working conditions? A) management organization B) company hierarchy C) labor union D) external organization E) permanent employees Answer: C Explanation: C) The percentage of American workers belonging to a labor union has been declining over the last 15 years. Page Ref: 257 Difficulty: Easy Objective: 10.6 Learning Outcome: Summarize the functions carried out by human resources Skill: Concept 33) What is the process of dealing with employees who are represented by a union? A) labor relations B) human resource management C) negotiation D) arbitration E) contingency recruitment Answer: A Explanation: A) A company executive needs special skills in dealing and negotiating with labor unions. Page Ref: 257 Difficulty: Easy Objective: 10.7 Learning Outcome: Summarize the functions carried out by human resources Skill: Concept 34) What is the process by which union leaders and managers negotiate common terms and conditions of employment for the workers represented by unions? A) striking B) picketing C) informational gathering D) collective bargaining E) compensation planning Answer: D Explanation: D) It is the process in which a labor contract can be drafted for union-represented workers. Page Ref: 258 Difficulty: Easy Objective: 10.8 Learning Outcome: Summarize the functions carried out by human resources Skill: Concept 35) Which of the following occurs when union members agree not to buy the products of a targeted employer? A) lockout B) boycott C) mediation D) arbitration E) slowdown Answer: B Explanation: B) Workers may also urge consumers to boycott the firm's products. Page Ref: 260 Difficulty: Easy Objective: 10.8 Learning Outcome: Summarize the functions carried out by human resources Skill: Concept 36) What term describes the situation when workers perform jobs at a less efficient pace? A) lockout B) boycott C) slowdown D) walkout E) strike Answer: C Explanation: C) A variation on the slowdown is a sickout, when large numbers of workers call in sick. Page Ref: 260 Difficulty: Easy Objective: 10.8 Learning Outcome: Summarize the functions carried out by human resources Skill: Concept 37) If negotiations fail, what can a company choose to use to prevent union employees from coming to work? A) wildcat strike B) lockout C) strikebreaker D) economic strike E) bailout Answer: B Explanation: B) Lockouts are illegal if they are used as offensive weapons to give management a bargaining advantage. Page Ref: 261 Difficulty: Easy Objective: 10.8 Learning Outcome: Summarize the functions carried out by human resources Skill: Concept 38) What occurs when a neutral third party dictates a settlement between two sides that have agreed to submit to outside judgment? A) mediation B) arbitration C) strikebreaker D) impasse E) picketing Answer: B Explanation: B) In arbitration, the neutral third party (the arbitrator) dictates a settlement between the two sides, which have agreed to submit to outside judgment. Page Ref: 261 Difficulty: Easy Objective: 10.8 Learning Outcome: Summarize the functions carried out by human resources Skill: Concept 39) Which of the following is considered the starting point of effective human resource management? A) external staffing B) recruitment C) planning D) financing E) compensation Answer: C Explanation: C) The starting point for effective human resource management is human resource planning, which involves job analysis and forecasting the demand for and supply of labor. Page Ref: 245 Difficulty: Moderate Objective: 10.1 Learning Outcome: Summarize the functions carried out by human resources Skill: Concept 40) What is the purpose of a skills inventory? A) to plan and identify people for transfer or promotion B) to list key managerial positions C) to describe the specifications for a job D) to assess trends in previous human resource usage E) to hire people from outside the organization Answer: A Explanation: A) Some organizations also use employee information systems. Page Ref: 246 Difficulty: Moderate Objective: 10.1 Learning Outcome: Summarize the functions carried out by human resources Skill: Concept 41) Plants Galore is a business selling a range of indoor plants. In the past there has been a problem with plants being stolen by staff. Which of the following represents the biggest limitation in reference checks as a reliable way of assessing the value of prospective employees? A) Reference checks are illegal in a number of states. B) Reference checks obtained from previous employers are out of date. C) Some previous employers may no longer be in business. D) Some reference checks can be time consuming. E) Reference checks tend to only tap positive recommendations. Answer: E Explanation: E) Reference checks have been shown to have limited value because individuals tend to provide the names of people who will give them positive recommendations. Page Ref: 248 Difficulty: Moderate AACSB: Reflective thinking skills Objective: 10.2 Learning Outcome: Summarize the functions carried out by human resources Skill: Application 42) Which of the following types of training occurs while the employee is at his or her place of work? A) on-the-job training B) vestibule training C) off-site training D) incentive training E) remote training Answer: A Explanation: A) On-the-job training occurs while an employee is at work; most of this training is informal. Page Ref: 249 Difficulty: Easy Objective: 10.3 Learning Outcome: Summarize the functions carried out by human resources Skill: Concept 43) Fletcher is meeting with each of his subordinates to discuss their performance over the past year. What process is he involved in? A) job description B) labor negotiations C) vestibule assessment D) performance appraisal E) financial remuneration Answer: D Explanation: D) In some small companies, performance appraisal takes place when the owner tells an employee that he or she is "doing a good job." In larger companies, performance appraisals are designed to show more precisely how well workers are doing their jobs. Page Ref: 249 Difficulty: Moderate Objective: 10.3 Learning Outcome: Discuss the factors that affect motivation and behavior in the workplace Skill: Application 44) What is the total package that a firm offers employees in return for their labor? A) wage B) salary C) benefits package D) compensation system E) retention plan Answer: D Explanation: D) In addition, the compensation package is a major factor in retaining skilled workers. Page Ref: 249 Difficulty: Moderate Objective: 10.3 Learning Outcome: Summarize the functions carried out by human resources Skill: Concept 45) What do top executives often receive as incentives? A) merit salary systems B) workers' compensation insurance C) salaries D) stock options E) wages Answer: D Explanation: D) Though stock options are common incentives, such options have been at the center of some of the recent accounting scandals plaguing some businesses today. Page Ref: 251 Difficulty: Moderate Objective: 10.4 Learning Outcome: Summarize the functions carried out by human resources Skill: Concept 46) A Web design company, Gen Max, is expanding rapidly and needs to hire middle managers in a climate where first to market is an imperative. Which incentive plan for these employees would best help the firm achieve its first to market goal? A) variable pay B) merit raises C) profit-sharing D) gainsharing E) pay-for-knowledge Answer: A Explanation: A) Variable pay has proven to be a high motivator for work performance. The popularity of variable pay programs has increased in the United States over the last decade. Page Ref: 251 Difficulty: Moderate AACSB: Reflective thinking skills Objective: 10.4 Learning Outcome: Discuss the factors that affect motivation and behavior in the workplace Skill: Application 47) Which term refers to the situation in which middle managers are rewarded for especially productive output, such as for producing earnings that significantly exceed the cost of bonuses? A) wage inflation B) a merit salary system C) a gainsharing plan D) pay for performance E) salary sacrifice Answer: D Explanation: D) Variable pay goes to middle managers on the basis of companywide performance, business unit performance, personal record, or all three factors. Page Ref: 251 Difficulty: Moderate Objective: 10.4 Learning Outcome: Summarize the functions carried out by human resources Skill: Concept 48) Ace Products' profits for fiscal year 2005 have exceeded estimates by over 2 million dollars. Which plan can Ace use to distribute part of this excess to employees? A) gainsharing plan B) profit-sharing plan C) cafeteria plan D) insurance plan E) salary-sharing plan Answer: B Explanation: B) Under profit-sharing plans, profits earned above a certain level are distributed to employees. Page Ref: 251 Difficulty: Moderate Objective: 10.4 Learning Outcome: Summarize the functions carried out by human resources Skill: Application 49) Which of the following companywide incentive plans distribute bonuses to employees when a company's costs are reduced through greater work efficiency? A) wage setting plans B) pay for performance plans C) profit-sharing plans D) gainsharing plans E) pay-for-knowledge plans Answer: D Explanation: D) Gainsharing plans distribute bonuses to employees when a company's costs are reduced through greater work efficiency. Page Ref: 251 Difficulty: Moderate Objective: 10.4 Learning Outcome: Summarize the functions carried out by human resources Skill: Concept 50) At Ace Products, employees receive a fixed dollar amount for benefits, but they may choose between various health insurance, life insurance, disability insurance, and educational benefits. What is the name of this plan? A) gainsharing B) profit-sharing C) cafeteria benefits D) equal employment E) retention benefits Answer: C Explanation: C) With a cafeteria benefit plan, a certain dollar amount of benefits per employee is set aside so that each employee can choose from a variety of alternatives. Page Ref: 252 Difficulty: Moderate AACSB: Reflective thinking skills Objective: 10.4 Learning Outcome: Summarize the functions carried out by human resources Skill: Application 51) What was the fundamental reason why affirmative action was established? A) to give certain people an unfair advantage over others in the work place B) to require diversity committees in all organizations with 50 or more employees C) to combat prejudices against baby boomers still in the workforce D) to prevent exclusion by racial and gender discrimination E) to increase global marketing for products and services Answer: D Explanation: D) Affirmative action required that organizations employ a certain amount of Hispanics, African Americans, and women to combat discrimination that those groups experience. Page Ref: 253 Difficulty: Moderate Objective: 10.5 Learning Outcome: Summarize the functions carried out by human resources Skill: Concept 52) Which legal concept holds that both employer and employee have the mutual right to terminate an employment relationship any time for any reason and without advance notice to the other? A) quid pro quo B) affirmative action C) employment at will D) mutual consent E) equal employment opportunity Answer: C Explanation: C) Specifically, employment at will holds that an organization employs an individual at its own will and can, therefore, terminate that employee at any time for any reason. Page Ref: 255 Difficulty: Moderate Objective: 10.5 Learning Outcome: Summarize the functions carried out by human resources Skill: Concept 53) Mary is a supervisor at her company. Under which legal concept is it legal for Mary to discharge all her employees without advance notice? A) quid pro quo B) affirmative action C) employment at will D) joint agreement E) arbitration Answer: C Explanation: C) This concept holds that both employer and employee have the mutual right to terminate an employment relationship anytime for any reason. Page Ref: 255 Difficulty: Moderate Objective: 10.5 Learning Outcome: Summarize the functions carried out by human resources Skill: Application 54) Workers such as computer scientists, engineers, and game developers who typically require extensive and highly specialize training are known as what? A) technical workers B) knowledge workers C) contingent workers D) labor relation workers E) union workers Answer: B Explanation: B) Employees who add value because of what they know are usually called knowledge workers, and the skill with which they are managed is a major factor in determining which firms will be successful. Page Ref: 256 Difficulty: Moderate Objective: 10.6 Learning Outcome: Summarize the functions carried out by human resources Skill: Concept 55) Which term refers to employees hired to supplement a firm's permanent workforce? A) union workers B) contingent workers C) knowledge workers D) flextime workers E) remote workers Answer: B Explanation: B) A contingent worker is a person who works for an organization on something other than a permanent or full-time basis. Page Ref: 257 Difficulty: Moderate Objective: 10.6 Learning Outcome: Summarize the functions carried out by human resources Skill: Concept 56) What are independent contractors, on-call workers, temporary employees, contract employees, and leased employees all examples of? A) union workers B) contingent workers C) migratory workers D) ancillary workers E) international workers Answer: B Explanation: B) These are categories of contingent workers, all of whom work for an organization on something other than a permanent or full-time basis. Page Ref: 257 Difficulty: Moderate Objective: 10.6 Learning Outcome: Summarize the functions carried out by human resources Skill: Concept 57) What employment issue would a labor union be best equipped to resolve? A) Aarif is Muslim and requires working an alternate schedule during Ramadan, but other employees still have to work the standard 8:00 to 5:00 while he is observing the month for religious purposes. B) Celia is a Latina woman who feels she did not get hired because of her ethnicity. C) Gary has a salary that is $8,000 higher than Sarah's. Sarah and Gary are the same age with similar professional experiences. The only difference is that Sarah has a Master's degree and Gary has a Bachelor's degree. D) Daniel is of retirement age, but wants to continue to work in his organization. E) Sean is interested in applying for a position that requests applicants have higher credentials than what he has. Answer: C Explanation: C) A labor union would help fight for equal pay for women. Page Ref: 258 Difficulty: Moderate AACSB: Reflective thinking skills Objective: 10.7 Learning Outcome: Summarize the functions carried out by human resources Skill: Application 58) When does the collective bargaining process begin? A) when the contract is ratified by the members B) when the bargaining zone is identified C) when the union is recognized as the official bargaining agent D) when union and management representatives agree to contract terms E) when the employer establishes a maximum limit Answer: C Explanation: C) When a union has been legally certified, it assumes the role of official bargaining agent for workers; collective bargaining then begins. Page Ref: 259 Difficulty: Moderate Objective: 10.8 Learning Outcome: Summarize the functions carried out by human resources Skill: Concept 59) The insurance company Montas is negotiating with its labor union on varying aspects of the compensation package including remuneration and benefits. Which strategy would be most appropriate for the CEO to use in reaching agreement on the contract terms for the amount of staff vacation? A) bargaining with union representatives for nonfinancial benefits B) expanding the use of tough bargaining tactics C) gathering information about staff expectations D) brokering the prospect of arbitration E) setting up mediation processes Answer: C Explanation: C) It would be important for the information to be gathered on the expectations for the maximum and minimum limits. Page Ref: 259 Difficulty: Moderate Objective: 10.8 Learning Outcome: Summarize the functions carried out by human resources Skill: Application 60) Which term refers to a labor action in which employees temporarily walk off the job and refuse to work? A) strike B) lockout C) boycott D) mediation E) bargaining Answer: A Explanation: A) Most strikes in the United States are economic strikes, triggered by stalemates over mandatory bargaining items, including such issues as working hours. Page Ref: 260 Difficulty: Easy Objective: 10.8 Learning Outcome: Summarize the functions carried out by human resources Skill: Concept 61) Who are temporary or permanent replacements for striking workers? A) strikebreakers B) union busters C) union breakers D) contingent workers E) labor relation teams Answer: A Explanation: A) However, the law forbids the permanent replacement of workers who strike because of unfair practices. In some cases, an employer can obtain legal injunctions that either prohibit workers from striking or prohibit a union from interfering with its efforts to use replacement workers. Page Ref: 261 Difficulty: Moderate Objective: 10.8 Learning Outcome: Summarize the functions carried out by human resources Skill: Concept 62) Gretta is observing and interviewing several database administrators to determine the duties and responsibilities associated with their job as well as the knowledge, skills, and abilities that someone must have to perform the job. Which HR function is Gretta performing? A) staffing B) promotion C) job analysis D) training E) retention Answer: C Explanation: C) Job analysis is a systematic analysis of jobs within an organization, resulting in the job description and the job specification. Page Ref: 245 Difficulty: Difficult AACSB: Reflective thinking skills Objective: 10.1 Learning Outcome: Summarize the functions carried out by human resources Skill: Application 63) Where is a skills inventory often maintained? A) replacement chart B) employee information system C) internal supply forecast D) work flow chart E) job description spreadsheet Answer: B Explanation: B) Employee information systems are usually computerized; they contain information on each employee's education, skills, experience, and aspirations. Page Ref: 246 Difficulty: Moderate Objective: 10.1 Learning Outcome: Summarize the functions carried out by human resources Skill: Concept 64) Which selection technique is most likely to introduce bias? A) interview B) application forms C) aptitude tests D) drug testing E) polygraph tests Answer: A Explanation: A) The interview is sometimes a poor predictor of job success; biases inherent in the way people perceive and judge others on first meeting affect subsequent evaluations. Page Ref: 247 Difficulty: Moderate Objective: 10.2 Learning Outcome: Summarize the functions carried out by human resources Skill: Concept 65) Health Connect is a start-up software development company specializing in the design of imaging for medical practitioners. They are at the stage where they need to recruit a team of technical employees. Which of the following is the BEST reason for NOT relying solely on interviews when recruiting the technical staff? A) Interviews tend to have a poor record as a predictor of successful job performance. B) Technical staff tend to be confident people, and they all perform equally well in the interview situation. C) Start-up companies are not in the position to afford extra recruiting procedures. D) Interviews used in conjunction with polygraphs can cause job candidates to confess to crimes. E) Technical people need to be able to assess situations quickly and accurately. Answer: A Explanation: A) The interview is a widely used selection device but can be affected by the bias of the interviewer and first meeting impressions. A practical task can be effective in showing how the prospective employee would undertake a real work task. Page Ref: 247 Difficulty: Difficult AACSB: Reflective thinking skills Objective: 10.2 Learning Outcome: Summarize the functions carried out by human resources Skill: Application 66) When does the appraisal process begin? A) when the manager defines performance standards for an employee B) after the manager observes the employee's performance C) as soon as the employee begins working D) when the employee leaves an organization E) as soon as the employee requests direction Answer: A Explanation: A) The process begins when a manager defines performance standards for an employee; after that, the manager observes the employee's performance. Page Ref: 249 Difficulty: Difficult Objective: 10.3 Learning Outcome: Summarize the functions carried out by human resources Skill: Concept 67) A restaurant is undergoing a change in cuisine and style of service. It is important that new staff are hired and existing staff retrained. Why might the HR manager recommend vestibule training as being highly effective in this situation? A) Prospective and present employees can be screened to see if they convey the restaurant's new ethos. B) Off-site training has great value for evaluating the effectiveness of performance appraisals. C) Employees will be reluctant to undertake training removed from the workplace. D) The owners can be more confident that the change to the restaurant will be popular with customers. E) Employees will see limited connections to everyday work in this type of training. Answer: A Explanation: A) Since vestibule training takes place in simulated environments, it offers employees the chance to learn new skills or retrain in a realistic setting. Page Ref: 249 Difficulty: Difficult AACSB: Reflective thinking skills Objective: 10.3 Learning Outcome: Summarize the functions carried out by human resources Skill: Application 68) Which of the following constitutes an illegal hiring practice? A) asking someone about their availability because the position requires a 50-hour work week B) establishing a diversity committee to help diverse employees connect with each other C) asking a candidate whether or not he attends church, because it is important that candidates are available to work on Saturdays and Sundays D) keeping track of how many males and females are in the organization E) requiring an applicant to specify gender on an application Answer: C Explanation: C) The Civil Rights Act of 1964 prohibits employers from screening a candidate for religious reasons. Page Ref: 253 Difficulty: Difficult AACSB: Reflective thinking skills Objective: 10.5 Learning Outcome: Discuss the factors that affect motivation and behavior in the workplace Skill: Application 69) A large clothing retailer, Finch Line, has recently expanded but is having problems retaining experienced staff, despite paying salaries that are above industry averages. Which strategy would best support the HR manager's initiative to decrease the rate of turnover of the company's experienced staff? A) offering employees a structured path to promotion B) offering across-the-board pay increases to all employees C) making cost-of-living adjustments in employees' wages D) offering staff more money for better performance E) evaluating staff expectations as they relate to management goals Answer: A Explanation: A) Naturally, employees feel better about their companies when they believe that they are being fairly compensated; however, studies reveal that beyond a certain point, more money will not produce better performance. Page Ref: 256 Difficulty: Difficult AACSB: Reflective thinking skills Objective: 10.6 Learning Outcome: Summarize the functions carried out by human resources Skill: Application 70) Jane, a supervisor, has offered George, a subordinate, a raise and promotion in exchange for a sexual relationship. What type of harassment is this offer an example of? A) quid pro quo B) hostile work environment C) employment at will D) reverse E) financial Answer: A Explanation: A) In cases of quid pro quo harassment, the harasser offers to exchange something of value for sexual favors. Page Ref: 255 Difficulty: Difficult Objective: 10.5 Learning Outcome: Summarize the functions carried out by human resources Skill: Application 71) Under what legal concept is Bill able to quit his job at the pickle factory at any time and without notice? A) quid pro quo B) mutual consent C) joint agreement D) employment at will E) labor relation negotiation Answer: D Explanation: D) This concept holds that both employer and employee have the mutual right to terminate an employment relationship anytime for any reason, with or without advance notice to the other. Page Ref: 255 Difficulty: Moderate Objective: 10.5 Learning Outcome: Summarize the functions carried out by human resources Skill: Application 72) In considering the contributions of knowledge workers in the information age, what value do these employees add? A) their work style B) their professional network C) their skill in a specialized field D) their management skills E) their capacity to change careers Answer: C Explanation: C) Employees who add value because of what they know are usually called knowledge workers. Page Ref: 256 Difficulty: Moderate Objective: 10.6 Skill: Concept 73) The Internet start-up company QuickThink is considering whether to hire recent college grad Thomas as a full-time employee or as a contingent worker. Which of the following is NOT a likely disadvantage to the company if they hired Thomas as a contingent worker rather than a full-time one? A) Thomas may not be fully committed to the organization and its long-term goals. B) QuickThink will have to provide Thomas with ongoing and highly specialized training to prevent his skills from becoming obsolete. C) Thomas may lack much firm-specific knowledge. D) Managers will face possibly complicated decisions about how to integrate Thomas into the organization. E) Thomas may be less willing to engage in organizational citizenship behaviors. Answer: B Explanation: B) QuickThink would be more likely to have to provide ongoing and highly specialized training to its full-time knowledge workers than to a contingent worker like Thomas. Page Ref: 257 Difficulty: Moderate Objective: 10.6 Learning Outcome: Summarize the functions carried out by human resources Skill: Application Chapter 11 Marketing Processes and Consumer Behavior 23) What is the term for the process of creating, communicating, and delivering value to customers and for managing customer relationships in ways that benefit the organization and its stakeholders? A) financial planning B) controlling C) operations management D) marketing E) demographic profiling Answer: D Explanation: D) Marketing involves processes that go beyond making advertisements and promoting sales through social networks. Page Ref: 270 Difficulty: Easy Objective: 11.1 Learning Outcome: Explain the role of marketing in organizations Skill: Concept 24) What is the term for intangible products, such as time, expertise, or an activity that can be purchased? A) industrial goods B) personal goods C) risky assets D) services E) ideas Answer: D Explanation: D) Examples of services include providing professional advice or information used in making decisions. Page Ref: 271 Difficulty: Easy Objective: 11.1 Learning Outcome: Explain the role of marketing in organizations Skill: Concept 25) Which of the following identifies products that are dissimilar from those of competitors but can fulfill the same need? A) brand competition B) international competition C) the marketing mix D) product shift E) substitute products Answer: E Explanation: E) These products may appear to be very different but can fulfil the same need. Page Ref: 274 Difficulty: Easy Objective: 11.1 Learning Outcome: Explain the role of marketing in organizations Skill: Concept 26) What is the term for the type of competition based on buyers' perceptions of the benefits of products offered by particular companies? A) institutional competition B) monopolistic competition C) forced competition D) perfect competition E) brand competition Answer: E Explanation: E) An example is the competition between Internet search companies Google, Yahoo, and Bing. Page Ref: 275 Difficulty: Easy Objective: 11.1 Learning Outcome: Explain the role of marketing in organizations Skill: Concept 27) What is the term for a detailed strategy for focusing marketing efforts on consumers' needs and wants? A) marketing mix B) marketing manager C) product differentiation D) product development E) marketing plan Answer: E Explanation: E) Marketing strategy begins when a company identifies a consumer need and develops a product to meet it. Page Ref: 275 Difficulty: Easy Objective: 11.2 Learning Outcome: Explain the role of marketing in organizations Skill: Concept 28) What is the term that encompasses a good, a service, or an idea designed to fill a consumer need? A) product B) firm C) want D) utility E) condition Answer: A Explanation: A) The conceptualization and development of new products is a constant challenge for marketers. Page Ref: 275 Difficulty: Easy Objective: 11.2 Learning Outcome: Explain the role of marketing in organizations Skill: Concept 29) What do marketers call groups of people with similar wants and needs? A) price points B) target markets C) distribution points D) target points E) product positioning Answer: B Explanation: B) Target markets is a key concept for marketers. Page Ref: 277 Difficulty: Easy Objective: 11.3 Learning Outcome: Explain the role of marketing in organizations Skill: Concept 30) What type of variables are lifestyles, opinions, interests, and attitudes referred to as? A) geographic variables B) demographic variables C) psychographic variables D) product use variables E) labor market variables Answer: C Explanation: C) These variables are important to marketers because they can be changed by marketing efforts. Page Ref: 278 Difficulty: Easy Objective: 11.3 Learning Outcome: Explain the role of marketing in organizations Skill: Concept 31) What is the decision process by which customers come to purchase and consume products? A) marketing B) segmentation C) resource management D) consumer behavior E) acculturation Answer: D Explanation: D) Marketers need to be able to understand why consumers purchase one good over another. Page Ref: 279 Difficulty: Easy Objective: 11.4 Learning Outcome: Explain the role of marketing in organizations Skill: Concept 32) What type of behavior do consumers exhibit by regularly purchasing products because they are satisfied with their performance? A) psychological influences B) need recognition C) brand loyalty D) emotional motives E) consumer activism Answer: C Explanation: C) Such people are less subject to influence and are more likely to stick with preferred brands. Page Ref: 279 Difficulty: Easy Objective: 11.4 Learning Outcome: Explain the role of marketing in organizations Skill: Concept 33) What are the tangible and intangible qualities that a company builds into its products? A) features B) requirements C) benefits D) safety devices E) guarantees Answer: A Explanation: A) It is important to remember that features must also provide benefits. Page Ref: 282 Difficulty: Easy Objective: 11.6 Learning Outcome: Explain the role of marketing in organizations Skill: Concept 34) What is selling the rights to place a brand name on products such as t-shirts referred to as? A) international branding B) national branding C) private branding D) regional branding E) licensed branding Answer: E Explanation: E) Marketers exploit brands for their public appeal. Page Ref: 288 Difficulty: Easy Objective: 11.7 Learning Outcome: Explain the role of marketing in organizations Skill: Concept 35) When a wholesaler or retailer develops a brand name and has the manufacturer place that name on the product, what is that product name called? A) national brand B) private brand C) luxury brand D) licensed brand E) manufacturer brand Answer: B Explanation: B) Examples of private brands include Craftsman tools and Canyon River Blues denim clothing. Page Ref: 289 Difficulty: Easy Objective: 11.7 Learning Outcome: Explain the role of marketing in organizations Skill: Concept 36) A product's benefits include all of the following EXCEPT which? A) the emotional satisfaction associated with owning the product B) the satisfaction associated with experiencing the product C) the time required to purchase the product D) the functions of the product E) the acceptance by a peer group Answer: C Explanation: C) Benefits include both the functions of the product as well as the emotional satisfaction associated with owning, experiencing, or possessing it. Page Ref: 270 Difficulty: Moderate Objective: 11.1 Learning Outcome: Explain the role of marketing in organizations Skill: Concept 37) What type of goods include raw materials and integrated circuits? A) substitute products B) industrial goods C) consumer goods D) marketing goods E) disposable goods Answer: B Explanation: B) Industrial goods are physical items used by companies to produce other products. Page Ref: 271 Difficulty: Moderate Objective: 11.1 Learning Outcome: Explain the role of marketing in organizations Skill: Concept 38) What type of marketing do insurance companies, airlines, investment counselors, health clinics, and public accountants typically engage in? A) personal selling B) service marketing C) industrial selling D) consumer marketing E) wholesale selling Answer: B Explanation: B) Service marketing, a major growth area in the United States, involves the sales of services to both consumer markets and industrial markets. Page Ref: 271 Difficulty: Moderate Objective: 11.1 Learning Outcome: Explain the role of marketing in organizations Skill: Application 39) What type of marketing is a "loyalty banking" program an example of? A) industrial marketing B) relationship marketing C) social marketing D) economic marketing E) consumer marketing Answer: B Explanation: B) Relationship marketing emphasizes building lasting relationships with customers and suppliers; loyalty banking programs are one example. Page Ref: 271 Difficulty: Moderate Objective: 11.1 Learning Outcome: Explain the role of marketing in organizations Skill: Application 40) What is data warehousing? A) a marketing strategy that creates sales agreements that stipulate product delivery B) the process of dividing a market into categories of customer types C) the study of consumer needs and wants and the way in which sellers can best meet them D) the process of collecting, storing, and retrieving data in electronic files E) a customer identification strategy that targets potential buyers Answer: D Explanation: D) The compiling and storage of data provides the raw materials from which marketers can extract information that could enable them to better know their customers and supply more of what they need or want. Page Ref: 272 Difficulty: Moderate AACSB: Use of information technology Objective: 11.1 Learning Outcome: Identify the different types of technologies used in business and describe their uses Skill: Concept 41) Congress has conducted hearings on the use of cell phones while driving. What factors would this constitute in a company's external environment? A) political-legal B) sociocultural C) technological D) competitive E) demographic Answer: A Explanation: A) Political activities, both foreign and domestic, have profound effects on marketing. Page Ref: 272 Difficulty: Moderate Objective: 11.1 Learning Outcome: Explain the role of marketing in organizations Skill: Application 42) Marketing managers often support the activities of political action committees (PACs). In doing this, which environment are they attempting to influence? A) competitive B) economic C) technological D) political-legal E) geo-demographic Answer: D Explanation: D) When companies contribute to political candidates and support the activities of PACs maintained by their respective industries, they are attempting to influence the political-legal environment. Page Ref: 272 Difficulty: Moderate Objective: 11.1 Learning Outcome: Explain the role of marketing in organizations Skill: Concept 43) A company decides to introduce a line of golf gear designed specifically for women. What environmental trend is the company responding to? A) competitive B) sociocultural C) political-legal D) economic E) technological Answer: B Explanation: B) Changes in social values encourage companies to develop and promote new products for both individual consumers and industrial consumers in a myriad of ways. Page Ref: 272 Difficulty: Moderate Objective: 11.1 Learning Outcome: Explain the role of marketing in organizations Skill: Application 44) What kind of factor in the business environment are cell phones considered an example of? A) economic B) technological C) global D) competitive E) sociocultural Answer: B Explanation: B) New technologies create new goods and services. Page Ref: 273 Difficulty: Moderate Objective: 11.1 Learning Outcome: Explain the role of marketing in organizations Skill: Concept 45) The satellite dish and home television shopping provide examples of changes to which of the following external environmental factors that affect marketing? A) economic B) technological C) legal D) political E) competitive Answer: B Explanation: B) Both satellite dishes and home television shopping are examples of technological change. Page Ref: 273 Difficulty: Moderate Objective: 11.1 Learning Outcome: Explain the role of marketing in organizations Skill: Application 46) What are fitness programs and drug regimens to lower cholesterol levels both an example of? A) brand competition B) marketing targets C) substitute products D) social trends E) brand extension Answer: C Explanation: C) Substitute products may not look alike, or they may seem very different from one another, but can fulfill the same need. Page Ref: 274 Difficulty: Moderate Objective: 11.1 Learning Outcome: Explain the role of marketing in organizations Skill: Application 47) What part of the marketing mix includes communicating information about products? A) product B) promotion C) price D) place E) publicity Answer: B Explanation: B) Promotion is the most highly visible element of the marketing mix; promotion involves all of the various techniques used for communicating information about products and services. Page Ref: 276 Difficulty: Moderate Objective: 11.2 Learning Outcome: Explain the role of marketing in organizations Skill: Concept 48) Which is usually the first step in the marketing strategy? A) distribution B) product differentiation C) target markets D) pricing E) integration Answer: C Explanation: C) Selecting target markets is usually the first step in the marketing strategy. Page Ref: 277 Difficulty: Moderate Objective: 11.3 Learning Outcome: Explain the role of marketing in organizations Skill: Concept 49) Burberry is repositioning itself as a global luxury brand, such as Gucci and Louis Vuitton, and calls for luring a different type of customer. This is an example of what type of market segmentation? A) geographic B) demographic C) psychographic D) product-use E) strategic Answer: C Explanation: C) Specific lifestyles, attitudes, and interests create certain customer tastes, such as luxury brands, among consumer segments; these are psychographic variables. Page Ref: 278 Difficulty: Moderate Objective: 11.3 Learning Outcome: Explain the role of marketing in organizations Skill: Application 50) Which of the following is the LEAST important influence on consumer behavior? A) psychological factors B) personal factors C) social factors D) technological factors E) corporate factors Answer: D Explanation: D) The influences that are most active in explaining consumers' choices and predicting future buying behaviors are psychological factors, personal influences, social influences, and cultural influences. Page Ref: 279 Difficulty: Moderate Objective: 11.4 Learning Outcome: Explain the role of marketing in organizations Skill: Concept 51) In consumer behavior, what is the name for the influences that include the "way of living" that distinguishes one large group from another and one ethnic group from another? A) psychological B) personal C) social D) domestic E) cultural Answer: E Explanation: E) Cultural influences also include subculture and social class. Page Ref: 279 Difficulty: Moderate Objective: 11.4 Learning Outcome: Explain the role of marketing in organizations Skill: Concept 52) What do consumers exhibit when they regularly purchase products because they are satisfied with the products' performance? A) aspirations B) cultural influence C) social influence D) brand loyalty E) expectations Answer: D Explanation: D) Further, such consumers are less subject to influence and tend to stick with preferred brands. Page Ref: 279 Difficulty: Moderate Objective: 11.4 Learning Outcome: Explain the role of marketing in organizations Skill: Concept 53) Reading the relevant issues of Consumer Reports prior to purchasing a product is an example of what part of the consumer buying process? A) problem recognition B) information seeking C) selecting substitutes D) need recognition E) consideration setting Answer: B Explanation: B) The search is not always extensive, but before making major purchases, most people seek information from personal sources, public sources, and experience. Page Ref: 280 Difficulty: Moderate Objective: 11.4 Learning Outcome: Explain the role of marketing in organizations Skill: Application 54) Marketing does not stop with the sale of a product. What is the term that describes what happens after the sale? A) postpurchase evaluation B) prepurchase evaluation C) rational motives D) emotional motives E) evoked set Answer: A Explanation: A) Marketers want consumers to be happy after buying products so that they are more likely to buy them again. Page Ref: 280 Difficulty: Moderate Objective: 11.4 Learning Outcome: Explain the role of marketing in organizations Skill: Concept 55) What do B2B buyer-seller situations often involve? A) frequent and enduring buyer-seller relationships B) incidental buyer-seller interactions C) buyers and sellers with limited training D) short term buyer-seller relationships E) commercial buyer-seller networks Answer: A Explanation: A) In contrast, consumer-seller relationships are often impersonal, short-lived, one-time interactions. Page Ref: 282 Difficulty: Moderate Objective: 11.5 Learning Outcome: Explain the role of marketing in organizations Skill: Concept 56) Bill is looking for an insurance policy at an affordable price. What type of good or service is he looking for? A) shopping good B) shopping service C) specialty good D) specialty service E) convenience good Answer: B Explanation: B) Shopping services (such as insurance) are more expensive and are purchased less often than convenience products. Consumers often compare brands, sometimes in different stores. They may also evaluate alternatives in terms of style, performance, color, price, and other criteria. Page Ref: 283 Difficulty: Moderate Objective: 11.6 Learning Outcome: Explain the role of marketing in organizations Skill: Application 57) What type of good is a custom-made wedding gown classified as? A) convenience good B) shopping good C) ceremonial good D) luxury good E) specialty good Answer: E Explanation: E) A custom-made wedding gown is a specialty good, often extremely important to the consumer and expensive. Consumers of such goods decide precisely on what they want and will accept no substitutes. Page Ref: 283 Difficulty: Moderate Objective: 11.6 Learning Outcome: Explain the role of marketing in organizations Skill: Application 58) A firm makes bookcases, bicycles, soft drinks, armored cars, and novelty dishes. What do these items make up? A) product line B) product variety C) product assortment D) product mix E) product inventory Answer: D Explanation: D) The group of products that a company makes available for sale, whether consumer, industrial, or both, is its product mix. A group of products that are closely related because they function in a similar manner is a product line. Page Ref: 284 Difficulty: Moderate Objective: 11.7 Learning Outcome: Explain the role of marketing in organizations Skill: Application 59) What results when companies extend their horizons and identify opportunities outside existing product lines? A) disconnected product lines B) multiple product lines C) uniform product lines D) universal product lines E) tailored product lines Answer: B Explanation: B) Many marketers extend their product lines to include multiple, or diversified, product lines. Starbucks, for example, has a line of home brewing equipment, a line of supermarket products, a line of music, etc. Page Ref: 284 Difficulty: Moderate Objective: 11.7 Learning Outcome: Explain the role of marketing in organizations Skill: Concept 60) During the product life cycle, when do sales begin to climb and the product begin to show profit? A) ascending B) mature C) initial D) growth E) accelerated Answer: D Explanation: D) In the growth stage, sales start to climb rapidly if the new product attracts enough consumers. The product starts to show profit, and other firms move rapidly to introduce their own versions. Page Ref: 286 Difficulty: Moderate Objective: 11.7 Learning Outcome: Explain the role of marketing in organizations Skill: Concept 61) During the product life cycle, when do sales begin to slow down while profit levels are high? A) decline B) maturity C) introduction D) growth E) promotion Answer: B Explanation: B) Sales start to slow in the maturity stage; although the product earns its highest profit level early in this stage, increased competition eventually forces price cutting and lower profits. Page Ref: 286 Difficulty: Moderate Objective: 11.7 Learning Outcome: Explain the role of marketing in organizations Skill: Concept 62) LiveLong Company provides volunteer participants with new products to try and then asks them to share the information with their friends, family, coworkers, and others in their social network. What is this type of marketing called? A) share marketing B) buzz marketing C) viral marketing D) placement marketing E) target marketing Answer: B Explanation: B) Buzz marketing relies on word of mouth to spread "buzz" about a particular product or idea. Page Ref: 288 Difficulty: Moderate Objective: 11.7 Learning Outcome: Explain the role of marketing in organizations Skill: Application 63) Noxzema, Prudential, and Minute Maid would be examples of which kind of brand name? A) licensed B) private C) national D) international E) convenience Answer: C Explanation: C) National brands are produced by, widely distributed by, and carry the name of the manufacturer. Page Ref: 288 Difficulty: Moderate Objective: 11.7 Learning Outcome: Explain the role of marketing in organizations Skill: Application 64) What type of brands are Sears Craftsman tools and Kenmore appliances examples of? A) national B) licensed C) private D) global E) social Answer: C Explanation: C) When a wholesaler or retailer develops a brand name and has a manufacturer put it on a product, the resulting name is a private brand. Page Ref: 289 Difficulty: Moderate Objective: 11.7 Learning Outcome: Explain the role of marketing in organizations Skill: Application 65) Some products can be sold abroad with virtually no changes. Which of the following is an example of such a product? A) Coca-Cola B) electric razors C) Gerber baby food D) Apple computers E) fruit juices Answer: A Explanation: A) On the other hand, a variety of products must be created with inbuilt flexibility in order to be able to be marketed internationally. Page Ref: 290 Difficulty: Moderate Objective: 11.8 Learning Outcome: Explain the benefits and challenges of engaging in international business Skill: Application 66) Little Earth Productions is a company that makes fashion accessories such as handbags. After examining shopping habits, what should they focus on for better visibility of the product for customers? A) better shipping techniques B) better in-store displays C) zero response time D) lower pricing E) distribution Answer: B Explanation: B) Better display techniques showing the products to their best advantage can increase product visibility and sales. Page Ref: 290 Difficulty: Moderate Objective: 11.8 Learning Outcome: Describe the major components of effective promotion Skill: Application 67) Leif designs and sells surfboards. Although his sales are declining, he decides to keep his prices constant. The marketing professionals with whom Leif consults all support his decision. Which of the following statements is LEAST likely to be true? A) Surfboards at the other shops are priced much higher. B) Leif's shop is more strategically located than the other stores are. C) The other shops' promotional materials are far superior to Leif's. D) Leif offers a much wider variety of surfboards than his competitors do. E) Leif and the marketing professionals believe that low demand will continue. Answer: E Explanation: E) Marketing professionals are well-versed in the best marketing practices. If they support Leif's decision, they must have reason to believe his business will recover. Page Ref: 275 Difficulty: Difficult AACSB: Reflective thinking skills Objective: 11.8 Learning Outcome: Discuss strategies for setting and adjusting prices Skill: Application 68) Which of the following occurs between similar products and is based on buyers' perceptions of the benefits of products offered by particular companies? A) international competition B) substitute competition C) brand competition D) benefit competition E) placement competition Answer: C Explanation: C) Brand competition is based on buyers' perceptions of the benefits of products offered by particular companies. Page Ref: 275 Difficulty: Difficult Objective: 11.1 Learning Outcome: Explain the role of marketing in organizations Skill: Concept 69) Garrett is in the business of selling baseball bats. He decides to employ focus groups to determine which bats will likely be bestsellers. Which of the following groups will he be LEAST likely to invite as participants? A) players from the state's professional baseball team B) local college and high school coaches C) administrators of the local Little League baseball program D) spectators at a baseball tournament E) junior high players from the surrounding counties Answer: D Explanation: D) Spectators have an interest in baseball but are not necessarily baseball bat users. They would not provide the best feedback to inform a good decision. Page Ref: 277 Difficulty: Difficult AACSB: Reflective thinking skills Objective: 11.3 Learning Outcome: Explain the role of marketing in organizations Skill: Application 70) Tamika starts a business making teddy bears and is excited to deliver the first batch to young patients. When Tamika arrives at the hospital, she learns that she has more bears than she needs. Which of the following would likely explain this? A) Tamika failed to collect primary data. B) Tamika did not interview all of the children. C) Tamika was not told that five more children had been admitted. D) Tamika conducted too much market research. E) Tamika received incorrect secondary data from the hospital. Answer: E Explanation: E) Secondary data are data that have already been collected and processed. The hospital most likely gave Tamika the wrong number of children. Page Ref: 279 Difficulty: Difficult AACSB: Reflective thinking skills Objective: 11.3 Learning Outcome: Explain the role of marketing in organizations Skill: Application 71) Tamika starts a business making teddy bears and elects to use focus groups to determine whether customized bears would be well received in a hospital setting. The focus groups prove to be unhelpful in answering this question. Which of the following could NOT be a valid explanation? A) Not enough focus groups were used. B) Focus groups are not a recommended method for obtaining people's opinions. C) The people in the focus groups could not relate to a hospitalized child's experience. D) Responses from focus group members were not recorded accurately. E) The questions asked were not designed to reveal the information being sought. Answer: B Explanation: B) This statement is untrue. Focus groups are a primary data source used to solicit feedback in reference to a product or service. Page Ref: 279 Difficulty: Difficult AACSB: Reflective thinking skills Objective: 11.4 Learning Outcome: Explain the role of marketing in organizations Skill: Application 72) Quinn Clothes is a successful business selling fashionable and athletic swimwear. The marketing staff has said that the company's plan to develop and market waterproof perfumes might be hindered by a lack of experience in the perfume field. Which of the following is the BEST response to that weakness? A) Hire a chemist to develop the fragrances. B) Seek product ideas from existing staff. C) Employ new graphic designers to develop a new logo. D) Ensure that the perfume can be sold in the same retail stores as the swimwear. E) Rethink the company's promotional approach to selling products. Answer: A Explanation: A) Bringing in a specialist will best help develop a good product. Page Ref: 275 Difficulty: Difficult AACSB: Reflective thinking skills Objective: 11.1 Learning Outcome: Explain the role of marketing in organizations Skill: Application 73) What does the process of "positioning" involve? A) hiring only employees who have the strongest credentials B) designing a company's products to appeal to target customers C) focusing on only one component of the marketing mix D) disregarding customers' reactions to a company's products E) reorganizing employees according to seniority Answer: B Explanation: B) By definition, positioning is the process of developing a unique marketing mix that best satisfies a target market. Page Ref: 277 Difficulty: Moderate AACSB: Reflective thinking skills Objective: 11.3 Learning Outcome: Explain the role of marketing in organizations Skill: Concept 74) What would be the best target market for waterproof fragrances? A) male lifeguards B) college fraternity members C) Olympic ice skaters D) school-aged children who live in warm climates E) female swimmers and divers Answer: E Explanation: E) This product would best meet the needs of female (with the perfume) swimmers and divers (with the bathing suits). Page Ref: 278 Difficulty: Moderate AACSB: Reflective thinking skills Objective: 11.3 Learning Outcome: Explain the role of marketing in organizations Skill: Application 75) Which of the following is the best example of an emotional motive in consumer purchasing? A) A major shoe company purchases the latest designer styles in order to please customers. B) A leather shop provides goods to local handbag makers. C) A businesswoman buys a new suit for an important presentation. D) A housewife donates unworn dresses to a resale shop. E) A group of neighborhood moms holds a free childcare seminar for new mothers. Answer: C Explanation: C) A consumer market transaction involves less sophisticated and more emotional purchase decisions made by an individual. Page Ref: 280 Difficulty: Difficult Objective: 11.4 Learning Outcome: Explain the role of marketing in organizations Skill: Application 76) Accessing a Web site in search of magazine articles about a product before its purchase is an example of what part of the consumer buying process? A) post-purchase evaluation B) problem-solving C) information seeking D) purchasing the product E) recognizing a need Answer: C Explanation: C) The search is not always extensive, but before making major purchases, most people seek information from personal sources, public sources, and experience. Page Ref: 280 Difficulty: Moderate Objective: 11.4 Learning Outcome: Explain the role of marketing in organizations Skill: Application 77) Which of the following is the best example of a B2B market transaction? A) A college student buys dorm furniture from an on-campus store. B) A group of teenage girls buy new clothes for the upcoming school year. C) A high-school coach borrows sports equipment from a neighboring school. D) A dentist provides free toothbrushes to patients. E) A chain of ice cream parlors regularly purchases napkins from the same supplier. Answer: E Explanation: E) Often buyers in B2B markets purchase directly from sellers, as opposed to consumer markets, where products typically go through many wholesalers before the product is delivered to the end user. Page Ref: 282 Difficulty: Moderate AACSB: Reflective thinking skills Objective: 11.5 Learning Outcome: Explain the role of marketing in organizations Skill: Application 78) Which defining word or phrase below best captures the meaning of capital items? A) bulk loads B) directly used in the production process C) to the consumer within a year D) permanent E) stable Answer: D Explanation: D) Capital items are permanent goods or services; they have expected lives of more than a year and, typically, of several years. Page Ref: 283 Difficulty: Difficult Objective: 11.6 Learning Outcome: Explain the role of marketing in organizations Skill: Concept Chapter 12 Pricing, Distributing, and Promoting Products 30) Which of the following terms refers to the monetary amount added to a product's cost to the seller in order to sell it at a target profit? A) profit margin B) breakeven cost C) revenue D) revenue margin E) markup Answer: E Explanation: E) Products are marked up from the cost to produce the product in order that they may be sold at a profit. Page Ref: 301 Difficulty: Easy Objective: 12.1 Learning Outcome: Discuss strategies for setting and adjusting prices Skill: Concept 31) Which of the following costs refers specifically to those that change with the quantity of a product sold or produced? A) annual costs B) variable costs C) fixed costs D) marginal costs E) market-adjusted costs Answer: B Explanation: B) Variable costs are adjusted with regard to how much of a product is produced or how much is projected to be sold, so that a certain profit can be obtained. Page Ref: 301 Difficulty: Easy Objective: 12.1 Learning Outcome: Discuss strategies for setting and adjusting prices Skill: Concept 32) Which of the following costs refers specifically to those that are unaffected by the quantity of a product sold or produced? A) annual costs B) variable costs C) fixed costs D) marginal costs E) market-adjusted costs Answer: C Explanation: C) Fixed costs are those that it cost to produce the product in the first place. Page Ref: 301 Difficulty: Easy Objective: 12.1 Learning Outcome: Discuss strategies for setting and adjusting prices Skill: Concept 33) Odd-even pricing falls under the category of which of the following? A) price fixing B) psychological pricing C) market pricing D) price lining E) price skimming Answer: B Explanation: B) Odd-even pricing posits that consumers generally don't like to pay an even amount of dollars for their products. It also takes advantage of those consumers who don't understand the concept of "rounding up." Page Ref: 304-305 Difficulty: Easy Objective: 12.2 Learning Outcome: Discuss strategies for setting and adjusting prices Skill: Concept 34) Which of the following pricing strategies has stimulation of sales as a higher priority than maximization of profits? A) price fixing B) price discounting C) price skimming D) price pointing E) price lining Answer: B Explanation: B) This is a strategy often used to introduce a product in an already very populated market. Page Ref: 303 Difficulty: Easy Objective: 12.2 Learning Outcome: Discuss strategies for setting and adjusting prices Skill: Concept 35) Which of the following refers to the combination of channels that a firm selects to get a product to end users? A) distribution mix B) transportation strategy C) merchandising system D) marketing plan E) retail system Answer: A Explanation: A) The distribution mix is the total use of distribution to optimally get a product into the hands of consumers. Page Ref: 305 Difficulty: Easy Objective: 12.3 Learning Outcome: Describe the major components of effective distribution Skill: Concept 36) Rachel has set up a firm that buys circuit boards from overseas producers and sells them to a local computer manufacturing company. Which title best describes her role in the computer manufacturing distribution mix? A) marketer B) sales agent C) broker D) retailer E) wholesaler Answer: E Explanation: E) A wholesaler buys products from a producer in bulk and sells them to a retailer or other producer. Page Ref: 305 Difficulty: Moderate Objective: 12.3 Learning Outcome: Describe the major components of effective distribution Skill: Application 37) An online comic book store is an example of which type of intermediary? A) marketer B) sales agent C) broker D) wholesaler E) retailer Answer: E Explanation: E) Retailers are those intermediaries who sell the product directly to the consumer. Page Ref: 305 Difficulty: Moderate Objective: 12.3 Learning Outcome: Describe the major components of effective distribution Skill: Application 38) A farmers' market is an example of which type of distribution channel? A) direct distribution B) retail distribution C) wholesale distribution D) distribution by brokers E) distribution by sales agents Answer: A Explanation: A) Since the product is channeled directly from producer to consumer, this is direct distribution. Page Ref: 305-306 Difficulty: Moderate Objective: 12.3 Learning Outcome: Describe the major components of effective distribution Skill: Application 39) Phillip is an independent intermediary who deals in the local produce market of northeast Ohio. Over the years, he has formed long-term relationships and come to represent several farmers in the area in the sale of their produce to local grocery stores. Which of the following titles best describes Phillip's role in the distribution mix of northeast Ohio produce? A) wholesaler B) retailer C) sales agent D) broker E) direct marketer Answer: C Explanation: C) A sales agent sells products from producers directly to consumers or retailers. Page Ref: 306 Difficulty: Moderate Objective: 12.3 Learning Outcome: Describe the major components of effective distribution Skill: Application 40) Which of the following intermediaries matches numerous sellers and buyers as needed, often without knowing in advance who they will be? A) producer representative B) retailer C) syndicated seller D) sales agent E) broker Answer: E Explanation: E) A broker is an agent that matches sellers and buyers in the market. Page Ref: 306 Difficulty: Easy Objective: 12.4 Learning Outcome: Describe the major components of effective distribution Skill: Concept 41) Which of the following denotes small retail stores carrying only one product line or category of related products? A) department stores B) bargain retailers C) supermarkets D) convenience stores E) specialty stores Answer: E Explanation: E) Specialty stores carry only one type of product for a certain type of consumer. Page Ref: 307 Difficulty: Easy Objective: 12.4 Learning Outcome: Describe the major components of effective distribution Skill: Concept 42) In which of the following bargain retailers do the customers view display samples, place orders, and wait briefly while clerks retrieve orders from attached warehouses? A) off-price store B) catalog showroom C) factory outlet D) wholesale club E) discount house Answer: B Explanation: B) Catalog showrooms display samples of items available to order by catalog. Page Ref: 309 Difficulty: Easy Objective: 12.4 Learning Outcome: Describe the major components of effective distribution Skill: Concept 43) Which of the following best describes department stores? A) A department store is a large product line retailer organized into smaller, convenience-focused components. B) A department store is a large product line retailer organized into smaller, bargain-focused components. C) A department store is a large product line retailer organized into smaller, producer-focused components. D) A department store is a large product line retailer organized into smaller, product-focused components. E) A department store is a large product line retailer organized into smaller specialty stores. Answer: D Explanation: D) Department stores are characterized by organization by the types of products. Page Ref: 307 Difficulty: Moderate Objective: 12.4 Learning Outcome: Describe the major components of effective distribution Skill: Concept 44) Which of the following terms refers specifically to the activities needed to move products efficiently from manufacturer to consumer? A) physical distribution B) the distribution mix C) product distribution D) transportation modes E) product placement Answer: A Explanation: A) Physical distribution is the moving of products from the producer of the product to the eventual consumer who buys the product. Page Ref: 311 Difficulty: Easy Objective: 12.5 Learning Outcome: Describe the major components of effective distribution Skill: Concept 45) Within which of the following storage facilities would one expect to see the widest variety of products stored? A) private warehouse B) public warehouse C) food storage facility D) department store distribution center E) specialty store stockroom Answer: B Explanation: B) Given that public warehouses are utilized by anyone willing to pay the rental fee, this will necessarily have the widest variety of products within it. Page Ref: 311 Difficulty: Moderate AACSB: Reflective thinking skills Objective: 12.6 Learning Outcome: Describe the major components of effective distribution Skill: Concept 46) Which of the following concerns finding the most effective techniques for communicating information about and selling a product? A) market share B) advertising C) promotion D) profit management E) customer relations Answer: C Explanation: C) Promotion is the communication of attributes about a product to the consumer. Page Ref: 314 Difficulty: Easy Objective: 12.6 Learning Outcome: Describe the major components of effective promotion Skill: Concept 47) Which of the following describes two of the primary objectives of promotion? A) positioning and communicating information B) positioning and developing new uses for products C) developing new uses for products and communicating information D) adding value to a product and eliminating competition E) controlling sales volume and eliminating competition Answer: A Explanation: A) These are two of the main four objectives, the others being "adding value" and "controlling sales volume." Page Ref: 314 Difficulty: Moderate Objective: 12.6 Learning Outcome: Describe the major components of effective promotion Skill: Concept 48) Which of the following best characterizes the difference between the roles of advertising and personal selling in the promotional mix of a firm? A) Advertising is, at its worst, very expensive with little return, whereas personal selling is also very expensive, at its worst, but with better returns. B) Advertising, at its best, targets a specific audience about a product, whereas personal selling, at its best, is always dependent on the clients that a seller has. C) Advertising involves public relations with the overall population of consumers, whereas personal selling involves personalized relationships with select individuals. D) Advertising is a relatively new method of promotion, whereas personal selling is the oldest method of promotion. E) Advertising is nonpersonal in its communication about a product, whereas personal selling is highly individualized. Answer: E Explanation: E) The blanketing efforts of advertising versus the individualized care of personal selling best characterizes the different roles that these two methods play. Page Ref: 315-317 Difficulty: Difficult AACSB: Reflective thinking skills Objective: 12.6 Learning Outcome: Describe the major components of effective promotion Skill: Synthesis 49) A firm that wanted to conduct a short-term activity in order to boost consumer buying would use which of the following strategies? A) sales promotions B) special offers C) trade shows D) point-of-sale displays E) missionary selling Answer: A Explanation: A) These would include coupons or premiums. Page Ref: 319 Difficulty: Easy Objective: 12.7 Learning Outcome: Describe the major components of effective promotion Skill: Concept 50) Which of the following is NOT a type of sales promotion? A) warranties B) coupons C) point-of-sale displays D) trade show displays E) free samples Answer: A Explanation: A) This is not a sales promotion type—it is concerned with the product after it has been bought. Page Ref: 320 Difficulty: Easy Objective: 12.7 Learning Outcome: Describe the major components of effective promotion Skill: Concept 51) Companies that price their product low enough to attract a large number of buyers are most likely pursuing which of the following goals? A) stable market pricing B) high profits C) stable market activity D) low market volatility E) high market share Answer: E Explanation: E) Because marketers are willing to accept minimal profits, even losses, to get buyers to try products, they are likely pursuing a goal of high market share. A goal of high market share indicates that the marketer is aiming to eventually supply a larger portion of the market. Page Ref: 300 Difficulty: Moderate AACSB: Reflective thinking skills Objective: 12.1 Learning Outcome: Discuss strategies for setting and adjusting prices Skill: Concept 52) The sales price of a product is given as the product's cost to the seller plus which of the following quantities? A) breakeven price B) revenue price C) variable price D) fixed price E) markup price Answer: E Explanation: E) Markup is the amount added to an item's cost to sell it at a profit. Page Ref: 301 Difficulty: Moderate Objective: 12.1 Learning Outcome: Discuss strategies for setting and adjusting prices Skill: Concept 53) Which of the following gives the correct formula for calculating markup percentage? A) sales price/markup B) sales price/fixed costs C) fixed costs/variable costs D) markup/sales price E) sales price/variable costs Answer: D Explanation: D) For example, a reasonable sales price of $15 with a markup of $7 yields a markup percentage of 46.7 percent: $7/$15 = 46.7 percent. Page Ref: 301 Difficulty: Moderate Objective: 12.1 Learning Outcome: Discuss strategies for setting and adjusting prices Skill: Concept 54) From the choices below, pick the best definition of the term fixed costs. A) costs that are needed to acquire a fixed market share B) costs that are needed to acquire fixed profits C) costs that are unaffected by the number of goods sold by the firm D) costs that remain the same from year to year E) costs that remain the same across all products Answer: C Explanation: C) Fixed costs are costs that must be paid to produce the product at all, regardless of the number of goods sold by the firm. Page Ref: 301 Difficulty: Moderate Objective: 12.1 Learning Outcome: Discuss strategies for setting and adjusting prices Skill: Concept 55) Which of the following strategies is involved with price skimming? A) setting an initial low price to establish a new product in the market B) setting an initial low price to cover new product costs and still generate a profit C) setting a limited number of prices for certain categories or products D) setting an initial high price to establish a new product in the market E) setting an initial high price to cover new product costs and still generate a profit Answer: E Explanation: E) The revenue is often needed to cover development and production costs. Skimming works only if marketers can convince consumers that a new product is truly different from existing products. Page Ref: 303 Difficulty: Moderate Objective: 12.2 Learning Outcome: Discuss strategies for setting and adjusting prices Skill: Concept 56) Which of the following strategies is involved with penetration pricing? A) setting an initial low price to establish a new product in the market B) setting an initial low price to cover new product costs and still generate a profit C) setting a limited number of prices for certain categories or products D) setting an initial high price to establish a new product in the market E) setting an initial high price to cover new product costs and still generate a profit Answer: A Explanation: A) Penetration pricing is used to create consumer interest and stimulate trial purchases among consumers. Page Ref: 303 Difficulty: Moderate Objective: 12.2 Learning Outcome: Discuss strategies for setting and adjusting prices Skill: Concept 57) Which of the following pricing strategies is used by many e-tailers in order to provide flexibility between buyers and sellers in setting a price? A) penetration pricing B) psychological pricing C) breakeven pricing D) fixed pricing E) dynamic pricing Answer: E Explanation: E) Dynamic pricing is a highly variable pricing system. Page Ref: 304 Difficulty: Moderate Objective: 12.2 Learning Outcome: Discuss strategies for setting and adjusting prices Skill: Application 58) Which of the following pricing strategies best describes price lining? A) setting an initial low price to establish a new product in the market B) setting an initial high price to cover new product costs and generate a profit C) setting individually negotiated prices for certain categories of products D) setting a limited number of prices for certain categories of products E) setting individually negotiated prices for all categories of products Answer: D Explanation: D) Price lining is illustrated when a retailer offers sweatshirts at three amounts: $25, $35, and $50. The ranges in price likely indicate varying quality levels. Page Ref: 304 Difficulty: Moderate Objective: 12.2 Learning Outcome: Discuss strategies for setting and adjusting prices Skill: Concept 59) Which distribution channel is used by Valley Farm Dairy to distribute its products to consumers without intermediaries? A) direct distribution B) retail distribution C) wholesale distribution D) distribution by agents E) distribution by brokers Answer: A Explanation: A) That is, the product travels from the producer directly to the consumer. Page Ref: 305-306 Difficulty: Moderate Objective: 12.3 Learning Outcome: Describe the major components of effective distribution Skill: Application 60) Which distribution channel is used by companies which distribute their goods through their own system of outlets? A) direct distribution B) retail distribution C) wholesale distribution D) distribution by agents E) distribution by brokers Answer: B Explanation: B) This illustrates retail distribution; producers distribute products through retailers. Goodyear maintains its own system of retail stores; Levi's has its own outlets but also produces jeans for other retailers such as Gap. Page Ref: 305 Difficulty: Moderate Objective: 12.3 Learning Outcome: Describe the major components of effective distribution Skill: Concept 61) Convenience stores utilize which kind of distribution? A) direct distribution B) retail distribution C) wholesale distribution D) distribution by agents E) distribution by brokers Answer: C Explanation: C) Wholesalers play a large role in the storage function; faced with rising costs of storage space, many retailers have found that they cannot afford both retail and storage space. Page Ref: 305-306 Difficulty: Moderate Objective: 12.3 Learning Outcome: Describe the major components of effective distribution Skill: Concept 62) Which component of the distribution mix do agents and brokers represent in receiving commissions? A) producers B) consumers C) retailers D) industrial users E) other intermediaries Answer: A Explanation: A) Agents and brokers represent producers and sell to consumers, industrial users, or wholesalers. Page Ref: 306 Difficulty: Moderate Objective: 12.3 Learning Outcome: Describe the major components of effective distribution Skill: Concept 63) In which type of store would you most likely find shoes, furniture, a women's section, and a men's section? A) a supermarket B) a discount house C) a department store D) a convenience store E) a specialty store Answer: C Explanation: C) Department stores are organized into specialized departments; these stores usually handle a wide array of goods and offer a variety of services. Page Ref: 307 Difficulty: Moderate Objective: 12.4 Learning Outcome: Describe the major components of effective distribution Skill: Concept 64) Which of the following gives examples of bargain retailers? A) bakery outlet, athletic shoe store B) local bakery, discount shoe chain C) bakery thrift shop, athletic shoe store D) bakery thrift shop, shoe factory outlet E) local bakery, shoe factory outlet Answer: D Explanation: D) Bargain retailers, such as these, carry wide ranges of products and come in many forms. Page Ref: 307-309 Difficulty: Moderate Objective: 12.4 Learning Outcome: Describe the major components of effective distribution Skill: Application 65) Which of the following bargain retailers generate large sales volume by offering a wide variety of new products at substantial price reductions? A) discount houses B) catalog showrooms C) second-hand stores D) factory outlets E) direct-response bargain marketing Answer: A Explanation: A) Discount houses began by selling large numbers of items at large discounts on a cash-only basis. As these retailers began to establish themselves, they eventually evolved into moving to better locations, improving décor, selling better-quality merchandise, and offering department store services. Page Ref: 307 Difficulty: Moderate Objective: 12.4 Learning Outcome: Describe the major components of effective distribution Skill: Concept 66) In which of the following retail outlets is there a most direct communication line between producer and consumer? A) e-retailing B) specialty store retailing C) catalog showroom retailing D) mail-order retailing E) brokered retailing Answer: D Explanation: D) Mail order is an example of direct-response retailing, in which the producer is directly communicating to the consumers, usually via mail catalogs. Page Ref: 307-309 Difficulty: Difficult AACSB: Reflective thinking skills Objective: 12.4 Learning Outcome: Describe the major components of effective distribution Skill: Synthesis 67) What are the three major classifications of retail outlets in the United States? A) large-sized outlets, medium-sized specialty shops, small-sized convenience stores B) product-based retailers, discount-based retailers, convenience-based retailers C) quality-based retailers, quantity-based retailers, specialty-based retailers D) producer-owned retailers, franchised retailers, independently owned retailers E) brick-and-mortar retailers, e-retailers, hybrid retailers Answer: B Explanation: B) The three major classifications relate to the overall goal of service to the customer. Page Ref: 307 Difficulty: Difficult AACSB: Reflective thinking skills Objective: 12.4 Learning Outcome: Describe the major components of effective distribution Skill: Concept 68) Which of the following describes the function of search engines that act as collections of business Web sites representing diverse products? A) Internet service providers B) electronic storefronts C) cybermalls D) e-catalogs E) interactive retailing Answer: C Explanation: C) After entering a cybermall, consumers can navigate by choosing from a list of stores, product listings, or departments. Page Ref: 310 Difficulty: Moderate Objective: 12.5 Learning Outcome: Describe the major components of effective distribution Skill: Concept 69) Home Shopping Club and QVC are examples of which of the following retail sales outlet? A) direct selling B) video marketing C) electronic shopping D) virtual selling E) telemarketing Answer: B Explanation: B) Video marketing, a long-established form of interactive marketing, lets viewers shop at home from TV screens by phoning in or e-mailing orders. Page Ref: 311 Difficulty: Moderate Objective: 12.5 Learning Outcome: Describe the major components of effective distribution Skill: Application 70) J.C. Penney owns and maintains its own warehouses. What is this type of warehouse called? A) public warehouse B) relay warehouse C) distribution center D) private warehouse E) storage warehouse Answer: D Explanation: D) A private warehouse is owned by a single manufacturer, wholesaler, or retailer. Page Ref: 311 Difficulty: Moderate Objective: 12.6 Learning Outcome: Describe the major components of effective distribution Skill: Application 71) Which of the following will a large firm utilize when it needs temporary storage space for a large amount of products in immediate demand? A) public warehouses B) distribution centers C) relay warehouses D) private warehouses E) storage warehouses Answer: A Explanation: A) Public warehouses are independently owned and operated; because companies rent only the space they need, these facilities are popular with firms needing storage only during peak periods. Page Ref: 311 Difficulty: Moderate Objective: 12.6 Learning Outcome: Describe the major components of effective distribution Skill: Concept 72) Which of the following is the fastest and most expensive transportation mode? A) railroads B) airplanes C) pipelines D) water carriers E) trucks Answer: B Explanation: B) Air is the fastest and most expensive form of transportation. Page Ref: 312 Difficulty: Moderate Objective: 12.6 Learning Outcome: Describe the major components of effective distribution Skill: Concept 73) Which of the following is the slowest and the cheapest mode of transportation? A) railroads B) planes C) pipelines D) water carriers E) trucks Answer: D Explanation: D) Networks of waterways allow water carriers to reach many areas throughout the world; this mode is the slowest and the cheapest. Page Ref: 312 Difficulty: Moderate Objective: 12.6 Learning Outcome: Describe the major components of effective distribution Skill: Concept 74) Which of the following is involved in the final stage in the buyer decision process? A) After the buyer has purchased the product, the marketer asks for feedback on the quality of the product. B) After the buyer has purchased the product, the marketer promotes more advanced versions of the product. C) After the buyer has purchased the product, the marketer provides customer assistance for use of the product. D) After the buyer has purchased the product, the marketer reminds the buyer that he or she made a wise purchase. E) After the buyer has purchased the product, the marketer promotes related products by the same company. Answer: D Explanation: D) During the final stage, advertising and personal selling can remind customers that they made wise purchases. Page Ref: 315 Difficulty: Moderate Objective: 12.6 Learning Outcome: Describe the major components of effective promotion Skill: Concept 75) Which of the following is crucial for high-priced consumer products such as homes and cars? A) creative selling B) interactive marketing C) missionary selling D) order processing E) trade shows Answer: A Explanation: A) Creative selling can help persuade customers to purchase a high-priced item. Page Ref: 319 Difficulty: Moderate Objective: 12.7 Learning Outcome: Describe the major components of effective promotion Skill: Concept 76) Which of the following are examples of premiums? A) contests that reward winners with free samples of the product B) free items given out at trade shows C) coupons for reduced prices on certain products D) reduced-price items available upon purchase of a product E) reduced-price items available during a sales promotion Answer: D Explanation: D) Premiums are free or reduced-priced items given to consumers in return for buying a specified product. Page Ref: 318 Difficulty: Moderate Objective: 12.7 Learning Outcome: Describe the major components of effective promotion Skill: Application 77) Which of the following best describes the purpose of point-of-sale displays? A) to inform customers about the benefits of the products B) to entice customers to buy additional products C) to make it easier for customers to find products D) to promote the reputation of the company offering certain products E) to target specific messages about the product to customers Answer: C Explanation: C) POS displays make it easier for customers to find products and easier for sellers to eliminate competitors. Page Ref: 318 Difficulty: Moderate Objective: 12.7 Learning Outcome: Describe the major components of effective promotion Skill: Concept 78) Which of the following describes best what distinguishes publicity among all other forms of promotion? A) Publicity is less directed than other forms of promotion. B) Publicity is less effective than other forms of promotion. C) Publicity is more impersonal than other forms of promotion. D) Publicity is cheaper than other forms of promotion. E) Publicity is more direct than other forms of promotion Answer: D Explanation: D) Given that publicity is free, it is indeed cheaper than any other form of promotion. Page Ref: 319 Difficulty: Moderate Objective: 12.7 Learning Outcome: Describe the major components of effective promotion Skill: Concept 79) A well-established Internet provider is facing stiff competition from several new companies. Which of the following techniques would it use in order to demonstrate to consumers that its product is superior to that of its competitors? A) order processing B) sales promotions C) direct marketing D) creative selling E) missionary selling Answer: D Explanation: D) With creative selling, salespeople persuade buyers to purchase their product by providing information about its benefits. Page Ref: 319 Difficulty: Difficult Objective: 12.7 Learning Outcome: Describe the major components of effective promotion Skill: Application 80) A highly successful bakery specializes in making cherry tarts. Which of the following costs at the bakery is NOT variable? A) cost of cherries B) cost of flour C) cost of wax paper liners D) cost of the oven E) cost of heating the oven Answer: D Explanation: D) The oven is the only cost listed here that is not directly dependent on the number of tarts made—its cost will remain constant no matter how many tarts are made and sold. Page Ref: 301 Difficulty: Difficult Objective: 12.1 Learning Outcome: Discuss strategies for setting and adjusting prices Skill: Application 81) Betty's Breads is trying to calculate its breakeven point. Monthly fixed costs are $5,255. The cost of making one loaf of bread, considering labor and materials, is $3.50. Betty's sells the bread at $6 per loaf. How many loaves does Betty's need to sell each month in order to break even? A) 251 B) 626 C) 1502 D) 2102 E) 2200 Answer: D Explanation: D) Breakeven point (in units) = fixed costs/(price - variable cost per unit). In this scenario, breakeven point = $5255/($6 - $3.50) = 2102 loaves of bread. Page Ref: 302 Difficulty: Difficult AACSB: Analytic skills Objective: 12.1 Learning Outcome: Discuss strategies for setting and adjusting prices Skill: Application 82) Which of the following assumptions is the strategy of pricing above the market based on? A) Higher price implies higher quality. B) Higher price means higher customer support. C) Higher prices means higher customer satisfaction. D) Higher price creates higher demand. E) Higher price means easier distribution. Answer: A Explanation: A) Godiva Chocolates and Patek Philippe watches price high by promoting prestige and quality images, for example. Page Ref: 303 Difficulty: Difficult Objective: 12.2 Learning Outcome: Discuss strategies for setting and adjusting prices Skill: Concept 83) Which of the following scenarios would allow a firm to price below prevailing market price and still succeed? A) The public does not learn that the firm's product is indeed inferior. B) The firm can offer a product of acceptable quality. C) The firm can offer an entirely new product. D) The firm has effective promotional tactics and marketing strategies. E) The firm can effectively criticize its competitors' products. Answer: B Explanation: B) Budget and Dollar car rental companies following this strategy, for example. Page Ref: 303 Difficulty: Difficult Objective: 12.2 Learning Outcome: Discuss strategies for setting and adjusting prices Skill: Concept 84) Which of the following options best describes an example of price lining? A) having the price of each individual song on an online music store be determined by length of song B) having the price of each individual song on an online music store be determined by popularity of the band C) having the price of each individual song on an online music store be determined by user auction D) having the price of each individual song on an online music store be determined by reverse auction E) having the price of each individual song on an online music store be determined by which of several categories the song falls into Answer: E Explanation: E) With price lining, a retailer predetermines three or four price points at which a particular product will be sold. Page Ref: 304 Difficulty: Difficult AACSB: Analytic skills Objective: 12.2 Learning Outcome: Discuss strategies for setting and adjusting prices Skill: Application 85) Which of the following is most directly related to differences in transportation cost from mode to mode? A) packing necessary for the product B) fuel efficiency of the carrier C) regularity of use of a given carrier D) discount agreements with the carriers E) delivery speed Answer: E Explanation: E) Differences in transportation cost from mode to mode are most directly related to delivery speed. Page Ref: 312 Difficulty: Difficult Objective: 12.6 Learning Outcome: Describe the major components of effective distribution Skill: Concept 86) Marshall is a route salesperson for a beverage company. Which of the following would most likely occupy his work time? A) trade show demonstrations B) sales promotions C) missionary selling D) direct mail advertising E) order processing Answer: E Explanation: E) Since route salespeople have a set of regular customers, their main task is processing orders for inventory. Page Ref: 317 Difficulty: Difficult Objective: 12.7 Learning Outcome: Describe the major components of effective promotion Skill: Application 87) A new Internet provider has entered a very crowded market. Which of the following methods would it most likely use in order to make consumers aware of its services? A) creative selling B) direct marketing C) interactive marketing D) missionary selling E) personal selling Answer: D Explanation: D) This technique is most concerned with promoting the reputation of the company rather than increasing sales. Page Ref: 318 Difficulty: Difficult Objective: 12.7 Learning Outcome: Describe the major components of effective promotion Skill: Application Chapter 13 Information Technology for Business 23) Who has access to intranets generally? A) employees B) suppliers C) customers D) shareholders E) dealers Answer: A Explanation: A) Employees are able to access confidential information on matters related to the conditions of their employment or materials related to the company business. Page Ref: 335 Difficulty: Easy AACSB: Use of information technology Objective: 13.2 Learning Outcome: Identify the different types of technologies used in business and describe their uses Skill: Concept 24) What system allows outsiders limited access to a firm's internal information systems? A) intranets B) firewalls C) extranets D) directories E) satellites Answer: C Explanation: C) An example of limited access is the ability of buyers to enter a system to see what goods are available. Page Ref: 335 Difficulty: Easy AACSB: Use of information technology Objective: 13.2 Learning Outcome: Identify the different types of technologies used in business and describe their uses Skill: Concept 25) What is a computer-based system that allows people to communicate simultaneously from different locations via telephone, e-mail, or video? A) groupware B) a data communication network C) the Internet D) electronic conferencing E) hypertext Answer: D Explanation: D) An advantage of electronic conferencing is that it can eliminate travel time and save money for the company. Page Ref: 336 Difficulty: Easy AACSB: Use of information technology Objective: 13.2 Learning Outcome: Identify the different types of technologies used in business and describe their uses Skill: Concept 26) What feature allows people in remote locations to work simultaneously on one document? A) videoconferencing B) voice mail C) dataconferencing D) digital information service E) text messaging Answer: C Explanation: C) Dataconferencing can be used to work on documents that specify design components and production schedules. Page Ref: 336 Difficulty: Easy AACSB: Use of information technology Objective: 13.2 Learning Outcome: Identify the different types of technologies used in business and describe their uses Skill: Concept 27) What type of network links computers and workstations, usually within one building? A) satellite communication B) global positioning C) personal digital D) wide area E) local area Answer: E Explanation: E) A LAN requires only one computer system with one database and software system. Page Ref: 337 Difficulty: Easy AACSB: Use of information technology Objective: 13.2 Learning Outcome: Identify the different types of technologies used in business and describe their uses Skill: Concept 28) In client-server networks, who is the usual client? A) the computer through which users make requests for information B) the application through which users answer information requests C) the customer who does business online D) the supplier who checks the availability of components E) the network which supports the database and software systems Answer: A Explanation: A) The clients are usually the laptop or desktop computers. Page Ref: 337 Difficulty: Moderate AACSB: Use of information technology Objective: 13.2 Learning Outcome: Identify the different types of technologies used in business and describe their uses Skill: Concept 29) What feature does a firewall provide? A) Internet access B) protection from electrical surges C) connectivity between stations in a LAN D) privacy protection E) links to computers in a WAN Answer: D Explanation: D) A firewall is set up to filter incoming information. Page Ref: 346 Difficulty: Easy AACSB: Use of information technology Objective: 13.2 Learning Outcome: Identify the different types of technologies used in business and describe their uses Skill: Concept 30) Which component directs the computer to the correct resources to use and how to use them? A) an application program B) system software C) an electronic spreadsheet D) a database E) management information systems Answer: B Explanation: B) System software are the programs that tell the computer how to function. Page Ref: 338 Difficulty: Easy AACSB: Use of information technology Objective: 13.2 Learning Outcome: Identify the different types of technologies used in business and describe their uses Skill: Concept 31) Which type of communication technology is attractive to businesses because it eliminates travel expenses? A) VSAT Satellite Communications B) extranets C) intranets D) electronic conferencing E) social networks Answer: D Explanation: D) Business colleagues can interact as if they were in the same room, even when oceans apart, thanks to electronic conferencing technology. Page Ref: 339 Difficulty: Easy AACSB: Use of information technology Objective: 13.2 Learning Outcome: Identify the different types of technologies used in business and describe their uses Skill: Concept 32) Who is the person who is in charge of operating systems for gathering, organizing, and distributing information? A) production manager B) operations manager C) information systems manager D) research and development manager E) communications manager Answer: C Explanation: C) Companies view their information as a private resource and therefore need the services of an information systems manager. Page Ref: 340 Difficulty: Easy AACSB: Use of information technology Objective: 13.3 Learning Outcome: Identify the different types of technologies used in business and describe their uses Skill: Concept 33) What is the name of the application used for searching, sifting, and reorganizing pools of data to uncover useful information? A) data warehousing B) data access C) data tapping D) data cleaning E) data mining Answer: E Explanation: E) Data mining is a powerful tool that aids managers in a range of tasks including costing, planning, and setting production targets. Page Ref: 341 Difficulty: Easy AACSB: Use of information technology Objective: 13.3 Learning Outcome: Identify the different types of technologies used in business and describe their uses Skill: Concept 34) Which cybercriminal activity refers to the act of luring Internet users to bogus Web sites? A) computer virus distribution B) spamming C) phishing D) Trojan horse distribution E) piracy Answer: C Explanation: C) schemes bait Internet users into visiting false Web sites that are able to steal their information. Page Ref: 344 Difficulty: Easy AACSB: Use of information technology Objective: 13.4 Learning Outcome: Identify the different types of technologies used in business and describe their uses Skill: Concept 35) Which term refers to the unauthorized stealing of personal information from a computer system? A) espionage B) leverage C) denial of service D) identity theft E) network flooding Answer: D Explanation: D) Identity theft is the fastest growing crime in the United States. Page Ref: 345 Difficulty: Easy AACSB: Use of information technology Objective: 13.4 Learning Outcome: Identify the different types of technologies used in business and describe their uses Skill: Concept 36) What is "spam"? A) a form of firewall B) intellectual property C) junk e-mail D) a form of hacking E) wireless mooching Answer: C Explanation: C) Spam is a greater problem than postal junk mail because of the low costs of the Internet and its large reach. Page Ref: 346 Difficulty: Easy AACSB: Use of information technology Objective: 13.4 Learning Outcome: Identify the different types of technologies used in business and describe their uses Skill: Concept 37) What is the name of software and hardware that prevent hackers from accessing a company's internal network? A) partition B) firewall C) extranet D) index E) security Answer: B Explanation: B) Firewalls are essential systems that secure a computer network. Page Ref: 346 Difficulty: Easy AACSB: Use of information technology Objective: 13.5 Learning Outcome: Identify the different types of technologies used in business and describe their uses Skill: Concept 38) A local preschool is considering the use of webcams inside of their classrooms. Which statement, if true, would BEST support the claim that this technology would NOT be a good idea? A) Parents prefer to communicate with their children's teachers face-to-face and via phone. B) Some parents want more information on how their children work in class. C) Teachers want feedback and recognition from parents. D) Administration encourages parents to communicate daily with the teachers. E) Teachers send weekly videos and photos of the children working in class. Answer: A Explanation: A) Clients have expressed preferred communication methods (face-to-face and via phone) that are in opposition to webcams (communicating with teachers through video-conferencing). Page Ref: 331 Difficulty: Moderate AACSB: Reflective thinking skills Objective: 13.1 Learning Outcome: Identify the different types of technologies used in business and describe their uses Skill: Application 39) Which of the following BEST describes enterprise resource planning? A) an information processing application for routine business activities of a company's business units B) an information system for organizing and managing a firm's activities across product lines, departments, and geographic locations C) a system used for transforming data into information for use in decision making D) a computer-based electronic technology that assists in designing products by simulating a real product and displaying it in three-dimensional graphics E) an internal inventory of a company's product lines Answer: B Explanation: B) Enterprise resource planning (ERP) is an information system for organizing and managing a firm's activities across products, departments, and geographic locations. Page Ref: 332 Difficulty: Moderate AACSB: Use of information technology Objective: 13.1 Learning Outcome: Identify the different types of technologies used in business and describe their uses Skill: Concept 40) How can customers order a personal computer from the online Dell site with exactly the features they desire? A) through mass production B) through unit production C) through process management D) through mass-customization E) through anti-virus software Answer: D Explanation: D) Although companies can produce in large volumes, each unit features the unique options the customer prefers with mass-customization. Page Ref: 332 Difficulty: Moderate Objective: 13.1 Learning Outcome: Identify the different types of technologies used in business and describe their uses Skill: Application 41) Which of the following best describes an extranet? A) a software and hardware system that prevents outsiders from accessing a company's network B) a tool that searches Web pages containing the user's search terms and then displays pages that match C) a private network of internal Web sites and other sources of information available to a company's employees D) an Internet connection allowing outsiders limited access to a firm's internal information system E) a delivery system linked to production schedules Answer: D Explanation: D) The most common application, for example, allows buyers to enter a system to see which products are available for sale and delivery, thus providing convenient product-availability information. Page Ref: 335 Difficulty: Moderate AACSB: Use of information technology Objective: 13.2 Learning Outcome: Identify the different types of technologies used in business and describe their uses Skill: Concept 42) At Chaparral Steel, delivery times, sales, profits, and customer service and loyalty have been boosted by a system that gives customers electronic access to the mill's inventories. What is the name of this system? A) distribution channel B) intranet C) handbook D) extranet E) product availability Answer: D Explanation: D) Extranets allow outsiders limited access to a firm's internal information network. Page Ref: 335 Difficulty: Moderate AACSB: Use of information technology Objective: 13.2 Learning Outcome: Identify the different types of technologies used in business and describe their uses Skill: Application 43) What system includes data conferencing and videoconferencing? A) electronic communication systems B) electronic conferencing C) group systems D) data communication networks E) data mining Answer: B Explanation: B) Electronic conferencing allows groups of people to communicate simultaneously from various locations via e-mail, phone, or video; two forms of electronic conferencing are videoconferencing and data conferencing. Page Ref: 336 Difficulty: Moderate AACSB: Use of information technology Objective: 13.2 Learning Outcome: Identify the different types of technologies used in business and describe their uses Skill: Concept 44) What advantage does a communication system using VSAT satellite have? A) speed B) low cost C) privacy D) groupware E) spyware Answer: C Explanation: C) A company that operates its own VSAT system has total control over its communications without dependence on other companies. Page Ref: 336 Difficulty: Moderate AACSB: Use of information technology Objective: 13.2 Learning Outcome: Identify the different types of technologies used in business and describe their uses Skill: Concept 45) What does a company need to operate so that it has total control over its communications without dependence on other companies? A) Web site B) VSAT system C) extranet D) groupware site E) enterprise planning Answer: B Explanation: B) One advantage, therefore, is privacy. Page Ref: 336 Difficulty: Moderate AACSB: Use of information technology Objective: 13.2 Learning Outcome: Identify the different types of technologies used in business and describe their uses Skill: Concept 46) What is the most common type of network used in businesses? A) intranet B) client-server C) extranet D) hotspot E) anti-virus Answer: B Explanation: B) Most businesses have laptop or desktop computers through which users make requests for information or resources; servers are the computers that provide the services shared by users. Page Ref: 337 Difficulty: Moderate AACSB: Use of information technology Objective: 13.2 Learning Outcome: Identify the different types of technologies used in business and describe their uses Skill: Concept 47) What is the major limitation of Wi-Fi? A) high cost B) privacy concerns C) productivity D) short range of distance E) installation challenges Answer: D Explanation: D) Each "hotspot," or access point, is actually its own small network; some of these services are free. Page Ref: 338 Difficulty: Moderate AACSB: Use of information technology Objective: 13.2 Learning Outcome: Identify the different types of technologies used in business and describe their uses Skill: Concept 48) What are keyboards, monitors, and printers examples of? A) control B) hardware C) software D) databases E) servers Answer: B Explanation: B) Hardware includes any of the physical components of a computer network. Page Ref: 338 Difficulty: Moderate AACSB: Use of information technology Objective: 13.2 Learning Outcome: Identify the different types of technologies used in business and describe their uses Skill: Concept 49) Which of the following best describes information? A) the useful interpretation of facts and figures B) a list of company names provided by the local Chamber of Commerce C) the collection of raw facts and figures D) controlled flow of words for a useful purpose E) databases and flowcharts Answer: A Explanation: A) Data are raw facts and figures that are transformed into information. Page Ref: 339 Difficulty: Moderate AACSB: Use of information technology Objective: 13.3 Learning Outcome: Identify the different types of technologies used in business and describe their uses Skill: Concept 50) A cell phone store manager needs to forecast sales for the upcoming Black Friday sales weekend. Which information would be MOST helpful? A) the store with the least sales of all store locations for the past five years B) the store with the highest sales of all store locations for the past five years C) the cell phone model which earned the most revenue in all store locations for the past five years D) the total amount spent per customer in each cell phone store for all locations for the past five years E) the number of customers who entered the cell phone store for all locations Answer: D Explanation: D) Customer sales per store over time could be used to forecast how stores would do during the coming year. Page Ref: 339 Difficulty: Moderate AACSB: Analytic skills Objective: 13.2 Learning Outcome: Identify the different types of technologies used in business and describe their uses Skill: Application 51) Who most often needs information in order to conduct technical projects? A) middle managers B) members of the board of directors C) top-level managers D) knowledge workers E) network administrators Answer: D Explanation: D) Knowledge workers are employees for whom information and knowledge are the raw materials of their work—engineers, scientists, and IT specialists. Page Ref: 343 Difficulty: Moderate Objective: 13.3 Learning Outcome: Identify the tools and resources used for managing personal finances Skill: Concept 52) Which software can create products ranging from cell phones to auto parts at lower costs compared to manual modeling methods? A) application software B) operations software C) computer-aided manufacturing software D) computer-aided design software E) data warehousing software Answer: D Explanation: D) Computer-aided design helps knowledge workers design products by simulating them and displaying them in three-dimensional graphics. Page Ref: 343 Difficulty: Moderate AACSB: Use of information technology Objective: 13.3 Learning Outcome: Identify the different types of technologies used in business and describe their uses Skill: Concept 53) Which system supports an organization's managers by providing daily reports, schedules, plans, and budgets? A) management application B) management operations C) management information D) management expert E) management enterprise Answer: C Explanation: C) Many management information systems—cash flow, sales, production scheduling, shipping—help managers find answers to such questions. Page Ref: 343 Difficulty: Moderate Objective: 13.3 Learning Outcome: Identify the different types of technologies used in business and describe their uses Skill: Concept 54) What is a management information system? A) a system that supports managers by providing daily reports, schedules, plans, and budgets that can be used for making decisions B) an easy-access application of an information system designed for instant access by upper-level managers C) an information processing application for use in business activities involving well-defined processing steps D) an interactive system that locates and presents information needed to support the decision-making process E) a repository of information resources related to the collection of data Answer: A Explanation: A) Many management information systems—cash flow, sales, production scheduling, and shipping—help managers find answers to such questions. Page Ref: 343 Difficulty: Moderate AACSB: Use of information technology Objective: 13.3 Learning Outcome: Identify the different types of technologies used in business and describe their uses Skill: Concept 55) A small parochial school uses handwritten timecards and attendance sheets to calculate biweekly payroll and vacation time. Which statement, if true, would BEST support the idea by managers to convert to computer software to maintain employee records? A) The school has multiple types of employees, with different work schedules and different contracts, whose hours are calculated by hand. B) Part-time staffing has been allocated for payroll and human resource function. C) Employees often submit their timecards incorrectly. D) Employees often forget to submit their vacation request forms. E) The existing management is eager to switch to the new system for human resources. Answer: A Explanation: A) This statement indicates the complexity and amount of work involved with using the current system. Page Ref: 343 Difficulty: Moderate Objective: 13.3 Learning Outcome: Identify the different types of technologies used in business and describe their uses Skill: Critical Thinking 56) A small preschool decided to give its teachers access to a portable laptop during daily naptime. Administrators found over time that teacher productivity actually decreased. Which statement would BEST explain why teacher productivity has decreased in spite of the increased computer time? A) Too many children were disruptive during naptime hours. B) Teachers added more tasks to get done and could not complete them all during naptime. C) Teachers could not watch the children and work on the computer at the same time. D) The classroom lacked the necessary office equipment needed to be productive. E) Teachers spent most of the naptime surfing Internet Web sites unrelated to their work assignments. Answer: E Explanation: E) The teachers' actions directly relate to the decrease in productivity. Page Ref: 343 Difficulty: Moderate AACSB: Reflective thinking skills Objective: 13.3 Learning Outcome: Identify the different types of technologies used in business and describe their uses Skill: Critical Thinking 57) Which of the following best describes a decision support system? A) a system that supports an organization's managers by providing daily reports, schedules, plans, and budgets that can be used for making decisions B) a quick-reference, easy-access application of information systems specifically designed for instant access by upper-level managers C) a system of hardware and software that imitates human behavior D) an interactive system that creates virtual business models for a particular kind of decision and tests them with different data to see how they respond E) a processor that plans production and delivery schedules Answer: D Explanation: D) Decision support systems are interactive systems that create virtual business models and test them with different data to see how they respond. Page Ref: 344 Difficulty: Moderate AACSB: Use of information technology Objective: 13.3 Learning Outcome: Identify the different types of technologies used in business and describe their uses Skill: Concept 58) The increase of technology in the workplace has resulted in more employees working at home and telecommuting. Telecommuting employees rely heavily on e-mail communication to complete work tasks. Which statement, if true, BEST supports those who claim that e-mail communication fosters increased misunderstandings? A) The high volume of e-mail discourages employees from interacting with coworkers face to face, thus leading to more misunderstandings. B) Most e-mail communication among workers is polite and to the point. C) People who e-mail frequently don't know how to communicate verbally with others. D) E-mail fosters more nonverbal communications between workers in a company. E) Workers communicate more frequently about work when e-mailing. Answer: A Explanation: A) The effects of the e-mail overload that are commonly experienced by employees can be discouraging and increase misunderstandings. Page Ref: 344 Difficulty: Moderate AACSB: Reflective thinking skills Objective: 13.3 Learning Outcome: Identify the different types of technologies used in business and describe their uses Skill: Critical Thinking 59) What does phishing or pharming involve the use of? A) intellectual property theft B) firewalls that prevent hacking C) viruses sent from one station to another in a LAN D) deceptive e-mails that imitate popular Web sites E) trading of company secrets Answer: D Explanation: D) Phishing and pharming trick computer users into giving out personal information. Page Ref: 345 Difficulty: Moderate AACSB: Use of information technology Objective: 13.4 Learning Outcome: Identify the different types of technologies used in business and describe their uses Skill: Concept 60) In order for a virus to spread from computer to computer, what must it always attach itself to first? A) all available computer memory B) a worm C) a piece of spyware D) a piece of hardware E) a program Answer: E Explanation: E) A virus resides in a file that attaches itself to a program in order to migrate from computer to computer. Page Ref: 346 Difficulty: Moderate AACSB: Use of information technology Objective: 13.4 Learning Outcome: Identify the different types of technologies used in business and describe their uses Skill: Concept 61) Which type of IT threat can spread without a computer user needing to open any software? A) worms B) Trojan horses C) spam D) spyware E) computer viruses Answer: A Explanation: A) Worms are a kind of virus that travel from computer to computer within networked computer systems without your needing to open any software to spread the contaminated file. Page Ref: 346 Difficulty: Moderate AACSB: Use of information technology Objective: 13.4 Learning Outcome: Identify the different types of technologies used in business and describe their uses Skill: Concept 62) What type of users would download spyware? A) users seeking to prevent viruses from attacking B) users who are unaware that their computer activities will be monitored C) users who wish to prevent spam from entering their computer D) users who want easy access to their coworkers' files E) users who are keen to share documents Answer: B Explanation: B) Spyware is downloaded by users who are lured by "free software" offers; once installed, it crawls around to monitor the host's computer activities, gathering e-mail addresses and other important information. Page Ref: 346 Difficulty: Moderate AACSB: Use of information technology Objective: 13.4 Learning Outcome: Identify the different types of technologies used in business and describe their uses Skill: Concept 63) What is a particularly sinister quality of spyware? A) It replicates itself, using up valuable computer memory. B) It often disguises itself as helpful software. C) It consumes bandwidth by flooding user inboxes with junk e-mail. D) It often leads to wireless mooching. E) It is completely legal. Answer: B Explanation: B) Spyware can masquerade as a friendly file available as freeware or shareware. Page Ref: 346 Difficulty: Moderate AACSB: Use of information technology Objective: 13.4 Learning Outcome: Identify the different types of technologies used in business and describe their uses Skill: Concept 64) Which of the following best describes a firewall? A) a software and hardware system that prevents outsiders from accessing a company's internal network B) a tool that searches Web pages containing the user's search terms and then displays pages that match C) a private network of internal Web sites and other sources of information available to a company's employees D) an Internet connection allowing outsiders limited access to a firm's internal information system E) an extranet site that gives customers access to different product lines Answer: A Explanation: A) Firewalls are security systems with special software or hardware devices designed to keep computers safe from hackers. Page Ref: 346-347 Difficulty: Moderate AACSB: Use of information technology Objective: 13.5 Learning Outcome: Identify the different types of technologies used in business and describe their uses Skill: Concept 65) Tamasaki Sushi Bar is located in the Cotswold Shopping Center in a busy uptown business district. It caters to a few art galleries, a theater, and a nearby technical college. Recently, Tamasaki has noticed a decrease in customers around the lunchtime hour, which is one of its primary sales times during the day. Many customers use cell phones and frequent the Internet cafe in the area during the lunch hour. Which statement, if true, could BEST explain the sagging sales experienced by Tamasaki? A) Most of the workers use the lunchtime hour for taking food out instead of eating in. B) The nearby establishments have their own cafeteria for employees and students to use. C) Technically savvy customers take advantage of deals offered via the Web sites and Facebook sites of the deli and bar-grill down the street. D) Limited parking spaces discourage customers from stopping by for lunch. E) College students prefer pizza and burgers for lunch and restaurant food for dinner. Answer: C Explanation: C) This statement indicates the willingness of technically savvy customers to patronize stores with online sales and service. Page Ref: 330 Difficulty: Difficult Objective: 13.1 Learning Outcome: Identify the different types of technologies used in business and describe their uses Skill: Critical Thinking 66) Many hotels have been exploring ways to allow customers to check out without waiting at the front desk upon leaving. This would greatly decrease checkout time and allow the front desk to be used for incoming and existing customers. One idea is to permit customers to pay for their hotel stay online the night before checkout and then to leave their keys in the room. Which statement, if true, would BEST support this new idea for checking out? A) Hotel customers have continually been requesting Internet access in their rooms. B) Hotel rooms now come equipped with free Internet access directly to the hotel's Web site through the television. C) More hotel customers now travel with laptops that have wireless Internet access. D) Most hotel checkout counters have additional services for customers' needs during their stay. E) Customers are willing to pay for Internet access through wireless and DSL connections. Answer: B Explanation: B) Existing technology and customers' use of television can facilitate online checkout by the greatest number of customers. Page Ref: 330 Difficulty: Difficult AACSB: Reflective thinking skills Objective: 13.1 Learning Outcome: Identify the different types of technologies used in business and describe their uses Skill: Critical Thinking 67) A small preschool collects its tuition payments from parents by automated bank drafts each month. Administrators at the school are considering online tuition collections through the use of a secure logon on their school Web site. Which statement, if true, would BEST support the idea of online tuition collections? A) The Web site already has a wealth of information on it that parents use each day. B) Parents are highly technically savvy and prefer the convenience of using the school's Web site for other functions like enrollment and admissions. C) School newsletters and flyers went paperless a few years ago. D) A small group of parents has expressed interest in paying their bills online. E) The teachers have also expressed a need to go digital with paperwork. Answer: B Explanation: B) The parents are willing to switch and have been receptive to using technology in related functions such as enrollment and admissions. Page Ref: 331 Difficulty: Difficult AACSB: Reflective thinking skills Objective: 13.1 Learning Outcome: Identify the different types of technologies used in business and describe their uses Skill: Critical Thinking 68) Why might businesses want to analyze data that reveal what consumers do in the marketplace? A) to be able to send them tailor-made advertisements B) to be able to close the customer service department C) to be able to reduce production costs D) to be able to stabilize employee turnover E) to be able to manage long-term costs Answer: A Explanation: A) Understanding how consumers act in the marketplace can provide enough information that the marketer can determine which specific media options and types of advertisements will be most appealing to specific consumer groups. Page Ref: 339 Difficulty: Moderate AACSB: Use of information technology Objective: 13.3 Learning Outcome: Describe the major components of effective promotion Skill: Concept 69) Kinsey Lighting, Inc. is a small retailer of decorative lamps and accessories for commercial businesses and residential homes. The company has been family-owned for two decades and has always advertised its business through local media outlets such as newspapers, yellow pages, and occasional radio spots. Recently, Kinsey has been talking with a Web designer about developing a company Web site. Which statement, if true, would BEST support Kinsey's decision to create a Web site for the business? A) There have been more younger families moving into homes in the area. B) The local business association has an online directory for its business partners. C) A new Web site business has opened up across the street from the lamp store. D) Online sales of similar businesses in the area have increased substantially over the years. E) A neighborhood supplier of lamp products just moved to another county. Answer: D Explanation: D) The positive online selling experiences of similar businesses can indicate success for this particular business with the same customers in the area. Page Ref: 335 Difficulty: Difficult AACSB: Reflective thinking skills Objective: 13.1 Learning Outcome: Identify the different types of technologies used in business and describe their uses Skill: Critical Thinking 70) A supermarket is considering automated checkouts that would allow customers to scan the prices of their foods while shopping. Scanners would be placed around the store in different locations. Which statement, if true, would BEST support the claim that this idea would work better than a floating checkout idea? A) There are good locations in the meat, dairy, and bakery aisles to support scanners. B) Scanners would be operated by customers during peak times. C) Portable scanners require yearly training for staff and customers. D) Handheld scanner technology doesn't require extra resources to operate and is virtually self-managed. E) The scanners are not portable and hence cannot be broken by customers. Answer: D Explanation: D) The scanner technology would have minimal cost to operate on a daily basis. Page Ref: 338 Difficulty: Difficult AACSB: Reflective thinking skills Objective: 13.2 Learning Outcome: Identify the different types of technologies used in business and describe their uses Skill: Application 71) Black Friday is well-known as the start of the end-of-the-year holiday shopping season in the U.S. retail market. Stores offer a variety of up-to-the-minute deals to attract customers into the store between certain hours. Customers have begun to utilize electronic PDAs and cell phones to keep up with the myriad sales and deals in the stores. Which condition would MOST threaten customers' use of mobile technology while shopping in stores? A) Customers, upon entering stores, were body-scanned with magnetic detectors. B) Store sales were broadcast over loudspeakers. C) There were a limited number of Wi-Fi transmission hotspots inside the stores. D) Lighting inside of stores was too dim. E) Store personnel were not as helpful to customers with smartphones in their hands. Answer: C Explanation: C) Limited transmission ability would greatly hinder the use of the mobile devices while customers were shopping in stores. Page Ref: 338 Difficulty: Difficult AACSB: Reflective thinking skills Objective: 13.2 Learning Outcome: Identify the different types of technologies used in business and describe their uses Skill: Application 72) Home Garden Center uses mobile customer service representatives in all of its stores. These floating personnel walk the huge warehouse with smart phones answering customer questions, pointing out locations of items, and directing customers to checkout. Which condition would MOST threaten the use of the mobile customer service at Home Garden Centers in the next business season? A) Incoming employees seem less technically savvy with mobile devices. B) Several store departments are swapping locations within the store facility. C) The size of the smartphone is causing some employees to continually drop the device. D) The current model of smartphone uses an operating system that no longer supports the store's customer service application program. E) There is a slight increase in the theft rate of smartphones from the stores. Answer: D Explanation: D) Upgrading the operating system on the smartphones would be a huge incurred cost for the company. Page Ref: 338 Difficulty: Difficult AACSB: Reflective thinking skills Objective: 13.3 Learning Outcome: Identify the different types of technologies used in business and describe their uses Skill: Application 73) Which piece of information would be MOST helpful for college officials to determine whether the latest college open house was a success? A) The total number of attendees indicated more interest than in the past five years. B) The percentage of people interested in enrolling within six months was 20 percent. C) The number of people who attended was the highest in the past five years. D) The number of enrollment applications received was 80 percent of the total number of attendees. E) The number of attendees showing interest was high. Answer: D Explanation: D) A high percentage of the attendees applied for enrollment. Page Ref: 341 Difficulty: Difficult Objective: 13.3 Learning Outcome: Describe the major components of effective promotion Skill: Application 74) Which piece of information would be MOST helpful for executives at Sanmina-Sci, Inc. to determine whether they should close their service and repair facilities during the last week of the year? A) The employees of the facility have more vacation time toward the end of the year. B) The amount of service and repair calls in the facility is at the lowest levels during that week. C) New equipment deliveries are at their lowest prior to that week. D) The amount of work completed by employees prior to that week is the highest in the past year. E) The businesses that need equipment repaired from Sanmina-Sci are closed for holidays during that week. Answer: B Explanation: B) The lack of work during that week would warrant closing the company. Page Ref: 341 Difficulty: Difficult AACSB: Reflective thinking skills Objective: 13.3 Learning Outcome: Identify the different types of technologies used in business and describe their uses Skill: Application 75) Sally wants to determine any changes in the plant's performance based on an estimated 20 percent increase in sales. What device will Sally most likely use? A) decision support system (DSS) B) groupware application C) spyware application D) CAD system E) enterprise resource planning (ERP) Answer: A Explanation: A) A DSS is an interactive system that creates virtual business models and tests them with different data to see how they respond; in other words, the managers can look at "what-if" scenarios by inputting various pieces of data and then studying the system's results. Page Ref: 344 Difficulty: Moderate Objective: 13.3 Learning Outcome: Identify the different types of technologies used in business and describe their uses Skill: Application 76) Greg feels comfortable that no one can open and read his e-mails. What does Greg have installed in his computer? A) anti-virus software B) spyware C) encryption software D) a Trojan horse E) a worm Answer: C Explanation: C) The encryption system works by locking an e-mail message to a unique code number for each computer so only that computer, not others, can open and read the message. Page Ref: 347 Difficulty: Difficult Objective: 13.5 Learning Outcome: Identify the different types of technologies used in business and describe their uses Skill: Application Chapter 14 The Role of Accountants and Accounting Information 33) Which of the following terms refers to the comprehensive system for collecting, analyzing, and communicating financial information? A) bookkeeping B) accounting C) controlling D) auditing E) budgeting Answer: B Explanation: B) Accounting is the practice of making financial information useful to internal and external parties. Page Ref: 358 Difficulty: Easy Objective: 14.1 Learning Outcome: Identify the basic concepts of and tools used for business accounting Skill: Concept 34) Which of the following terms refers to the recording of financial transactions? A) bookkeeping B) accounting C) controlling D) budgeting E) auditing Answer: A Explanation: A) Bookkeeping is the accurate recording of transactions, which accountants use to generate useful information. Page Ref: 358 Difficulty: Easy Objective: 14.1 Learning Outcome: Identify the basic concepts of and tools used for business accounting Skill: Concept 35) Which of the following terms refers to the examination of a company's accounting system to determine whether its financial reports fairly represent its operations? A) company-wide analysis B) forensic accounting C) portfolio analysis D) audit E) preliminary investigation Answer: D Explanation: D) An audit is an examination by an external party to ensure that a company's financial reports are faithful and accurate. Page Ref: 359 Difficulty: Easy Objective: 14.1 Learning Outcome: Identify the basic concepts of and tools used for business accounting Skill: Concept 36) Which of the following terms refers to any economic resource that is expected to benefit a firm or individual who owns it? A) asset B) liability C) equity D) account E) deposit Answer: A Explanation: A) An asset counts as something of value for accounting purposes. Page Ref: 363 Difficulty: Easy Objective: 14.2 Learning Outcome: Identify the basic concepts of and tools used for business accounting Skill: Concept 37) Which of the following terms refers to the amount of money that owners would receive if they sold all of a company's assets and paid all of its liabilities? A) asset B) owners' equity C) inventory D) liability E) credit Answer: B Explanation: B) Owners' equity is that value that represents the financial stake in a company by the owners. Page Ref: 364 Difficulty: Easy Objective: 14.2 Learning Outcome: Identify the basic concepts of and tools used for business accounting Skill: Concept 38) Which of the following refers to the financial statement detailing a firm's assets, liabilities, and owners' equity? A) income statement B) statement of cash flows C) balance sheet D) expense report E) annual budget Answer: C Explanation: C) A balance sheet uses the accounting equation to show the current value of a company. Page Ref: 364 Difficulty: Easy Objective: 14.3 Learning Outcome: Identify the basic concepts of and tools used for business accounting Skill: Concept 39) Which of the following describes assets such as land, building, and equipment? A) liquid assets B) fixed assets C) intangible assets D) current assets E) limited assets Answer: B Explanation: B) Fixed assets are tangible things of value. Page Ref: 364 Difficulty: Easy Objective: 14.3 Learning Outcome: Identify the basic concepts of and tools used for business accounting Skill: Concept 40) Which of the following best describes depreciation? A) the process of the decreasing value of fixed assets due to use B) the process of deducting operating expenses from the value of fixed assets C) the process of reducing the value of fixed assets equally over time D) the process of distributing the cost of liabilities equally over time E) the process of distributing the cost of intangible assets equally over time Answer: C Explanation: C) Depreciation is the natural reduction of value of certain assets over time. Page Ref: 365 Difficulty: Easy Objective: 14.3 Learning Outcome: Identify the basic concepts of and tools used for business accounting Skill: Concept 41) Which of the following describes assets such as patents and trademarks? A) liquid assets B) fixed assets C) intangible assets D) current assets E) limited assets Answer: C Explanation: C) Intangible assets are nonphysical items that still have monetary value attached to them. Page Ref: 364 Difficulty: Easy Objective: 14.3 Learning Outcome: Identify the basic concepts of and tools used for business accounting Skill: Concept 42) Which of the following terms refers to the amount paid for an existing business beyond the value of its other assets? A) goodwill B) excess compensation C) licensing D) asset promotion E) liability deduction Answer: A Explanation: A) Goodwill is the intangible value of the expectation that a company will continue to do business wisely. Page Ref: 366 Difficulty: Easy Objective: 14.3 Learning Outcome: Identify the basic concepts of and tools used for business accounting Skill: Concept 43) Which of the following terms refers to the difference between operating income and income taxes ? A) gross revenue B) net income C) gross profit D) cash flow E) leverage Answer: B Explanation: B) Net income is operating income minus taxes. Page Ref: 368 Difficulty: Easy Objective: 14.3 Learning Outcome: Identify the basic concepts of and tools used for business accounting Skill: Concept 44) Which of the following ratios measures a firm's ability to meet its current debt obligations? A) short-term solvency ratio B) long-term solvency ratio C) profitability ratio D) activity ratio E) equity ratio Answer: A Explanation: A) The short-term solvency ratio is calculated by dividing current assets by current liabilities. Page Ref: 372 Difficulty: Moderate Objective: 14.5 Learning Outcome: Identify the basic concepts of and tools used for business accounting Skill: Concept 45) Which of the following ratios measures a firm's potential earnings? A) short-term solvency ratio B) long-term solvency ratio C) profitability ratio D) activity ratio E) equity ratio Answer: C Explanation: C) This ratio is calculated by dividing net income by the number of common shares of stock. Page Ref: 373 Difficulty: Moderate Objective: 14.5 Learning Outcome: Identify the basic concepts of and tools used for business accounting Skill: Concept 46) ABC Company has a net income of $5 million. ABC's number of outstanding shares of stock is 3,870,000 shares. What is the company's earnings per share? A) $1.29 B) $.77 C) $1.25 D) $.80 E) $18.75 Answer: A Explanation: A) Earnings per share = Net Income/Number of Common Shares Outstanding; therefore, in this scenario, $5,000,000/3,870,000 = $1.29. Page Ref: 373 Difficulty: Easy AACSB: Analytic skills Objective: 14.5 Learning Outcome: Identify the basic concepts of and tools used for business accounting Skill: Application 47) Which of the following ratios evaluates management's use of resources? A) short-term solvency ratio B) long-term solvency ratio C) activity ratio D) profitability ratio E) equity ratio Answer: C Explanation: C) The activity ratio is used to compare relative efficiencies between similar companies. Page Ref: 374 Difficulty: Moderate Objective: 14.5 Learning Outcome: Identify the basic concepts of and tools used for business accounting Skill: Concept 48) Which are the two major fields of accounting? A) managerial and forensic B) financial and managerial C) bookkeeping and advisory D) corporate and individual E) public and private Answer: B Explanation: B) The two fields of accounting can be classified according to the users they serve; it is convenient and accurate to classify users as those outside the company and those inside the company. Page Ref: 358-359 Difficulty: Easy Objective: 14.1 Learning Outcome: Identify the basic concepts of and tools used for business accounting Skill: Concept 49) Bennequin Accounting has been hired by XYZ Manufacturing to help plan its merger with ABC Distribution. Which of the following terms best describes the capacity for which Bennequin has been hired? A) regulatory accounting B) corporate bookkeeping C) company auditing D) fraud investigation E) management advisory Answer: E Explanation: E) In this capacity, private accountants help management make decisions. Page Ref: 361 Difficulty: Moderate Objective: 14.1 Learning Outcome: Identify the basic concepts of and tools used for business accounting Skill: Application 50) Which of the following statements is true? A) As common practice, a union representative for the workers at a firm receives information from the firm's financial accounting system. B) As common practice, a union representative for the workers at a firm receives information from the firm's managerial accounting system. C) As common practice, a union representative for the workers at a firm receives information from both the firm's managerial accounting system and its financial accounting system. D) As common practice, a union representative for the workers at a firm receives information from the firm's forensic accounting system. E) As common practice, a union representative for the workers at a firm does not receive any information from the firm's accounting system. Answer: A Explanation: A) The financial accounting system is concerned with external users, such as unions. Page Ref: 358 Difficulty: Moderate Objective: 14.1 Learning Outcome: Identify the basic concepts of and tools used for business accounting Skill: Application 51) Which of the following statements is true? A) As common practice, a sales representative for a firm receives information from the firm's financial accounting system. B) As common practice, a sales representative for a firm receives information from the firm's managerial accounting system. C) As common practice, a sales representative for a firm receives information from both the firm's managerial accounting system and its financial accounting system. D) As common practice, a sales representative for a firm receives information from the firm's revenue accounting system. E) As common practice, a sales representative for a firm does not receive any information from the firm's accounting system. Answer: B Explanation: B) The managerial accounting system is concerned with internal users. Page Ref: 358 Difficulty: Moderate Objective: 14.1 Learning Outcome: Identify the basic concepts of and tools used for business accounting Skill: Application 52) Due to investor disgruntlement, Howard Industries needs to provide a complete review of its financial records in order to confirm that they have been conforming with generally accepted accounting principles. Which of the following processes does Howard Industries need to start? A) company-wide analysis B) company-wide audit C) portfolio analysis D) preliminary investigation E) fraud investigation Answer: B Explanation: B) An audit examines a company's accounting information system to determine whether financial reports reliably represent its operations. Page Ref: 359 Difficulty: Moderate Objective: 14.1 Learning Outcome: Identify the basic concepts of and tools used for business accounting Skill: Application 53) Which of the following refers to the rules and procedures governing the content and form of financial reports? A) national bookkeeping standards B) core competencies for accounting C) accounting and financial rules D) national guidelines of accounting principles E) generally accepted accounting principles Answer: E Explanation: E) GAAP is formulated by the Financial Accounting Standards Board (FASB) of the AICPA and should be used to determine whether a firm has controls to prevent errors and fraud. Page Ref: 359 Difficulty: Moderate Objective: 14.1 Learning Outcome: Identify the basic concepts of and tools used for business accounting Skill: Concept 54) Jose is a recent college graduate who has been hired for an entry-level professional position in the accounting department of a large corporation. Which of the following terms best describes Jose's capacity as an accountant? A) public accountant B) private accountant C) forensic accountant D) advisory accountant E) management accountant Answer: B Explanation: B) Private accountants are hired by individual businesses to perform day-to-day activities. Page Ref: 361 Difficulty: Moderate Objective: 14.1 Learning Outcome: Identify the basic concepts of and tools used for business accounting Skill: Application 55) Which of the following describes a liability? A) the potential profit from selling a firm's assets and settling all of its debts B) a debt owed by a firm to an outside organization or individual C) any economic resource expected to benefit a firm or an individual who owns it D) any economic deficit expected to cost a firm or an individual who owns it E) the amount of money originally invested in a business by its owners Answer: B Explanation: B) All debts owed to others are liabilities. Page Ref: 363 Difficulty: Moderate Objective: 14.2 Learning Outcome: Identify the basic concepts of and tools used for business accounting Skill: Concept 56) Which of the following statements best describes the difference between current and long-term liabilities? A) Current liabilities are debts that need to be paid immediately, whereas long-term liabilities do not. B) Current liabilities are those which will cost less in debt interest than long-term liabilities. C) Current liabilities are debts that are settled sooner than long-term debts. D) Current liabilities are debts on tangible assets, whereas long-term liabilities are debts on intangible assets. E) Current liabilities are debts on current assets, whereas long-term liabilities are debts on fixed and intangible assets. Answer: C Explanation: C) Current liabilities include accounts payable: unpaid bills to suppliers for materials as well as wages and taxes that must be paid in the coming year. Long-term liabilities are debts that are not due for at least a year. Page Ref: 366 Difficulty: Moderate Objective: 14.3 Learning Outcome: Identify the basic concepts of and tools used for business accounting Skill: Concept 57) A company's owners invested $10,000 total into the company, which made a profit of $3,500 last year. Its total liabilities are currently at $7,000. What is the value of the total assets of this company? A) $3,000 B) $3,500 C) $6,500 D) $13,500 E) $27,500 Answer: C Explanation: C) 6500 = (10,000 + 3500) - 7000 Page Ref: 366 Difficulty: Moderate AACSB: Analytic skills Objective: 14.2 Learning Outcome: Identify the basic concepts of and tools used for business accounting Skill: Application 58) A company has an assets-to-liabilities ratio of 3:2 and total assets worth $6 million. What is its owners' equity? A) $1 million B) $2 million C) $3 million D) $9 million E) $12 million Answer: B Explanation: B) assets/(assets - owners' equity) = 3/2 Page Ref: 366 Difficulty: Moderate AACSB: Analytic skills Objective: 14.2 Learning Outcome: Identify the basic concepts of and tools used for business accounting Skill: Application 59) How much profit does a company which has a total of $5 million invested by its owners and $4 million in liabilities need to make in order to have an assets-to-liabilities ratio of 2:1? A) $2 million B) $3 million C) $5 million D) $7 million E) $8 million Answer: D Explanation: D) (profits + investment - liabilities)/liabilities = 2 Page Ref: 366 Difficulty: Difficult AACSB: Analytic skills Objective: 14.2 Learning Outcome: Identify the basic concepts of and tools used for business accounting Skill: Application 60) What distinguishes the two most commonly used categories of owners' equity? A) the amount of equity within each type of asset—current versus fixed and intangible B) the financial form of the equity—cash investments versus stock and bond investments C) the source of the equity—investments versus earnings D) the use of the equity—invested versus saved E) the age of the equity—older investments and earnings versus newer investments and earnings Answer: C Explanation: C) Paid-in capital is additional money invested by owners; retained earnings are net profits kept by a firm rather than paid out as dividend payments to stockholders. Page Ref: 366 Difficulty: Moderate Objective: 14.3 Learning Outcome: Identify the basic concepts of and tools used for business accounting Skill: Concept 61) What are the major categories within an income statement? A) operating costs and profits, investing costs and profits, and financing costs and profits B) current revenue, long-term revenue, and net revenue C) revenue, sales, and income D) assets, liabilities, and owners' equity E) revenues, cost of revenues, operating expenses, and net income Answer: E Explanation: E) An income statement shows the financial results that occurred during a period of time, such as a month or a quarter or a year. Page Ref: 366 Difficulty: Moderate Objective: 14.3 Learning Outcome: Identify the basic concepts of and tools used for business accounting Skill: Concept 62) Which of the following is the difference between revenues and the cost of revenues? A) net income B) gross profit C) accounts payable D) interest expense E) cash flow Answer: B Explanation: B) Managers are often interested in gross profit, a preliminary profit figure that considers revenues and cost of revenues from the income statement. Page Ref: 367 Difficulty: Easy Objective: 14.3 Learning Outcome: Identify the basic concepts of and tools used for business accounting Skill: Concept 63) Which of the following best illustrates cash flow from operations? A) the revenue from selling bread at a local bakery B) the revenue from selling stocks by an insurance company C) the revenue from new investors in a software start-up D) the revenue from selling obsolescent trucks by a delivery company E) the revenue from issuing stock by a financial management firm Answer: A Explanation: A) This figure represents how much of the year's cash balance results from the firm's main line of business. Page Ref: 368 Difficulty: Moderate Objective: 14.3 Learning Outcome: Identify the basic concepts of and tools used for business accounting Skill: Application 64) Which of the following documents would most likely come from a planning strategy meeting? A) income statement B) flow of cash statement C) balance sheet D) budget E) statement of projected earnings Answer: D Explanation: D) A budget is the most important internal financial statement. Page Ref: 369 Difficulty: Moderate Objective: 14.3 Learning Outcome: Identify the basic concepts of and tools used for business accounting Skill: Concept 65) A company brings in $3.25 million in revenue and has a revenue cost of $1 million. Which of the following statements is true? A) In order to have a net income of more than $1.5 million, the operating expenses and income taxes need to be less than $0.25 million. B) In order to have a net income of more than $1.5 million, the operating expenses and income taxes need to be less than $0.75 million. C) In order to have a net income of more than $1.5 million, the operating expenses and income taxes need to be less than $1.75 million. D) In order to have a net income of more than $1.5 million, the operating expenses and income taxes need to be less than $2.25 million. E) In order to have a net income of more than $1.5 million, the operating expenses and income taxes need to be less than $2.75. Answer: B Explanation: B) operating expenses + taxes = net - gross profit Page Ref: 369 Difficulty: Moderate AACSB: Analytic skills Objective: 14.3 Learning Outcome: Identify the basic concepts of and tools used for business accounting Skill: Application 66) A company ended the year with $4.5 million in cash. During this year, it had earned $2.4 million in operating and financing activities. Which of the following statements is necessarily true? A) The company started the year with less than $2.1 million. B) The company started the year with more than $2.1 million. C) The company started the year with $2.1 million minus the amount of investing activities during the year. D) The company started the year with $2.1 million plus the amount of investing activities during the year. E) The company started the year with $4.5 million in cash. Answer: D Explanation: D) net cash flow = start + revenue - all costs Page Ref: 368 Difficulty: Moderate AACSB: Analytic skills Objective: 14.3 Learning Outcome: Identify the basic concepts of and tools used for business accounting Skill: Application 67) Which of the following terms refers to the formal recording and reporting of revenues in financial statements? A) full disclosure B) revenue recognition C) compliance D) ethical practice E) materiality Answer: B Explanation: B) Revenue recognition is the formal recording and reporting of revenues at the appropriate time; although a firm earns revenues continuously as it makes sales, earnings are not reported until the earnings cycle is completed. Page Ref: 370 Difficulty: Easy Objective: 14.4 Learning Outcome: Identify the basic concepts of and tools used for business accounting Skill: Concept 68) Which of the following are the three categories on a company's cash flow statement? A) operating activities, purchase activities, taxes B) profit inflow, debt outflow, taxes C) profitable activities, deficit activities, taxes D) operating activities, investing activities, taxes E) operating activities, investing activities, financing activities Answer: E Explanation: E) Those are the three main figures by which net cash flow is calculated. Page Ref: 368 Difficulty: Easy Objective: 14.4 Learning Outcome: Identify the basic concepts of and tools used for business accounting Skill: Concept 69) Financial statements that do not include interpretation and explanation by management are in violation of which GAAP principle? A) principle of sincerity B) principle of continuity C) principle of full disclosure D) principle of regularity E) principle of prudence Answer: C Explanation: C) Because they know about events inside the company, the people in management prepare additional information to explain certain events or transactions or to disclose the circumstances behind certain results. Page Ref: 370 Difficulty: Moderate Objective: 14.4 Learning Outcome: Identify the basic concepts of and tools used for business accounting Skill: Concept 70) A large company specializing in bread baking has made its annual balance sheet available to its investors. Profits were slightly lower than expected this year. Which of the following items does NOT need to be included in this statement in order to fulfill the principle of full disclosure? A) The current rise of local micro-bakeries has not affected and probably will not affect the revenue of the company. B) The company is contemplating a merger with another major baking company. C) Much of the loss of profits can be explained by the sudden rise in the price of wheat during the past year. D) A baking start-up company founded by some former executives of the company looks like it will be a major competitor in the upcoming years. E) The company and the union for its workers were able to halt a labor strike by negotiating increased health benefits for its workers, causing a slight increase in operating costs. Answer: B Explanation: B) Since this would be a company secret at this point and not directly affect costs or revenues from the previous year, this would be unnecessary (and probably unwise) to reveal. Page Ref: 370 Difficulty: Moderate Objective: 14.4 Learning Outcome: Identify the basic concepts of and tools used for business accounting Skill: Application 71) A company ends the year with a banker's ratio of 5.3. How much current liabilities does it have if its current asset total is $15.3 million? A) $3.1 million B) $5.1 million C) $10.1 million D) $10.2 million E) $15.1 million Answer: A Explanation: A) current assets/current liabilities = 5.3 Page Ref: 372 Difficulty: Moderate AACSB: Analytic skills Objective: 14.4 Learning Outcome: Identify the basic concepts of and tools used for business accounting Skill: Application 72) A company ends the year with a net income of $7.5 million and an earnings per share value of $1.50. What is the number of common shares outstanding for this company? A) 3 million B) 4.5 million C) 5 million D) 6.5 million E) 11.25 million Answer: C Explanation: C) 1.5 = net income / # of shares Page Ref: 373 Difficulty: Moderate Objective: 14.4 Learning Outcome: Identify the basic concepts of and tools used for business accounting Skill: Application 73) A local bakery is suddenly inundated with many more orders for wedding cakes than it has the physical capacity to make. Rather than turn away customers, the owner decides to rent another kitchen across town to fulfill all of the orders. But she needs to borrow money to pay for the rent and additional workers, the additional cost of which will be paid by the profits from the additional cakes. Which of the following ratios will be most affected by this decision? A) the bakery's short-term solvency ratio B) the bakery's long-term solvency ratio C) the bakery's profitability ratio D) the bakery's activity ratio E) the bakery's equity ratio Answer: A Explanation: A) Since this involves a short-term loan with no additional assets, this ratio will temporarily go down. Page Ref: 372 Difficulty: Moderate Objective: 14.5 Learning Outcome: Identify the basic concepts of and tools used for business accounting Skill: Application 74) Which of the following bodies formulates the generally accepted accounting principles ? A) Accounting Disclosure Board B) Financial Accounting Standards Board C) Association of Certified Fraud Examiners D) Securities and Exchange Commission E) American Institute of Certified Public Accountants Answer: B Explanation: B) GAAPs are formulated by the Financial Accounting Standards Board (FASB) of the AICPA and should be used to determine whether a firm has controls to prevent errors and fraud. Page Ref: 359 Difficulty: Moderate Objective: 14.1 Learning Outcome: Identify the basic concepts of and tools used for business accounting Skill: Concept 75) Which of the following bodies enforces accounting and auditing rules and procedures? A) Accounting Disclosure Board B) Financial Accounting Standards Board C) Association of Certified Fraud Examiners D) Securities and Exchange Commission E) American Institute of Certified Public Accountants Answer: D Explanation: D) The Securities and Exchange Commission, a government regulatory agency, requires firms to file such disclosures so that potential investors have valid information about their financial status. Page Ref: 359 Difficulty: Moderate Objective: 14.1 Learning Outcome: Identify the basic concepts of and tools used for business accounting Skill: Concept 76) Which of the following statements is true? A) Forensic accounting is mostly used as a tool for government regulation and investigation. B) Forensic accounting is mostly used as a tool for intracompany investigations. C) Forensic accounting is rarely used to investigate the misuse of government funds. D) Forensic accounting is often used to investigate the misuse of government funds. E) Forensic accounting is mostly used as a tool for local police enforcement. Answer: D Explanation: D) Forensic accountants may be called upon for both investigative accounting and litigation support in crimes against companies, crimes by companies, and civil disagreements. Page Ref: 361 Difficulty: Moderate Objective: 14.1 Learning Outcome: Identify the basic concepts of and tools used for business accounting Skill: Concept 77) Which of the following BEST describes a certified fraud examiner? A) a specialized managerial accountant concerned with issues of fraud B) a specialized certified public accountant concerned with issues of fraud C) a specialized forensic accountant concerned with issues of fraud D) an accounting agent for the Securities and Exchange Commission concerned with issues of fraud E) an accounting agent provided by the Sarbanes-Oxley Act of 2002 concerned with issues of fraud Answer: C Explanation: C) The CFE's activities focus specifically on fraud-related issues: fraud detection, evaluating accounting systems for weaknesses and fraud risks, investigating white collar crime on behalf of law enforcement agencies, evaluating internal organizational controls for fraud prevention, and expert witnessing. Page Ref: 362 Difficulty: Moderate Objective: 14.1 Learning Outcome: Identify the basic concepts of and tools used for business accounting Skill: Concept 78) Which of the following areas of the Certified Fraud Examiner (CFE) exam includes theories of fraud prevention? A) criminology and ethics B) financial transactions C) fraud investigation D) fraud litigation E) legal elements of fraud Answer: A Explanation: A) The criminology and ethics section includes theories of fraud prevention and ethical situations. Page Ref: 362 Difficulty: Moderate Objective: 14.1 Learning Outcome: Identify the basic concepts of and tools used for business accounting Skill: Concept 79) Which of the following inequalities is always true? A) liabilities + assets < assets + owners' equity B) liabilities < assets - owners' equity C) assets < owners' equity + liabilities D) owners' equity < assets - liabilities E) assets < owners' equity - liabilities Answer: E Explanation: E) Since assets = owners' equity + liabilities, this is always true. Page Ref: 364 Difficulty: Difficult AACSB: Analytic skills Objective: 14.2 Learning Outcome: Identify the basic concepts of and tools used for business accounting Skill: Concept 80) Bennequin Manufacturing was a highly profitable firm five years ago but has since been severely declining in net revenue, even though the amount of business has held steady. Marlena has been hired as the new Chief Executive Officer to help turn things around for the company. She would like to understand what happened financially to the company in the past five years in order to start coming up with possible solutions. Which of the following documents would be most useful for her? A) income statements from the past five years B) debt statements from the past five years C) statements of cash flows from the past five years D) budgets from the past five years E) balance sheets from the past five years Answer: E Explanation: E) All of these might be helpful. But, since balance sheets give a detailed accounting of the financial condition at a particular time, by comparing several in a row Marlena might be able to pinpoint more accurately what is causing Bennequin's problems. Page Ref: 364 Difficulty: Difficult AACSB: Reflective thinking skills Objective: 14.3 Learning Outcome: Identify the basic concepts of and tools used for business accounting Skill: Application 81) Which of the following scenarios would most likely occur with a company that has reported disappointing earnings recently but still looks to be financially stable for quite some time? A) high long-term solvency ratio but low profitability ratio B) high short-term solvency ratio but low profitability ratio C) high long-term solvency ratio but low activity ratio D) high short-term solvency ratio but low activity ratio E) high long-term solvency ratio but low short-term solvency ratio Answer: A Explanation: A) Many of these situations may apply, but this one is the most certain. Page Ref: 372-373 Difficulty: Difficult AACSB: Analytic skills Objective: 14.5 Learning Outcome: Identify the basic concepts of and tools used for business accounting Skill: Synthesis 82) With fixed assets of $4 billion and current assets of $1.7 billion, Realm Company has long-term liabilities of $2 billion and current liabilities of $0.34 billion. What is Realm's current ratio? A) 0.85 B) 2 C) 2 .44 D) 5 E) 11.76 Answer: D Explanation: D) The current ratio = current assets/current liabilities; in this scenario, $1.7/.34 = 5. Page Ref: 372 Difficulty: Difficult AACSB: Analytic skills Objective: 14.5 Learning Outcome: Identify the basic concepts of and tools used for business accounting Skill: Application 83) Which organization provides guidelines for ethical conduct for the accounting profession? A) AICPA B) SEC C) GAAP D) FASB E) CMA Answer: A Explanation: A) The code of professional conduct for public accountants in the United States is maintained and enforced by the AICPA (American Institute of Certified Public Accountants). Page Ref: 374 Difficulty: Easy Objective: 14.6 Learning Outcome: Identify the basic concepts of and tools used for business accounting Skill: Concept 84) Which of the following statements BEST describes the need for global accounting standards? A) The complexity of the global market increases the risk of unethical accounting behavior by allowing accountants to choose which national GAAP standards are best for their clients. B) The rise of the global market increases the risk of money laundering and offshore accounting. C) The rise of the global market increases the risk of fraudulent business transactions across borders. D) The rise of the global market increases the risk of international dumping and profit-skimming. E) The rise of the global market increases the risk of the creation of dummy companies and accounts for the purpose of dumping toxic assets. Answer: A Explanation: A) All of these statements are true, but this one is the only one that is best addressed by an international code of conduct. Page Ref: 376 Difficulty: Difficult AACSB: Reflective thinking skills Objective: 14.6 Learning Outcome: Identify the basic concepts of and tools used for business accounting Skill: Application 85) Which of the following is NOT an unethical or illegal accounting action? A) hiding expenses B) underreporting liabilities C) overestimating depreciation value of equipment D) inflating revenues E) maximizing the value of a company's intangible assets Answer: E Explanation: E) This is not illegal nor unethical as long as it is done legitimately. Page Ref: 375 Difficulty: Difficult AACSB: Ethical understanding and reasoning abilities Objective: 14.6 Learning Outcome: Identify the basic concepts of and tools used for business accounting Skill: Concept Chapter 15 Money and Banking 34) Which of the following has a fixed term? A) demand deposits B) time deposits C) money market mutual funds D) credit card accounts E) savings accounts Answer: B Explanation: B) Time deposits are intended to be held until maturity. Page Ref: 390 Difficulty: Easy Objective: 15.1 Learning Outcome: Discuss the functions of different types of financial institutions in the monetary system Skill: Concept 35) Which of the following financial institutions is considered a thrift institution? A) pension fund B) credit union C) commercial bank D) savings and loan association E) securities investment firms Answer: B Explanation: B) Credit unions formed to help certain groups of people manage their money with more thrift. Page Ref: 392 Difficulty: Easy Objective: 15.2 Learning Outcome: Discuss the functions of different types of financial institutions in the monetary system Skill: Application 36) The Norton County Bank performs such tasks as making monthly bill payments, managing investment portfolios, and managing the estates of deceased persons. What are such services called? A) special banker's services B) financial advice services C) public services D) trust services E) amalgamation services Answer: D Explanation: D) For a fee, a bank's trust department will perform all of these functions. Page Ref: 393 Difficulty: Easy Objective: 15.2 Learning Outcome: Discuss the functions of different types of financial institutions in the monetary system Skill: Application 37) Which of the following agencies guarantees the safety of all of its members' bank accounts? A) Internal Revenue Service B) Federal Open Market Committee C) Federal Reserve Bank D) Federal Deposit Insurance Corporation E) Federal Exchange Commission Answer: D Explanation: D) This is the primary responsibility of this agency. Page Ref: 396 Difficulty: Easy Objective: 15.3 Learning Outcome: Discuss the functions of different types of financial institutions in the monetary system Skill: Concept 38) Which of the following terms refers to the percentage of its deposits a bank must hold, in cash or on deposit, with a Federal Reserve bank? A) discount rate B) key rate C) prime rate D) federal insurance premium E) reserve requirement Answer: E Explanation: E) This is controlled by the Fed. Page Ref: 398 Difficulty: Easy Objective: 15.4 Learning Outcome: Discuss the functions of different types of financial institutions in the monetary system Skill: Concept 39) Which of the following terms refers to the interest level at which member banks can borrow money from the Federal Reserve System? A) discount rate B) key rate C) prime rate D) federal insurance premium E) reserve requirement Answer: A Explanation: A) This is one of the many interest controls that the Fed has at its disposal. Page Ref: 399 Difficulty: Easy Objective: 15.4 Learning Outcome: Discuss the functions of different types of financial institutions in the monetary system Skill: Concept 40) Which of the following terms refers to the interest level at which commercial banks lend reserves to each other, usually overnight? A) discount rate B) key rate C) prime rate D) federal insurance premium E) reserve requirement Answer: B Explanation: B) The Fed has no direct control over this. Page Ref: 399 Difficulty: Easy Objective: 15.4 Learning Outcome: Discuss the functions of different types of financial institutions in the monetary system Skill: Concept 41) Which of the following best describes what is called the "open-market operations" of the Federal Reserve System? A) the development of new financial opportunities for banks B) the minting of new money to disperse into the money market C) the control of interest rates among banks D) the maintenance of a well-functioning lending system among banks E) the buying and selling of securities Answer: E Explanation: E) This allows the Fed to ensure an active market. Page Ref: 399 Difficulty: Easy Objective: 15.4 Learning Outcome: Discuss the functions of different types of financial institutions in the monetary system Skill: Concept 42) Which of the following indices indicates the relative strength of the currencies of two countries? A) trade rate B) exchange rate C) trade balance D) relative gross national products E) international loan rate Answer: B Explanation: B) This is the ratio of one currency against the other. Page Ref: 403 Difficulty: Easy Objective: 15.6 Learning Outcome: Discuss the functions of different types of financial institutions in the monetary system Skill: Concept 43) Toyota, based in Japan, would like to build a new factory and a new retail distribution center. Both or either of these may be built in the United States and in Canada. Currently, the U.S. dollar is weak compared to the Canadian dollar. Which of the following would be in Toyota's best interest? A) build the cars in Canada and sell them in the United States B) build the cars in the United States and sell them in Canada C) build the cars in Canada and sell them in both countries D) build the cars in the United States and sell them in both countries E) build the cars in both countries and sell them in both countries Answer: B Explanation: B) Since transportation costs would be negligible here, it would behoove Toyota to build the cars where it would be cheaper to build them and to sell them where they could get the higher price. Page Ref: 404 Difficulty: Moderate Objective: 15.6 Learning Outcome: Discuss the functions of different types of financial institutions in the monetary system Skill: Application 44) Which of the following best describes the portability characteristic of money? A) It neither dies nor spoils and if it wears out, it can be replaced. B) Units of money can be matched with the value of goods. C) It can be exchanged across national borders. D) It allows people to measure the relative value of goods and services. E) It is light and easy to handle. Answer: E Explanation: E) Money is easy to carry, which makes it easy to use. Page Ref: 388 Difficulty: Moderate Objective: 15.1 Learning Outcome: Discuss the functions of different types of financial institutions in the monetary system Skill: Concept 45) Which of the following is a consequence of the divisibility characteristic of money? A) Units of money do not expire after a certain time limit. B) Units of money can be accurately matched with the value of goods. C) Units of money are easily carried. D) Units of money have relatively stable value. E) Units of money of different nations are easily converted to other units of money. Answer: B Explanation: B) Modern currency is easily divisible into smaller parts, each with a fixed value; one dollar can be exchanged for 10 dimes. Thus, the actual value of goods can be approximated very well by units of money. Page Ref: 388 Difficulty: Moderate Objective: 15.1 Learning Outcome: Discuss the functions of different types of financial institutions in the monetary system Skill: Concept 46) Which of the following affects the value of money? A) how much currency is in circulation B) how many units the currency can be divided into C) how portable the form of currency is D) how durable the form of currency is E) how much currency has been saved Answer: A Explanation: A) If there is too much money in circulation, we end up with inflation. If there is too little, everyone starves. Page Ref: 389 Difficulty: Moderate Objective: 15.1 Learning Outcome: Discuss the functions of different types of financial institutions in the monetary system Skill: Concept 47) Which of the following best describes the durability characteristic of money? A) Units of money do not expire after a certain time limit. B) Units of money can be matched with the value of goods. C) Units of money allow people to measure the relative value of goods and services. D) Units of money of different nations are easily converted to other units of money. E) Units of money have lasting value. Answer: A Explanation: A) In addition, currency is difficult to counterfeit. Page Ref: 388 Difficulty: Moderate Objective: 15.1 Learning Outcome: Discuss the functions of different types of financial institutions in the monetary system Skill: Concept 48) Which of the following best describes the stability characteristic of money? A) Units of money do not expire after a certain time limit. B) Units of money of different nations are easily converted to other units of money. C) Units of money can be matched with the value of goods. D) Units of money allow people to measure the relative value of goods and services. E) Units of money have relatively constant value. Answer: E Explanation: E) The value of currency fluctuates a little, though its value is related to what we can buy with it. Page Ref: 388 Difficulty: Moderate Objective: 15.1 Learning Outcome: Discuss the functions of different types of financial institutions in the monetary system Skill: Concept 49) Money frees society from a system of barter. In doing so, money is performing which of the following functions? A) store of value B) measure of worth C) unit of account D) determination factor E) medium of exchange Answer: E Explanation: E) We use money as a way of buying and selling things; without money, we would all be involved in a system of barter. Page Ref: 389 Difficulty: Moderate Objective: 15.1 Learning Outcome: Discuss the functions of different types of financial institutions in the monetary system Skill: Concept 50) Which of the following is NOT part of the M-1 supply? A) currency at hand B) currency amount of written checks C) currency available through a debit card D) currency in checking accounts E) currency value of time deposits Answer: E Explanation: E) M-1 includes only the most liquid forms of money: cash, checks, and checking accounts. Page Ref: 389 Difficulty: Moderate Objective: 15.1 Learning Outcome: Discuss the functions of different types of financial institutions in the monetary system Skill: Concept 51) Which of the following is NOT part of the M-2 money supply? A) debit card account B) time deposits C) money market mutual funds D) savings account E) credit card account Answer: E Explanation: E) The major components of M-2 are M-1, time deposits, money market mutual funds, and savings accounts. Page Ref: 390 Difficulty: Moderate Objective: 15.1 Learning Outcome: Discuss the functions of different types of financial institutions in the monetary system Skill: Concept 52) Which of the following financial institutions earn profits from loans taken from the funds in each of its depositors' accounts? A) securities investment firms B) pension funds C) credit unions D) commercial banks E) mutual savings banks Answer: D Explanation: D) This is one of the primary ways that commercial banks earn a profit. Page Ref: 391 Difficulty: Moderate Objective: 15.2 Learning Outcome: Discuss the functions of different types of financial institutions in the monetary system Skill: Concept 53) Which of the following statements is true? A) Since insurance companies generate the vast majority of revenue from investments of premium payments, market fluctuations will not affect one's health care benefits. B) Since insurance companies generate the vast majority of revenue from premium payments, market fluctuations generally do not affect one's health insurance premium. C) Since insurance companies also generate revenue from investments of premium payments, market fluctuations may affect one's health insurance premium. D) Since insurance companies also generate revenue from investments of premium payments, market fluctuations may affect one's health care benefits. E) Since insurance companies generate revenue from both premium payments and investments thereof, market fluctuations will not generally affect one's health insurance benefits. Answer: C Explanation: C) Insurance premiums are set to make a profit, not simply to have an expected break-even value (as they were originally meant to do). Thus, if market fluctuations get too erratic, premiums might be set higher to compensate for this. Page Ref: 392 Difficulty: Moderate AACSB: Reflective thinking skills Objective: 15.2 Learning Outcome: Discuss the functions of different types of financial institutions in the monetary system Skill: Synthesis 54) Which of the following financial institutions are considered to be owned by their depositors? A) commercial banks and savings and loan associations B) savings and loan associations and mutual savings banks C) mutual savings banks and credit unions D) credit unions and pension funds E) pension funds and savings institutions Answer: C Explanation: C) These are the co-owned institutions. The others are owned by the investors. Page Ref: 391-392 Difficulty: Moderate Objective: 15.2 Learning Outcome: Discuss the functions of different types of financial institutions in the monetary system Skill: Concept 55) When buying from a supplier in France, LTD, based in Cleveland, Ohio, has arranged for its bank to pay the supplier the cost of the desired material when it has passed customs in the United States. Which of the following terms is this type of bank promise called? A) banker's acceptance B) bank trust service C) certified check D) letter of credit E) currency exchange agreement Answer: D Explanation: D) A letter of credit is payable only after certain conditions are met. Page Ref: 393 Difficulty: Moderate Objective: 15.2 Learning Outcome: Discuss the functions of different types of financial institutions in the monetary system Skill: Application 56) David has a savings account and a home mortgage with a financial institution. Each year, this institution sends him a dividend check based on his accounts. Which of the following financial institutions is David using here? A) commercial bank B) savings institution C) credit union D) savings and loan association E) mutual savings bank Answer: E Explanation: E) In mutual savings banks, all depositors are considered owners of the bank; all profits are divided proportionately among depositors, who receive dividends. Page Ref: 391-392 Difficulty: Moderate Objective: 15.2 Learning Outcome: Discuss the functions of different types of financial institutions in the monetary system Skill: Application 57) Which of the following actions can the Federal Deposit Insurance Company take in the event of a failure of one of its insured banks? A) The FDIC can seize the assets of the bank and its investors and settle the bank's debts. B) The FDIC can allow the bank to stay afloat by granting a loan of federal money. C) The FDIC can conduct an inquiry into the investors' assets and actions to determine if there was any malfeasance that caused the bank failure. D) The FDIC can allow another bank to take responsibility for the failed bank's liabilities through sale of the failed bank. E) The FDIC can settle the bank's debts through its insurance deposit fund and regulate the bank's transactions more strictly. Answer: D Explanation: D) The FDIC can do one of two actions: sell the failed bank or pay off its debts by seizure of its assets (and not those of the investors). Page Ref: 397 Difficulty: Moderate Objective: 15.3 Learning Outcome: Discuss the functions of different types of financial institutions in the monetary system Skill: Application 58) Which of the following statements best explains how financial institutions create money? A) By opening new checking accounts and giving more people access to readily available cash, financial institutions expand the money supply. B) By issuing money through government contracts, financial institutions expand the money supply. C) By taking deposits and loaning out these funds, financial institutions expand the money supply. D) By collecting interest on its accounts through investments, financial institutions expand the money supply. E) By giving interest from its accounts to its clients, financial institutions expand the money supply. Answer: C Explanation: C) Further, the money supply expands because banks are allowed to loan out most (although not all) of the money they take in from deposits. Page Ref: 396 Difficulty: Moderate Objective: 15.3 Learning Outcome: Discuss the functions of different types of financial institutions in the monetary system Skill: Concept 59) Which of the following political bodies appoints the members of the board of governors for the Federal Reserve System? A) the Senate B) the House of Representatives C) the President of the United States D) the Council of American Governors E) the people of the United States Answer: C Explanation: C) The Fed's board of governors consists of seven members appointed by the President for overlapping terms of fourteen years. Page Ref: 397 Difficulty: Moderate Objective: 15.4 Learning Outcome: Discuss the functions of different types of financial institutions in the monetary system Skill: Concept 60) How are the individual Federal Reserve Banks organized? A) by size of financial institution members—small, medium, large, etc. B) by type of financial institution members—commercial banks, credit unions, etc. C) by geographical region—Southeast, Northeast, West, etc. D) by financial purpose—buying, lending, interest controlling, etc. E) by legislative districts—10th District of Massachusetts, 1st District of Colorado, etc. Answer: C Explanation: C) There are a total of 12 districts. Page Ref: 397 Difficulty: Moderate Objective: 15.4 Learning Outcome: Discuss the functions of different types of financial institutions in the monetary system Skill: Concept 61) Which of the following best describes two of the three primary functions of the Federal Reserve System? A) developing new financial markets and aiding banks in conducting their business B) aiding banks in conducting business and managing the U.S. money supply and interest rates C) managing the U.S. money supply and interest rates and monitoring interbank relationships D) monitoring interbank relationships and acting as a bank for the federal government E) acting as a bank for the federal government and developing new financial markets Answer: B Explanation: B) In addition, the Fed also clears checks—about 56 million each day—for commercial banks. Page Ref: 398 Difficulty: Moderate Objective: 15.4 Learning Outcome: Discuss the functions of different types of financial institutions in the monetary system Skill: Concept 62) Which of the following is NOT a primary tool for controlling the money supply? A) taxation B) reserve requirements C) discount rate controls D) credit controls E) open-market operations Answer: A Explanation: A) The Fed uses four primary tools for controlling the money supply: reserve requirements, discount rate controls, open-market operations, and selective credit controls. Page Ref: 398 Difficulty: Moderate Objective: 15.4 Learning Outcome: Discuss the functions of different types of financial institutions in the monetary system Skill: Concept 63) Which of the following statements best describes why U.S. Treasury securities have always been considered risk-free investments? A) They are backed by the U.S. government. B) They have a guaranteed rate of return. C) They are easily sold. D) They are readily available for purchase. E) They earn a great profit on the international money market. Answer: A Explanation: A) The other reasons here are all true because of the backing of the U.S. government. Page Ref: 399 Difficulty: Moderate Objective: 15.4 Learning Outcome: Discuss the functions of different types of financial institutions in the monetary system Skill: Concept 64) Which of the following laws has had significant effects in maintaining bank identity records? A) the Check Clearing for the 21st Century Act B) the USA Patriot Act C) the Bank Secrecy Act D) the Federal Reserve Act E) the Emergency Economic Stabilization Act Answer: B Explanation: B) The CIP is a part of this law. Page Ref: 401 Difficulty: Easy Objective: 15.5 Learning Outcome: Discuss the functions of different types of financial institutions in the monetary system Skill: Concept 65) Which of the following federal laws made transactions between banks significantly more efficient? A) the Check Clearing for the 21st Century Act B) the Bank Secrecy Act C) the USA Patriot Act D) the Federal Reserve Act E) the Economic Emergency Recovery Act Answer: A Explanation: A) It made electronic clearing of checks legal. Page Ref: 401 Difficulty: Easy Objective: 15.5 Learning Outcome: Discuss the functions of different types of financial institutions in the monetary system Skill: Concept 66) Which of the following statements best describes how the Automated Clearing House Network (ACH) maintains its integrity as a processor of electronic payments? A) The ACH network is an institution within the Federal Bank System and is regulated by strict government standards. B) The ACH network is a for-profit company and is regulated by its need to make a profit, which it can only do by strictly regulating itself. C) The ACH network is a financial institution and is thus regulated by its obligations to its investors. D) The ACH network is a not-for-profit association and is regulated by its independence from outside interests. E) The ACH network is a professional organization to which all financial institutions are obligated to contribute resources and is regulated by the interests of these institutions. Answer: D Explanation: D) It is no longer a part of NACHA, but as a non-profit, it cannot be swayed by the personal interests of the investors and other institutions. Page Ref: 401 Difficulty: Moderate Objective: 15.5 Learning Outcome: Discuss the functions of different types of financial institutions in the monetary system Skill: Concept 67) Which of the following financial institutions was taken over by the Federal House Finance Agency in 2008 as a part of the Emergency Economic Recovery Act? A) Lehman Brothers Holdings B) Bear Stearns Companies C) Federal Home Loan Mortgage Corporation D) Federal Home Loan Banks System E) Federal Agricultural Mortgage Corporation Answer: C Explanation: C) The Federal Home Loan Mortgage Corporation was taken over by the government in order to ensure that no more subprime loans and other abuses were made. Page Ref: 400 Difficulty: Moderate Objective: 15.5 Learning Outcome: Discuss the functions of different types of financial institutions in the monetary system Skill: Concept 68) Which of the following transactions is not included among Automated Clearing House payments? A) ATM deposits B) business-to-business electronic payments C) checks D) Internet-initiated debit card payments E) local tax payments Answer: A Explanation: A) This is handled by the individual banks. Page Ref: 401 Difficulty: Moderate Objective: 15.5 Learning Outcome: Discuss the functions of different types of financial institutions in the monetary system Skill: Concept 69) Which of the following pieces of information is not required by the USA Patriot Act for banks to collect for every customer? A) name B) address C) date of birth D) place of birth E) tax identification number Answer: D Explanation: D) This is the only one not necessary to be collected. Page Ref: 401 Difficulty: Moderate Objective: 15.5 Learning Outcome: Discuss the functions of different types of financial institutions in the monetary system Skill: Concept 70) Which of the following institutions most helps simplify transactions made among buyers and sellers in different countries? A) the U.S. State Department B) the International Monetary Fund C) the World Bank D) commercial banks E) the Internet Answer: D Explanation: D) If money inflows and outflows are equal for two countries, money does not have to flow between them; if inflows and outflows are not equal, a flow of money is made to cover the difference. Page Ref: 404 Difficulty: Moderate Objective: 15.6 Learning Outcome: Discuss the functions of different types of financial institutions in the monetary system Skill: Concept 71) Which of the following is the main justification for the World Bank to fund national improvements by making loans to build roads and hospitals? A) The resulting improvements will help stabilize the political situation, and hence the international trade, of certain developing countries. B) The resulting improvements will help increase the productive capacity, and hence the international trade, of certain developing countries. C) The resulting improvements will help increase the population morale, and hence the international trade, of certain developing countries. D) The resulting improvements will help increase the overall quality of life, and hence the international trade, of certain developing countries. E) The resulting improvements will help increase the value of the currency, and hence the international trade, of certain developing countries. Answer: B Explanation: B) The World Bank provides only a very limited scope of services; the resulting improvements, however, eventually enable borrowing countries to increase productive capacity and international trade. Page Ref: 405 Difficulty: Moderate Objective: 15.6 Learning Outcome: Discuss the functions of different types of financial institutions in the monetary system Skill: Concept 72) Which of the following describes the requirements that the International Monetary Fund has made on certain developing countries? A) that the government decrease spending in order to bring inflation under control B) that private industries decrease prices in order to bring inflation under control C) that the government increase loans in order to create more money in the local markets D) that the local banks increase loans in order to create more money in the local markets E) that the government and private industries partner in creating more jobs in the region Answer: A Explanation: A) The IMF likes austerity in governments, but not private industries. Page Ref: 405 Difficulty: Moderate Objective: 15.6 Learning Outcome: Discuss the functions of different types of financial institutions in the monetary system Skill: Concept 73) David is saving $25 a week toward getting a new truck. Which function of money does this illustrate? A) stabilization of value B) foundation of banking C) medium of exchange D) store of value E) measure of worth Answer: D Explanation: D) In the form of currency, money can be used for future purchases and thus "stores" value. Page Ref: 389 Difficulty: Difficult Objective: 15.1 Learning Outcome: Discuss the functions of different types of financial institutions in the monetary system Skill: Application 74) A jeweler assesses the value of a flawless white diamond as considerably more than that of a rhinestone. Which function of money does this illustrate? A) foundation of banking B) medium of exchange C) store of value D) measure of worth E) stabilization of value Answer: D Explanation: D) Money lets us measure the value of goods and services. Page Ref: 389 Difficulty: Difficult Objective: 15.1 Learning Outcome: Discuss the functions of different types of financial institutions in the monetary system Skill: Application 75) In which of the following financial institutions would all of the people who allow it to hold their funds be able to participate in major decision-making? A) commercial bank B) savings and loan association C) savings institutions D) credit union E) pension fund Answer: D Explanation: D) A credit union is a nonprofit institution owned and run by its members. Page Ref: 391-392 Difficulty: Difficult Objective: 15.2 Learning Outcome: Discuss the functions of different types of financial institutions in the monetary system Skill: Concept 76) Which of the following reasons is the primary motivator for a commercial bank to acquire new depositors? A) A new deposit account will make more funds available to pay the interest on other deposit accounts. B) A new deposit account will make more funds available to pay the bank's employees. C) A new deposit account will make more funds available to give out in loans. D) A new deposit account will make more funds available to give out in dividends to its investors. E) A new deposit account will make more funds available to facilitate brokerage transactions. Answer: C Explanation: C) A commercial bank makes its profits from investments and loans derived from its depositors' cash. Page Ref: 391 Difficulty: Difficult Objective: 15.2 Learning Outcome: Discuss the functions of different types of financial institutions in the monetary system Skill: Concept 77) Suppose a list of all American deposit institutions were created and a name chosen randomly from this list. Which of the following would the chosen institution most likely be? A) a commercial bank B) a savings institution C) a savings and loan association D) a mutual savings bank E) a credit union Answer: A Explanation: A) Despite the plethora of possible deposit financial institutions, the overwhelming majority of them are commercial banks. Page Ref: 391-392 Difficulty: Difficult Objective: 15.2 Learning Outcome: Discuss the functions of different types of financial institutions in the monetary system Skill: Concept 78) Which of the following statements best describes why a decrease in reserve requirements often results in an increase in the money supply? A) A decrease in reserve requirements gives banks more money to better determine their interest rates. B) A decrease in reserve requirements gives banks more money to pay off their debts. C) A decrease in reserve requirements gives banks more money to lend out. D) A decrease in reserve requirements gives banks more money to offer in dividend payments to their clients. E) A decrease in reserve requirements gives banks more money to reward their employees for meritorious financial ventures. Answer: C Explanation: C) A decrease in the reserve requirement "frees up" extra money that can be distributed in the form of loans, for example. Page Ref: 398 Difficulty: Moderate Objective: 15.4 Learning Outcome: Discuss the functions of different types of financial institutions in the monetary system Skill: Concept 79) Which of the following statements best describes why an increase in the discount rate often results in a decrease in the money supply? A) An increase in the discount rate will lower the overall values of loans from the Federal Reserve Banks to individual banks. B) An increase in the discount rate will lower the overall values of loans from individual banks to other financial institutions. C) An increase in the discount rate will lower the overall number of banks issuing loans to individuals. D) An increase in the discount rate will lower the overall number of loans from the Federal Reserve Banks to individual banks. E) An increase in the discount rate will lower the overall number of loans from the Federal Reserve Banks to individuals. Answer: D Explanation: D) An increase in the discount rate increases the amount of money each local bank will pay to the Federal Reserve bank to borrow money, for example; this tightens money supply. Page Ref: 397 Difficulty: Moderate Objective: 15.4 Learning Outcome: Discuss the functions of different types of financial institutions in the monetary system Skill: Concept 80) Under which of the following circumstances would the Federal Reserve System want to increase the money supply by increasing the discount rate? A) The economy is experiencing severe inflation, and market activity is very low. Yet the financial institutions have a great deal of M-1 resources at their disposal that they are not investing. B) The economy is experiencing severe inflation, and most financial institutions have low M-1 resources at their disposal. Yet there is a great deal of market activity. C) The economy is experiencing severe deflation, and market activity is very low. Yet the financial institutions have a great deal of M-1 resources at their disposal that they are not investing. D) The economy is experiencing severe deflation, and most financial institutions have low M-1 resources at their disposal. Yet there is a great deal of market activity. E) The economy is experiencing wild fluctuations, and most financial institutions have low M-1 resources at their disposal. Answer: A Explanation: A) The Fed helps counteract inflation by decreasing the money supply. By increasing the discount rate, the Fed is discouraging further loans from itself and encouraging the institutions to use their own funds for investment. Page Ref: 399 Difficulty: Difficult AACSB: Analytic skills Objective: 15.4 Learning Outcome: Discuss the functions of different types of financial institutions in the monetary system Skill: Application 81) Under which of the following circumstances would the Federal Reserve System want to decrease the money supply by increasing the reserve requirement? A) The economy is experiencing severe inflation, and market activity is very low. Yet the financial institutions have a great deal of M-1 resources at their disposal that they are not investing. B) The economy is experiencing severe inflation, yet market activity has been extremely high and erratic. C) The economy is experiencing severe deflation, and market activity is very low. Yet the financial institutions have a great deal of M-1 resources at their disposal that they are not investing. D) The economy is experiencing severe deflation, yet market activity has been extremely high and erratic. E) The economy is experiencing severe deflation, market activity is very low, and most financial institutions are low on M-1 resources. Answer: B Explanation: B) The Fed helps counteract inflation by decreasing the money supply. Here the market needs to be stabilized. By increasing the reserve requirement, the Fed will lower the amount of investments made by the financial institutions. Page Ref: 398 Difficulty: Difficult AACSB: Analytic skills Objective: 15.4 Learning Outcome: Discuss the functions of different types of financial institutions in the monetary system Skill: Application 82) Which of the following recent events represents the two most significant impacts on changes in the money and banking system since 2000? A) the rise in government intervention in helping to stabilize the U.S. financial system and the increasingly strict regulation of the mortgage industry B) the increasingly strict regulation of the mortgage industry and the enactment of anti-terrorist policies C) the enactment of anti-terrorist policies and the rise of electronic technologies D) the rise of electronic technologies in banking and the rise in government intervention in helping to stabilize the U.S. financial system E) the rise in government intervention in helping to stabilize the U.S. financial system and the enactment of anti-terrorist policies Answer: D Explanation: D) These changes have been the most far-reaching in banking practices. Page Ref: 400-401 Difficulty: Difficult AACSB: Reflective thinking skills Objective: 15.5 Learning Outcome: Discuss the functions of different types of financial institutions in the monetary system Skill: Synthesis 83) Regarding the international payments process, which statement is true? A) In international trade, money will flow from the country with the strongest currency to that of the weakest. B) The World Bank monitors the trade of its members to insure that money is flowing properly between the various countries. C) The World Bank monitors the national banks of its members to insure that money is flowing properly between the various countries. D) By international law, international trade must involve money flowing between two countries. E) Money does not actually have to flow between two countries. Answer: E Explanation: E) If trade between the two countries is in balance, meaning that money inflows and outflows are equal for both countries, money does not have to flow between the two countries. Page Ref: 404 Difficulty: Difficult Objective: 15.6 Learning Outcome: Discuss the functions of different types of financial institutions in the monetary system Skill: Concept Chapter 16 Managing Finances 26) What value of a stock is calculated by dividing total shareholders' equity by the total number of shares of stock issued? A) par value B) market value C) book value D) chime value E) total value Answer: C Explanation: C) Book value is usually less than market value. Page Ref: 417 Difficulty: Easy Objective: 16.1 Learning Outcome: Discuss the functions of different types of financial institutions in the monetary system Skill: Concept 27) Which of the following statements is NOT true about dividend payments? A) Many companies distribute between 30 and 70 percent of their profits to shareholders. B) Some firms, especially fast-growing companies, do not pay dividends. C) Many fast-growing companies use cash earnings for expanding the company so that future earnings can grow even faster. D) During unprofitable years, companies are still required by law to pay dividends to their stockholders. E) Dividend payments are made to stockholders on a regular basis. Answer: D Explanation: D) Companies are not compelled by law to pay dividends; stockholders receive dividends only if the company makes money. Page Ref: 418 Difficulty: Easy Objective: 16.1 Learning Outcome: Discuss the functions of different types of financial institutions in the monetary system Skill: Concept 28) Which of the following is an advantage of ETFs over mutual funds? A) ETFs can be traded throughout the day like a stock. B) ETFs have high operating expenses. C) ETFs do not require high initial investments. D) ETFs are not regulated by the government. E) ETFs are difficult to sell in a timely manner. Answer: A Explanation: A) ETFs are traded on the stock exchange, just like stocks. Page Ref: 419 Difficulty: Easy Objective: 16.2 Learning Outcome: Describe the goals and stages of financial management for companies Skill: Concept 29) Which of the following are stocks, bonds, and mutual funds representing secured, or financially viable, claims on the part of investors? A) private placements B) securities C) exchanges D) commodities E) dividends Answer: B Explanation: B) The name "securities" comes from the fact that these represent secured, or financially viable, claims on the part of investors. Page Ref: 420 Difficulty: Easy Objective: 16.3 Skill: Concept 30) New stocks and bonds are sold in what market? A) private placement market B) secondary market C) primary market D) tertiary market E) open market Answer: C Explanation: C) Sales may be public or private in the primary market. Page Ref: 420 Difficulty: Easy Objective: 16.3 Skill: Concept 31) Which of the following is the federal government agency that regulates U.S. securities markets? A) New York Stock Exchange B) Federal Reserve C) Commerce Department D) Securities and Exchange Commission E) Treasury Department Answer: D Explanation: D) The SEC must approve a new security before it is brought to market. Page Ref: 420 Difficulty: Easy Objective: 16.3 Learning Outcome: Discuss the functions of different types of financial institutions in the monetary system Skill: Concept 32) Which of the following is an organization of individuals coordinated to provide an institutional setting in which stock can be bought and sold? A) stock exchange B) brokerage C) server D) over-the-counter market E) commission Answer: A Explanation: A) Each exchange regulates where and when trading may occur. Page Ref: 420 Difficulty: Easy Objective: 16.3 Learning Outcome: Discuss the functions of different types of financial institutions in the monetary system Skill: Concept 33) Which of the following is a requirement for a firm to be listed on the New York Stock Exchange? A) capital value of at least $35 million B) total value of outstanding stock C) number of shareholders must exceed 100 D) number of shares of stock must exceed 1 million E) company must be headquartered in North America Answer: B Explanation: B) The number of shares of stock offered by firms listed on the NYSE varies widely, and there is no requirement for the number. Page Ref: 421 Difficulty: Easy Objective: 16.3 Skill: Concept 34) Which of the following receives and executes buy-and-sell orders for other people in return for commissions? A) stock exchanges B) stock brokers C) investment banks D) stock agents E) savings banks Answer: B Explanation: B) Stock brokers usually work on commission. Page Ref: 421 Difficulty: Easy Objective: 16.3 Skill: Concept 35) Lito is working with a discount broker. Which service would Lito NOT be likely to receive? A) stock research B) industry analysis C) face-to-face consultations D) screening for specific types of stocks E) quality suggestions for investments Answer: C Explanation: C) Although such consultations are not offered, discount brokers can be much less expensive than traditional brokers. Page Ref: 422 Difficulty: Easy Objective: 16.3 Skill: Application 36) Which of the following is a summary of price trends in a specific industry and/or the stock market as a whole? A) investment index B) average index C) market indicator D) market index E) prospectus Answer: D Explanation: D) Market indexes do not indicate the status of individual securities. Page Ref: 423 Difficulty: Easy Objective: 16.3 Skill: Concept 37) Hugh is an investor who opts for no-risk U.S. Treasury Bills and intermediate-term high-grade corporate bonds. What kind of investor is Hugh? A) professional B) aggressive C) amateur D) conservative E) sporadic Answer: D Explanation: D) Generally speaking, the higher the risk, the higher the return. Page Ref: 426 Difficulty: Moderate Objective: 16.4 Skill: Application 38) If you purchased a share of GE stock for $33.82, then sold it one year later for $36.50, what would the price appreciation be? A) $1.68 B) $2.68 C) $33.82 D) $36.50 E) $67.24 Answer: B Explanation: B) Price appreciation is an increase in the dollar value of an investment, thus you would subtract $33.82 from $36.50. Page Ref: 427 Difficulty: Easy AACSB: Analytic skills Objective: 16.4 Skill: Application 39) Kim has investments in stocks, and wants to calculate her total return. What should she do? A) Divide the yearly dollar amount of dividend income by the investment's current market value. B) Compare the dividend against current yields from other investments. C) Subtract the cost of the stock from what she sold it for. D) Divide the current dividend payment plus capital gain by the original investment. E) Subtract current liabilities from current assets. Answer: D Explanation: D) Total return is expressed as a percentage. Page Ref: 427 Difficulty: Easy Objective: 16.4 Skill: Application 40) What is the formula for measuring a firm's working capital? A) current assets = working capital /current liabilities B) working capital = current assets - current liabilities C) current liabilities = current assets + working capital D) working capital = current assets x current liabilities E) current liabilities = current assets / working capital Answer: B Explanation: B) Subtracting liabilities from assets lets you know how much money a firm has to work with. Page Ref: 429 Difficulty: Easy Objective: 16.5 Skill: Concept 41) Which of the following is a formal pledge (an IOU) obligating the issuer to pay interest periodically and repay the principal at maturity (a preset future date) to the lender? A) commercial bank loan B) corporate bond C) mutual fund D) bond indenture E) prospectus Answer: B Explanation: B) Bondholders have no claim to ownership in the firm and do not receive dividends. Page Ref: 430 Difficulty: Easy Objective: 16.5 Skill: Concept 42) What is a stock's market value? A) the current price of a share of stock in the stock market B) the average price of one share of stock over the past year C) the face value of a share of stock, set by the issuing company's board of directors D) the price of the stock plus the previous year's dividend E) the monthly average of a stock's price over a year Answer: A Explanation: A) Market value represents what buyers are willing to invest in a firm. Page Ref: 417 Difficulty: Moderate Objective: 16.1 Skill: Concept 43) What is a stock's book value? A) the current price of a share of stock in the stock market B) the average price of one share of stock over the past year C) the face value of a share of stock, set by the issuing company's board of directors D) the value of a common stock expressed as total stockholders' equity divided by the total number of shares of stock issued E) the value of a preferred stock as expressed by the number of stock shares issued by the board of directors Answer: D Explanation: D) Book value is used as a comparison indicator because the market value for successful companies is usually greater than its book value. Page Ref: 417 Difficulty: Moderate Objective: 16.1 Skill: Concept 44) Don Dillon is a 24-year-old graphic designer. He eventually wants to get married, raise a family, and buy a house. He knows that he has to start putting money away so that his goals can someday become a reality. He has heard that investing is risky because you can lose money as well as make money. So Don decides to play it safe and deposit money in a bank on a regular basis. Why might this not be his best strategy? A) Stock investments will always result in high returns. B) It pays to take chances with your money. C) You can actually lose money with bank savings due to inflation and taxes. D) People usually benefit from risky investments. E) Most savings accounts are insured by the FDIC. Answer: C Explanation: C) The interest on your savings is taxed, which is partly how you can lose money. Inflation can make the dollars in your savings account worth less over time. Page Ref: 416 Difficulty: Difficult Objective: 16.1 Learning Outcome: Discuss the functions of different types of financial institutions in the monetary system Skill: Critical Thinking 45) Which of the following is a function performed by an investment bank? A) handling a client's portfolio in the secondary market B) establishing relationships for a client with a commercial bank C) underwriting purchases of stocks and bonds D) arranging travel abroad for purchasers of foreign currencies E) providing insurance services for clients Answer: C Explanation: C) Investment banks advise companies on timing and financial terms of new issues, underwrite new securities, and create distribution networks for moving new securities through groups of other banks and brokers into the hands of individual investors. Page Ref: 420 Difficulty: Moderate Objective: 16.3 Learning Outcome: Discuss the functions of different types of financial institutions in the monetary system Skill: Concept 46) Peter Kim wanted to buy a new car. To help finance the purchase, he decided to sell his Organic Markets bond in the secondary market. Peter's bond had a par value of $10,000 and a coupon of 6 percent. Current interest rates were 3 percent. What would Peter's bond sell for? A) for par value B) for $10,600 C) at a discount to par value D) for $9,700 E) at a premium to par value Answer: E Explanation: E) Bonds trade in the secondary market at prices at a premium or discount to par value. A bond whose coupon is higher than prevailing interest rates will sell at a premium to its par value. Page Ref: 420 Difficulty: Difficult Objective: 16.3 Learning Outcome: Discuss the functions of different types of financial institutions in the monetary system Skill: Application 47) Hector Rivera was interested in buying shares of ABC Computer Corp, which was currently trading at $50 per share. Analysts expected the share price to rise sharply in the next few months. Hector decided to buy a call option for $400, giving him the right to purchase 100 shares of ABC Computer for $60 per share at any time between his purchase and the option's expiration date. Three months later at the expiration date, ABC stock was trading at $73. Hector used his option and purchased 100 shares of ABC Computer. How much money could Hector have saved had he bought the shares three months earlier instead of the call option? A) $1,400 B) $400 C) $1,000 D) $2,300 E) $200 Answer: A Explanation: A) By buying the call option for $400 and then the stock shares for $6,000, Hector spent $6,400. Had he bought the shares in the first place, he would have spent only $5,000 and therefore would have saved $1,400. Page Ref: 420 Difficulty: Difficult AACSB: Analytic skills Objective: 16.3 Learning Outcome: Discuss the functions of different types of financial institutions in the monetary system Skill: Application 48) Which of the following is considered by many to be the best single indicator of the U.S. equities market? A) NASDAQ Composite B) Russell 2000 C) Dow Jones Industrial Average D) Standard & Poor's 500 E) New York Stock Exchange Answer: D Explanation: D) The S&P 500 considers more companies than the Dow Jones, and so is regarded as a better indicator. Page Ref: 424 Difficulty: Easy Objective: 16.3 Skill: Concept 49) Which of the following is an electronic securities exchange? A) Pacific Stock Exchange B) NASDAQ C) Osaka Securities Exchange D) Securities and Exchange Commission E) Europa Answer: B Explanation: B) The NASDAQ telecommunications system operates the NASDAQ Stock Market by broadcasting trading information on an intranet to more than 350,000 terminals worldwide. Page Ref: 421 Difficulty: Moderate Objective: 16.3 Skill: Concept 50) Which stock exchange has the highest volume of shares traded? A) American Stock Exchange B) New York Stock Exchange C) NASDAQ D) Philadelphia Stock Exchange E) SEC Answer: C Explanation: C) Although the volume of shares traded surpasses that of the New York Stock Exchange, the total market value of NASDAQ's U.S. stocks is less than that of the NYSE. Page Ref: 421 Difficulty: Moderate Objective: 16.3 Skill: Concept 51) Twenty-six-year-old Lisa Lamont became an investor one day after a friend explained that there was a much better way to make her money work for her than by letting it sit in a savings account earning a very low rate of interest. Lisa decided to withdraw $10,000 from her savings account and invest it in stocks. She wanted to invest it in such a way as to minimize her risk. Which one of the following strategies makes the most sense for a new investor like Lisa? A) Invest all $10,000 in a small fast-growing computer company. B) Invest $5,000 in a gold mining company and $5,000 in a silver mine. C) Invest the $10,000 in four different home builders. D) Invest $10,000 in ten different oil companies. E) Invest $10,000 in a large-cap mutual fund. Answer: E Explanation: E) As a person investing in stocks for the first time, the possibility of losing money can be scary. For peace of mind, the safest way to go is to choose maximum diversification. A large-cap mutual fund offers this diversification, and large-cap companies offer potential capital gains with less risk than small-cap companies. Page Ref: 420 Difficulty: Moderate Objective: 16.3 Learning Outcome: Discuss the functions of different types of financial institutions in the monetary system Skill: Critical Thinking 52) Which of the following allows after-hours trading and protects traders' anonymity? A) National Association of Securities Dealers Automated Quotation (NASDAQ) B) New York Stock Exchange (NYSE) C) Electronic Communication Networks (ECNs) D) Standard & Poor's 500 E) mutual funds Answer: C Explanation: C) ECNs must register with the SEC as broker-dealers. Page Ref: 422 Difficulty: Moderate Objective: 16.3 Skill: Concept 53) What is the term for buying several different kinds of investments rather than just one? A) spreading B) shorting C) margin buying D) diversification E) management Answer: D Explanation: D) The risk of loss is reduced by spreading the investment across stocks. Page Ref: 427 Difficulty: Moderate Objective: 16.4 Skill: Concept 54) Javier wants to invest, but wants to reduce his risk of loss by spreading the total investment across more stocks. Javier would be most interested in which of the following? A) spreading B) shorting C) margin buying D) diversification E) asset allocation Answer: D Explanation: D) Diversification means buying several different kinds of investments rather than just one. Page Ref: 427 Difficulty: Moderate Objective: 16.4 Skill: Application 55) What is the term for the proportion of funds invested in each of several investment alternatives? A) spreading B) shorting C) margin buying D) asset allocation E) diversification Answer: D Explanation: D) For example, you may decide to allocate $10,000 to common stocks, $5,000 to a money market mutual fund, and $2,500 to a U.S. Treasury bond fund. Page Ref: 428 Difficulty: Moderate Objective: 16.4 Skill: Concept 56) Which of the following is figured by dividing the yearly dollar amount of dividend income by the investment's current market value? A) bond yield B) current dividend yield C) debenture D) trading volume E) EBIDA Answer: B Explanation: B) In 2008 each share of GE stock was receiving annual dividend payments of $1.24. If on a particular day the share price was $33.82, the current yield would be 3.66 percent ($1.24/$33.82 × 100). Page Ref: 426 Difficulty: Moderate Objective: 16.4 Skill: Concept 57) Marty and his sister Robin both invested in shares of the Heavy Metal Steel Company. Marty bought common shares and Robin bought the company's preferred shares. The two had been holding their stock for several years and had been pleased with their stock's performance. But sadly, Heavy Metal's business took a turn for the worse. After several quarters of disappointing numbers, the company suddenly announced it was going out of business. Marty and Robin, had they seen this coming, could have sold their shares at any time. But now it was too late. Which one of the following is the most likely outcome? A) Marty loses his entire investment, but Robin might receive a partial repayment after Heavy Metal's bondholders are repaid. B) Marty and Robin lose their entire investment. C) Robin is repaid immediately, while Marty is eventually repaid after the sale of all of the company's assets. D) Marty and Robin are repaid all of their initial investment due to a successful shareholder lawsuit. E) Marty and Robin's investments regain value once Heavy Metal restarts operations. Answer: A Explanation: A) As a common shareholder in Heavy Metal, Marty is likely to lose his entire investment. As a preferred shareholder, Robin is in a better position, being next in line for repayment after the company's bondholders. Page Ref: 417 Difficulty: Difficult Objective: 16.1 Learning Outcome: Discuss the functions of different types of financial institutions in the monetary system Skill: Application 58) Diamond Electronics has ascertained that it is has positive working capital. Which of the following must be true of Diamond Electronics? A) The firm's current liabilities are large enough to pay off current assets. B) The firm may need to borrow money from a commercial bank. C) The firm's current assets are large enough to pay off current liabilities. D) The firm does not need to issue stock. E) The firm would be a bad risk for any investor to consider. Answer: C Explanation: C) Working capital is calculated by subtracting current liabilities from current assets. Page Ref: 429 Difficulty: Moderate Objective: 16.5 Skill: Application 59) What is the term for a division of stock that gives stockholders a greater number of shares but does not change each individual's proportionate share of ownership? A) stock dividend B) stock split C) stock yield D) stock warrant E) stock exchange Answer: B Explanation: B) For example, a 2-for-1 split would indicate that 50 shares of a $10 stock become 100 shares of a $5 stock. Page Ref: 432 Difficulty: Moderate Objective: 16.6 Skill: Concept 60) The stock of Simonsen, Inc. is being bought up by an individual who intends to take over Simonsen and sell off its assets for a profit. What is the individual in this scenario called? A) angel investor B) corporate raider C) inside trader D) stock broker E) bondholder Answer: B Explanation: B) If a corporate raider succeeds, the company will disappear. Page Ref: 432 Difficulty: Moderate Objective: 16.6 Skill: Application 61) "I am a chemist with the nation's largest chemical company. We have secretly developed a cure for three of the world's most serious epidemics. Before this news breaks out, I'm going to buy as many shares of stock in my company as I can get my hands on. Then I'll sit back and watch the stock price head for the sky." What is the person making this statement about to engage in? A) selling shares of stock B) insider trading C) blue-sky trading D) open-market operations E) investment banking Answer: B Explanation: B) Insider trading occurs when individuals use special knowledge about a firm for profit or gain. Page Ref: 435 Difficulty: Moderate Objective: 16.7 Skill: Application 62) Before Allied Corporation sells securities, it is required by law to offer future investors the opportunity to read through a document which contains complete information on both the new issue and the corporations. What is that required document called? A) book report B) affidavit C) prospectus D) portfolio E) balance sheet Answer: C Explanation: C) Further, false statements are subject to criminal penalty. Page Ref: 436 Difficulty: Difficult Objective: 16.6 Skill: Application 63) Trina is a loan borrower, and the terms of her loan say she must keep a portion of the loan amount on deposit with the bank in a non-interest-bearing account. What is Trina required to have? A) a compensating balance B) an unsecured loan C) an angel investor D) venture capital E) collateral Answer: A Explanation: A) Such an account protects the bank's investment. Page Ref: 429 Difficulty: Moderate Objective: 16.5 Skill: Application 64) Jessica Jenkins, a 30-year-old history teacher, planned to invest $10,000 in an equity mutual fund. She wanted a fund with maximum geographical diversification. Jessica was seeking high capital gains from her investment and was comfortable with a fair amount of risk. Which one of the following types of mutual fund would Jessica most likely choose? A) large-cap technology fund B) global large-cap value fund C) global small-cap growth fund D) Asian large-cap growth fund E) foreign medium-cap growth fund Answer: C Explanation: C) A global fund gives maximum geographical diversification because it includes U.S. stocks as well as foreign stocks. And small-cap growth stocks have the potential of delivering higher capital gains than large-cap value stocks. Page Ref: 418 Difficulty: Moderate Objective: 16.2 Learning Outcome: Discuss the functions of different types of financial institutions in the monetary system Skill: Application 65) Which of the following is the name for a portion of the ownership of a corporation? A) Schedule C B) stock C) partnership D) LLC E) dividend Answer: B Explanation: B) A corporation will sell stocks, portions of ownership, in order to raise capital. Page Ref: 417 Difficulty: Easy Objective: 16.1 Skill: Concept 66) Which of the following statements is true about common stocks as investments? A) Stocks are among the riskiest of all investments. B) Stocks are most suitable for investors who seek security. C) Stocks will never become worth less than their purchase price. D) Stock prices reflect only government action. E) Stock prices are easily manipulated. Answer: A Explanation: A) Stocks are a volatile and risky investment, and crashes can lead to sizable losses. Page Ref: 417 Difficulty: Moderate Objective: 16.1 Skill: Concept 67) Shawna has gotten into investing and is interested in aggressive growth funds. Which of the following must be true about Shawna? A) She is interested in the best possible returns over time. B) She is interested in making money as quickly as possible. C) She has comparatively little money to invest. D) She wants to make as secure an investment as possible. E) She is investing for a financial thrill ride. Answer: A Explanation: A) The chance to make more money on an investment appeals to investors who are not risk averse. Page Ref: 426 Difficulty: Moderate Objective: 16.4 Skill: Application 68) The profit realized when the market value of an investment increases is known as which of the following? A) capital gains B) asset allocation C) diversification D) price appreciation E) profiteering Answer: A Explanation: A) The capital gain can be calculated by subtracting the purchase price from the end value of the investment. Page Ref: 427 Difficulty: Moderate Objective: 16.4 Skill: Concept 69) The time it takes to double an investment can be estimated by using which of the following? A) book value B) the Rule of 72 C) blue-chips D) dividends E) market value Answer: B Explanation: B) By dividing the interest rate by 72, you can estimate how many years it will take to double your investment. Page Ref: 416 Difficulty: Easy Objective: 16.1 Skill: Concept 70) Tim wants to invest in stocks and bonds that will double his money in seven years. Which of the following interest rates should he be targeting? A) 7% B) 7.2% C) 8% D) 10% E) 10.3% Answer: E Explanation: E) Using the Rule of 72, dividing 72 by 7 years gives an interest rate of 10.3% to double the money in 7 years. Page Ref: 416 Difficulty: Moderate AACSB: Analytic skills Objective: 16.1 Skill: Application 71) Marco is a bullish investor, but the stock market is currently bearish. What is Marco most likely to do? A) sell every stock and go to a 100 percent cash position B) sell certain winning stocks to realize a profit and put the cash to work when the market decline appears to have ended C) maintain all stock positions until the market completely recovers and then buy more stocks D) sell losing stocks to avoid further losses, and hold onto winners E) buy defensive stocks in the expectation that they will not fall much Answer: B Explanation: B) Many investors in a bear market become bearish on stocks and gloomy about the market's prospects. A bullish investor, however, will continue to believe in the market's eventual recovery. Such an investor will look for opportunities to sell certain stocks if any profits can be realized, and take the opportunity to buy stocks at low prices in anticipation of higher prices ahead once the market recovers. Page Ref: 423 Difficulty: Moderate Objective: 16.3 Learning Outcome: Discuss the functions of different types of financial institutions in the monetary system Skill: Application 72) While it's invested, money grows by earning interest or yielding some other form of return. What is this known as? A) compound growth B) dividend C) capital gains D) security E) diversity Answer: A Explanation: A) The compound growth comes from interest paid over time. Page Ref: 416 Difficulty: Moderate Objective: 16.1 Skill: Concept 73) Dynamo Motors has begun to sell corporate securities. Dynamo is required by law to offer future investors the opportunity to read a document that contains complete information on both the new issue and the corporation itself. What is that required document called? A) book report B) affidavit C) prospectus D) order blank E) corporate profile Answer: C Explanation: C) Further, false statements are subject to criminal penalty. Page Ref: 434 Difficulty: Moderate Objective: 16.6 Skill: Application 74) Which is the most widely cited U.S. stock market index? A) Standard & Poor's Composite Index of 500 Stocks B) the NYSE index C) Dow Jones Industrial Average D) Moody's Stock List E) the NASDAQ Answer: C Explanation: C) Further, the Dow measures the performance of U.S. financial markets by focusing on 30 blue-chip companies as reflectors of economic health. Page Ref: 423 Difficulty: Moderate Objective: 16.2 Skill: Concept 75) Tonio has some stocks and is carefully watching the stock market. Which of the following should he pay attention to in order to find out the performance of the largest companies? A) Standard & Poor's 500 B) the NYSE index C) Dow Jones Industrial Average D) Moody's Stock List E) the NASDAQ Answer: C Explanation: C) The Dow includes only 30 of the thousands of companies on the market, but these companies are the largest and most important. Page Ref: 423 Difficulty: Moderate Objective: 16.3 Skill: Application 76) Marcia has purchased some corporate bonds, which come with bond indentures that stipulate the terms. What is the most important detail of the bond indenture that Marcia should pay attention to? A) maturity date B) face value C) par value D) dividend payments E) amount invested Answer: A Explanation: A) This is the date by which the firm must repay the bondholder. Page Ref: 430 Difficulty: Moderate Objective: 16.5 Skill: Application 77) What are low gy os? A) bankruptcy B) default bonds C) junk bonds D) corporate bonds E) grade B bonds Answer: C Explanation: C) These bonds have very low credit ratings. Page Ref: 426 Difficulty: Easy Objective: 16.4 Skill: Concept [Show More]

Last updated: 1 year ago

Preview 1 out of 340 pages

Reviews( 0 )

$24.50

Add to cart

Instant download

Can't find what you want? Try our AI powered Search

OR

GET ASSIGNMENT HELP
90
0

Document information


Connected school, study & course


About the document


Uploaded On

Feb 20, 2021

Number of pages

340

Written in

Seller


seller-icon
A+ Solutions

Member since 3 years

164 Documents Sold


Additional information

This document has been written for:

Uploaded

Feb 20, 2021

Downloads

 0

Views

 90

Document Keyword Tags

Recommended For You


$24.50
What is Browsegrades

In Browsegrades, a student can earn by offering help to other student. Students can help other students with materials by upploading their notes and earn money.

We are here to help

We're available through e-mail, Twitter, Facebook, and live chat.
 FAQ
 Questions? Leave a message!

Follow us on
 Twitter

Copyright © Browsegrades · High quality services·